Saturday, November 29, 2008

Antibiotics

1. Semisynthetic penicillinase-resistant penicillins: Oxacillin, cloxacillin, dicloxacillin, nafcillin
when sensitive, better than vancomycin, also treat strep: s. pneumonia, Viridans, group A, B, C, G
MRSA: use vancomycin, or Linezolid, quinupristin/dalfopristin as alternatives

2. Penicillin G, penicillin VK, ampicillin, amoxcillin
strep + E. Coli + listeria + Neisseria

3.
Piperacillin, ticarcillin, mezlocillin
Enterobacteriacceae + pseudomonas

4. Ceph for pseudomonas: Only ceftazidime and cefepime

5. Penicillin insensitive pheumococci: cefatriaxone & cefataxime

6. Quinolones for pseudomonas: only ciprofloxacin

7. Exclusive Gram (-): gentamicin, tobramycin, amikacin, aztreonam

8. all: imipenem, meropenem, ertapenem(no pseudomonas)

9. Anaerobes: Metronidazole, clindamycin, carbapenem

UW Notes

Abrupta Placenta - 4
Pt presents with vaginal bleeding, ABDOMINAL PAIN, and uterine tenderness. The absence of hemorrhage DOES NOT rule out this Dx. DDX:Placenta Previa, absence of bleeding RULES OUT this dx.****Risk factors are:1-HT and preecclampsia, 2-Placental abruption in previous pregnancy, 3-trauma, 4-short umbilical cord, 6-COCAINE abuse. AP is the mcc of DIC in pregnancy, which results from a release of activated thromboplastin from the decidual hematoma in to maternal circulation.****Risk factors are smoking and,Folate def. It can progress rapidly so careful monitoring is mandatory. Once dx is made, large-bore IV , as well as Foley cathater is inserted. Pts with AP in LABOR should be managed aggressively to insure rapid vaginal delivery, since this will remove the inciting cause of DIC and hemorrhage. Now if pt is stable tocolysis with MgSO4 is considered, but remmeber Ritordin is CI in pt with HT. ***Again, once we dx the next step is Vaginal delivery with augmentation of labor if necessary. Now if mother and baby are not stable or if there is CI, then Emergency C-section is indicated. Now if there is Dystocia ( narrowing of the birth passage) then Forcepts can be used.

ABCD of Homeostasis:
1-AIRWAY: An airway is needed for all unconscious pts, in the ER best method is Orotrachial intubation and in the field its needle cricothyroidectomy. For consciouns pt the best airway is chin lift with face mask. 2-BREATHING: Cervical spine injury should be analyzed but the first step is to establish ABC. 3-CIRCULATION: It needs control of bleeding and restoring the BP. In most external injuries pressure is enough to stop bleeding but in case of scalp laceration suturing is needed. Also all pts with hypotension should receive rapid infusion of isotonic fluid like ringer lactate to prevent life threatening hypotension. If IV line is not good for adults do saphaneous vein cut down and for children intraosseous membrane cannulation.

Absence seizures - 3
Ethosuximide is tx. Now remmeber that Phenytoin and Carbamazapine are first line drug used for primary generalized tonic clonic sezure or partial seizures, both work by blocking Na channels voltage dependent, Phenytoin is a second drug line for myoclonic and tonic clonic seizure, its available in both IV and oral forms, SE is gingivial hypertrophy, lymphadenopathy, hirsutism and rash, Both Phenytoina & Carbamazepine can cause Steven Johnson synd and Toxic Epidermal Necrolysis.*****Tx is Ethusuxamide or VALPROATE. Classic EEG is symetric 3mhtz spike and wave .

Acarbose SE
It blocks carbohydrate break down in the intestinal tract. The most significant SE is GI disturbance due to increased undifested CHO in the stool.

ACE inhibitor SE, Respira, 6/2
CAPTOPRIL (Cough, Angioedema, Pregnancy, Taste change, hypOtention, Proteinuria,Rash, Increase renin, Lower AII) and HyperKalemia. Cough is caused by accumulation of Kinins possibly by activation of arachadonic acid pathway. Kinins are degraded by ACE, when there is noACE they increase.*****Angioedema that is seen in ER. Pt presents with non-inflamatory subcutaneous edema and laryngeal edema due to bradykinin stimulation.

Acetaminophen toxicity - 2
Acute alcoholic intake can reduce the risk of hepatic injury by Acetaminophen because it competes with CYP2E1, so there is less production of toxic metabolites. Chronic alcohol intake increases risk of hepatic injury by stimulating P450 system and decreasing the amount of Glutathione (used for metabolism of acetaminophen). Management process: 1-4-hr post ingestion AA levels are determined to decide whether the pt will benefit from NAC or not. 2-On the other hand if the pt has ingested >7.5 gr AA and levels will not be available w/i 8 hours of ingestion, he should be given the antidote.

Acetazolamide Toxicity
Causes normal anion gap metabolic accidosis due to renal loss of bicarbonate. Anion Gap is 140-(114+116)=10 which is normal anionic gap metabolic acidosis.

Achalasia - 3
Dx 1-Barium studies, 2-Esopgaguscopy 3-Manometry. ** the CONFIRMATION test is Manometry. We also need to do Endoscopy to rule out malignancy.

ACL Injury
It prevents gliding of tibia under femur. Injury is seen after Hyperextension. A poping sensation is felt at time of injury. Commonly asso with Medial Meniscus and Medial Colateral Ligament (TRIAD). Lachman test is a test for ACL tear. Flex and pull tibia. Drawer sign also test ACL but its less sensitive. Posterior Drawer sign tests PCL. Mc murry's sign tests Meniscus injury. Valgus test is for MCL.

Acne - 2
1-Comedons (black/white heads): cuase minimal inflamation and tx is topical retinoids. If reactivation occur add topical Erythromycin or Benzoyl peroxide. 2-Papular and inflamatory acne: with moderate-severe inflamation: Oral Doxycycline. 3-Nodular or scaring acne: Oral Isotretinoin.

Acromegaly

Actinomycosis
Cervicofacial actinomycosis presents as slowly progressing , non tender, indurated mass, which evolves into multiple abscesses, fistula, and draining sinus tracts with sulfur granules, which appear yellow. Actinomyces israelii is the agent, Tx is high dose IV peniciline for 6-12 weeks. Surgical debrement comes after penicillin therapy.

Acute adrenal insufficiency:
Acute onset of naseau, vomiting, abdominal pain and hypoglycemia and hypotension after a stressful event (surgery) in a pt sho is steroid dependant is typical. A clue is preoperative steroid use. Exogenous steroids depress pit-adrenal axis and a stressful situation can precipitate AAI. DDX: insulin induced hypoglycemia does not cause naseau and vomit and abdominal pain and hypotension.

Acute Alkali ingestion
When a pt takes Lye (alkali substance for suicide), upper GI contrast studies should be performed as eary as possible, to assess the damage and posible perforation of esophagus. Normal x-ray does not rule out a perforation. Once you know there is no perforation then you can do Diagnostic peritoneal lavage if necessary. But the first thing is to rule out perforation.

Acute Appendicitis - 3
Pt who comes to hospital after 5 days of initial symptoms must be hospitalized with IV hydration and IV Cefotetan. If threre is abcess with CT, percutaneous drainage is an option.****Most pelvic abscesses are due to perforation of AA. Pt might have a 24 hour RUQ pain that resoves spontaneously and then later on in a few days he might come with anal abscess symptoms. Drainage of the abscess is tx of choice.****Experiecne has shown that right hemiclectomy with ileo-transvers anatomosis has best postoperative results when resection of part of ascending is requires. And that is when pt has shown gangrenous rupture of appendix with questionable necrotized colon.

Acute Bacterial Proctatitis:
MCC in young is Chlamydia and Gonococcus, in old E. Coli. To diagnose do culture of mid-stream urine sample and start empiric therapy. Prostatic massage is contraindicated due to septecemia chance.

Acute GI bleeding
There are three causes: 1-Diverticulosis (Painless. can be ruled out with Barium Enema), 2-Angiodysplasia (Painless. maybe seen as cherry-red sopts that maybe coagulated, dx with labeled erythrocyte scintigraphy). 3-PUD (Painfull. Dx with endoscopy, if there is Hematochezia, red bright blood,due to lower GI bleed, then there is no need for endoscopy, the blood is from lower UGI bleeding).

Acute renal transplant rejection
Renal transplant rejection in the early post-operative stage can be expained by, ureteral obstruction, Acute rejection, Cyclosporine tox, vascular obstruction and ATN. To determine the cause we do US, MRI and Biopsy. If biopsy shows infiltration of lymphocytes and vasular swelling and there is increase Crt and Bun and oliguria, then the cause is Acute Rejection. Tx is high dose IV steriods.

Acute Tubular Necrosis
Prolonged hypotention due to any reason (Hypovolemic shock) can lead to ATN. Hallmark finding on urin analysis is Muddy brown granular cast. DDX1:RBC cast, GN. DDX2:WBC cast, Interstitial Nephritis and Pyelonephritis. DDX3:Fatty cast, NephrOtic Synd. DDX4:Broad and Waxy cast:Chronic renal failure.

Acyclovir Toxicity
Can cause crystalluria with renal tubular obstruction during high dose parenteral therapy, especially in inadequately hydrated pts.


Addison's Disease - 2
MM-101. Aldosterone def leads to non-anion gap hyperkalemic, hyponatremic metabolic acidosis. ***80% of pt have primary adrenal deficiency due to Autoimmune adrenalitis. These pts also present with autoimmune involvement of other glands as well, like thyroid,parathyroid, ovaries.*** 70% of the Causes of Primary Adrenal Insufficiency autoimmune, mostly in developed countries. In underdeveloped countries TUBERCULOSIS, Fungal infection and CMV infection are the mcc, TB is the MCC in undeveloped countries. Adrenal Calcification is a typical feature of TB PAI. Pt presents with no rise in serum cortisol following injection of Cosyntropin (ACTH analog), CT shows calcification of adrenal glands. Tx of TB does not result in normalization of adrenal gland. PAI in HIV pt is common, mcc is CMV. Sometimes Ketoconazole can cause it. PAI is very rare with adrenal tumor metastasis, even then calcification is not seen.

Adenomyosis
Is defined as presence of Endometrial glands in the uterine muscle. MF in women above 49, , presents with severe dysmenorrhea, and menorrhagia. The typica lexam reveals enlarged sysmetrical uterus. If Adenomyosis is in one side of uterus then enlargment is asymetrical. DDX includes Myomatous Uterus , Leomyoma, Endometrial carcioma. For women above 35, its mandatory to perform an Endometrial curetage or even hysterectomyto rule out endometrial cancer. DDX1:Endometriosis is a benign condition, where foci of endometrial glands are found OUTSIDE of endometrium. They increase in size throgh out menstrual cycle. Asso with Adenomyosis occurs in 15% of cases. DDX2:Leomyomas, are difficult to ddx from Adenomyosis, except that consistency of Uterus is softer in Adenomyosis. DDX3:Endometrial Carcinoma, occurs in women after menopause . DDX4:Endometritis manifest with fever, and enlarged and tender uterus, asso with vaginal discharge . It usually occurs after a septic abortion, and the mc oranism responsible is Strep.

ADHD
Short attention span, impulsivity, hyperactivity for >6mo. Tx is Methylphenidate, se is decreased appetite.

Adjustment Disorder - 2
Emotional or behavioral symptoms that develop w/I three months of exposure to an identifiable streesor and raely lasts more than 6 months after the stressor.The tx choice is Conginitve or Psychotherapy, not drugs. DDX is GAD where pt worries about many things , AD pt worry about one thing. DDX PTSD is when pt relives the trauma that she experienced, nightmare, flashbacks. FOR >1 month. DDX Acute Stress Disorder is PTSD but FOR <1 month

Adrenal insufficiency, 2ary
Is caused by Pituitary tumor. There is Hypothalamic-Pit failure. There is Glucocorticoid def (weakness, fatigue, depresion,irritability,hypotention, lymphocytosis,eosinophilia), and Hypothyroidism(cold intolerance,constipation, dryskin), while Normal K level indicated Aldosterone production is not impaired, and absence of Hyperpigmentation(characteristic of Primary adrenal insuff), all suggest 2ary adrenal insufficiency. Other causes of Primary adrenal insuff are:Autoimmune destruction,adrenal CMV, adrenal TB and adrenoleukodystrophy.

Adrenal insufficiency, Acute
Pt presents with nasea and vomitting, abdominla pain , hypoglycemia and hypotension. Preoperative steriod use is the main cause.

Adrenal Tuberculosis: Endo, 6/2
Adrenal insufficiency plus adrenal calcifications. It’s the primary cause of Primary Adrenal insuff in developing countries. In contrast autoimmune adrenalitis is the mcc of Primary Adrenal insufficiency in developed countries.




Airway assess
An airway is always patent(SECURE) when a pt is conscious and able to speak. If he is tachypnea and noisy respiration he needs chin lift and face mask. An airway is needed in ALL UNCONSIOUS pts. In the FIELD best option is needle Crricothyroidectomy. In ER best option is Orotrachial intubation. Nasotracheal is time consuming. Surgical cricothyroidectomy is a good choice for Apneac pts with head and spine injury.

Alcohol withdrawl
It might occur after surgey when pt has not had drinnk for a few days. Prestns with fever, HA, N&V and TREMORS. Tx is Chlordiazepoxide.

Alcoholic Gastritis
Pt presents with epigastic pain, vomitting dark brown blood after alcohol binge, and has a hx of PUD. A BUN level >40 in a presence of normal creatine is highly suggestive of upper GI bleed, its due to bacterial break down of Hb in the GIT and the resulting absorption of urea. Another place that causes increase BUN w/o Crt is in administration of steriods.

Alcoholic liver disease - 2
T9Q9. The three major pathological findgins of ALD are: 1-Fatty liver (steatosis). 2-Alcoholic Hepatitis. 3-Alcoholic Fibrosis/Cirrhosis. Fatty liver is the result of short term alcohol ingestion, where as Hepatitis and Cirrhosis require long,sustain alcohol use. Alcohol Hepatitis is manifested by Mallory bodies, infiltration by neutrophils, liver cell necrosis, and a perivenular distribution of inflamation. Fatty liver, Alcohol Hepatitis and even early fibrosis can be potentially reversible if the pt stops alcohol consumption. ****Females are more suseptable to ALD. The most characteristic manifestation is ALT/AST > 2 . ALT & AST are almost always <500,>500 this raises the probability of injury from drugs. Fatty liver exist in 80% of alcoholics but only 15-20% develop alcoholic hepatitis, and only 50% of pts w alchoic hepatitis develop Cirrhosis. Malory bodies are NEITHER specific NOR required for dx of Alcoholc Hepatitis.**** Pts with Alcoholic Cirrhosis should have Esophagoscopy to prevent varices.

ALL - 2
Presence of more than 25% lymphoblast in BM and the Positive Periodic Acid Shiff reaction (PAS) makes the Dx. ****First symptoms are non specific, fatigue, palor, fever, anorexia, petechia and lymphadenopathy. Dx is suggested by thrombocytopenia and blast cells, but confirmed with BM bioposy. DDX1: Hodgkins, presents with painless, firm cervial adenopathy, sign and symptms are similar to ALL but LYMPHOBLASTS make ddx of ALL. DDX2: AML, occurs in adults, main dx is >25% MYELOblasts in BM biopsy. DDX3: Aplastic Anemia, can present lilke ALL BUT lab shows decrease in ALL cell lines including WBC. DDX4: ITP, children with ITP present with sudden onset of bruising,petechia and occasional Epistaxis. The only cells that are very low are Platelets and their size is LARGE. DDX5: Infectious mononucleosis, presnts with lymphadenopahty, fever and pharyngitis, due to EBV. ATYPICAL lymphocyte are seens on peripheral blood smear and MONOSPOT test is positive. ***** If parents refusing treatment, obtain court order for chemotherapy. ****TX meds.

Allergic Bronchopulmonar Aspergillosis
ABPA is characterized by transient recurrent pulmonary infiltrates, peripheral eosinophilia, asthma and immediate wheel and flair reaction to Aspergilus fumigatus and presence of antibodies in serum against AF. Other characteristics are Hx of Brownish plug in the sputum and high IgE levels. Glucocorticoids are used to tx this dis. Whenever an Asthmatic pt is suspected of having ABPA, skin testing with Aspergillos antibody is first dx step, if its negative ABPA is tuled out. If positive serum precipitants agianst Aspergilos and IgE levels are checked. ABPA is excluded if IgE is <1000,>40% is suggestive of this dis. Tx is Glococorticoid. DDX4: Churg-Strauss is a multisystem vasulitic disorder of unknown etiology hat affects skin,kidney, CNS, lungs, GI and heart. There is asthma, , fever, marked Eosinophilia. Tx is glucocorticoids.

Allergic Conjunctivitis:
Is an acute hypersensitivity reaction that is caused by environmental exposure to allergens. Characterized by intense itching hyperemia, tearing, conjunctivla edema and eyelid edema.

Allergic Contact Dermatitis -3
Caused by Nickel and poison IV. Type VI hypersensitivity reaction. It mostly occurs in adults. DDX1: Atopic Dermatitis, presents as pruritic lesions in infants <6mo. Prevention is the mainstay of tx. Everywhere is involved but diaper area apears spared. Give infant warm bath and moisterizers. Acute attack maybe helped with low dose corticosteriods.

Allergic Interestitial Nephritis
Its secondary to Nafciline use. It’s a type IV hypersensivity reaction. Nephrotoxic agents are antibiotics (pencilline,cephalosporine, sulanamide, rifampine, cipro), thiazides, omeprazole, NSAID. Triad of fever,petechial rash and peripherla eosinophilia in an azotemic (Increased Urea) pt is highly suggestive. DDX: Acute Tubal Necrosis is mostly seen in ischemic or nephrotoxic renal failure. MUDDY brown casts are characteristic.

Allergic Rhinitis - 2
Dark puffy eyelids is called allergic shiners. The red crease over the nose causes constant rubbing, called allergic salute. Tx is avoidance and decongestants. ****If rhinitis is not clear if its allergic or infectious, then next step is Nasal cytology. Demonstration of neutrophils in nasal secretions suggests infectious cause. Predominant of Eosinophils suggest allergic cause. Other cause of nasal eosinophilia include Nasal Polyposis (Aspirin sensitivity).

Allergy, Drug
for mild reactions just use antihistamines. For systemic reactions, like anaphylactic use Adrenalin or Steriods

Alpha Feto Protein
The mcc of its deficiency is gestational age error. HIGH levels are seen in Gastrochisis and omphalocele, as well as 'false positive' causes like fetal demise, multiple gestation, inacurate gestational age. In case of increased MSAFP should first do US to rule out false positive causes and to detect presence of any anomaly. Afterwards, Amniocenthesis must be ordered for confirmation by measuring amniotic level AFP and AchE. AchE is a protein that increased only in neural tube defects. LOW levels of MSAFP are seen in chromosomal abnormality especially Down's synd. The screening is more acurate when MSAFP is coupled with b-hCG and Unconjugated Etridiol (UE3) levels, Its called TRIPLE TEST. Combnation of Decreased MSAFP + Increased b-hCG + Decreased UE3 is typical for Down's. In trisomy 18, ALL three are decreased. Likewise, US has to be perfomed to rule out inacurate dates and fetal demise, then amniocenthesis to confirm the Dx. MSAFP and triple test should be performed by 16-18 week of gestation. ****AFP is produced by Yok sac and fetal liver, some passes to maternal circulation. Other procedures: CVS- is indicated in women who are known to have genetic abnormality or previous affected children. Its done 10-12 weeks and offers advantage of 1st trimester testing.

Alpha-1 Antitrypsin Deficiency
It’s a protease inhibitor synthesized in liver. Pts w homozygous def are at risk of Panlobular Emphysema in adult life. The mc manifestation in adults is Asymptomatic cirrhosis, and maybe complicated by Hepatocellular Carcinoma. Hepatocytes contain granules that are PAS positive and Diastase resistant. DDX1: Whipple's, which is PAS positive but doesnt cause cirrhosis.

Alport Synd
Recurent episodes of Hematuria, sensoryneural deafness and a family hx of renal disease. Alternating areas of thinned and thickened capilary loops with spliting of GBM.

Alprazolam:
Abrupt cessation is asso with significant withdrawl symptoms like generalized seizure and confusion.

ALS - 2
Tx is Riluzole (glutamate inhibitor. Side effects are dizziness, nasea, weight loss, elevated liver enzymes and skeletal weakness.) Both upper (spasticity, bulbar symptoms, hyperreflexia) and lower motor neuron (Fasciculation) damage. Muscle wasting of all body muscles. "Tuesdays with Morrie" Jack Lemon.

Altered Mental status in elderly
Major causes include: 1-Hypo/Hyper natremia. 2-Hypo/Hypercalcemia. 3-Hypomagnesemia. 4-Hypophosphatemia. 5-Hypoglycemia. 6-Stroke. 7-cardiac events. 8-infections.


Alturism
Alturism is minimizing internal fears by helping other who have same problem (Alcoholic volunteering in AA). DDX: Sublimation, turning unacceptable behavior to a more acceptable ones.


Alzheimer's Disease - 4
Diffused cortical atrophy. Tx is Donezapin, Tacrine, rivastigmine, galantamine. **** Elderly gradual memory decline with Apraxia (Loosing the ability to do routine acts), Aphasia and Agnosia (not recognizing familiar objects). *****DDx it from Picks by MMSE, which is decreased in AD. In picks you need to see more than just one indication of behavior changes(urinating is not enought).

Amaurosis Fugax
Amorosis Foo-Gacs: Visual loss that is monocular, transient "dropping of the curtain". Opthalmoscopy reveals zones of whitend, edematous retina, following retinal artery distribution. Seen in pt with atherosclerosis and CV disesae. Its caused by retinal emboli from ipsilateral carotid artery. It last about 15 minutes. Tx of atherosclerosis is important to reduce the risk of stroke. Dx is with Duplex of the carotid.

Amebic (liver) abscess - 2
More common in tropical males. After intestinal infection with Entameba Histolytica. Transmission by water or food. Dx of liver abscess is by CT. When aspirated has "Anchovi-paste" appearance, Tx is Metroniddazole, orally, given one to two weeks. ****Hx of travel to endemic areas followed by dysentry and RUQ pain with a single Cyst in right lobe of liver is indicative of ALA. Primary infection is i the colon, but then it goes to portal vein and liver. Dx is made by stool examination of trophozoit serology and liver imaging. Tx is Metronidazole. DDX:Hydatid Cyst, caused by Echinococcus acquired by contact with dogs

Amenorrhea - 3
1-Secondary Amenorrhea: the first step is to rule out pregnany, then hyperprolactinoma, then hypothyroidism. The 2nd step should be determination of pt's estrogen status with progestine challenge test. A- If pt has adequate estrogen and a history of intrauterine instrumentation then suspect Asherman's synd (intrauterine adhesions. A hysterosalpingogram can show). Pts with no such hx are all anovulatory or oligo-ovulatory. B- If estrogen is inadequate, FSH should be ordered to determine gonadal or central origin.****Prolactin production is inhibited by Dopamine and stimulated by serotonin and TRH. An increase in TSH and TRH may lead to Hypothyroidism. Hyperprolactinoma may also affect GnRh and gonadotropin secretion and thus result in ammenorrhea. Other causes are dopamine antagonist (antipsychotics,TCA), hypothalamic and pituitary tumors. In the case of ammenorrhea-hyperprolactinoma , first rule out hypothyroidism by measuring serum TSH.***2ary Ammenorhhea in athletes is due to Estrogen deficiency because menstruation happens because of Estrogen.******Check out Table in Q41, Exam 12 0r 13.

Amiodarone tox - 2
1-Pulmonary, 2-Hepatotox, 3-corneal deposits, 4-skin reactions. *** If a pt needs rate control but has Restrictive lung disease Amiodarone is CI.

Amlodipine side effect

Amphetamine intox
pt might act like schizo but HT is not normal. Cocaine is the same as Amphetamine. DDX is Manic episode that has the mnemonic DIGFAST (Distractbility,Insomina,Grandiosity,Flight of idea, Activity increase,Speech talkative,Thoughtlessnes risky actions.).And Herion Tox: Triad of altered consciousness,respiratory depression and pinpoint pupils. Herion Withdrawl: muscel and joint pain , N&V, diarrhea,abdominal cramps, rihnorea,lacrimation,sweating. Amphetamine Withdrwal:depression, increased appetite ,fatig , irritability.

Amphotericine Toxicity
Hypokalemia.

Amyloidosis:
In heart is the end stage and next step is Transplantation.

Anal Fisure
They are most comonly caused by passage of hard, large constipated stool. The mc symptoms are severe pain and bright red rectal bleeding during defecation. Tx of both acute and chronic fisures starts with dietry modification (high fiber diet and lots of fluids) along with stool softner and local anesthetics.

Analgesic Nephropathy - 2
Clinical senario describes a woman with chronic HA, almost everyday, who presents for Hematuria. Several years of abuse leads to chronic tubointerstitial damage. Hematuria is due to Papilary Necrosis**** It’s the mc form of drug induced chronic renal failure. Most commonly in femlaes . Papilary Necrosis and Tubulointerestitial nephritis are the mc pathologies seen. Polyuroa and sterile Pyuria with WBC casts are early manifestations. In advanved cases you see Proteinuria and hematuria.

Anaphylactic shock
One HOUR After bee sting in ER the first thing to do is SC Epi, not removing the sting. If after oneminute then first remove the sting.

Anemia of Prematurity
is the mc anemia in premature and low birth weight infants. Pathology involves a diminished RBC production, shortened RBC life span. And blood loss. Iron supplementation doesn’t help falling Hb levels and iron def is not the cause of prematurity.

Aneurysms
Causes are: 1-TARUMA:Aneurism in a young pt who presents with Desending Aortic aneurism. Pathophy is acceleration trauma. It might show in Cxr by wide midiastinum, 10% will have normal cxr so if you suspect it do CT or MRI. Tx is surgery to prevent rupture. 2-ATHEROSCLEROSIS:Is the mcc of Descending Aorta aneurism. Pts are older, smokers. They are generaly asymptomatic and are seen on Cxr. Majority of pts also have significant CAD. 3-MARFAN:nomonic is 'm.A.AR.f.A.n". Pts present with Ascending aneurism of Aorta. Asso findings are Aortic regurgitation Surgery is required to replace both aortic valve and entire ascending aorta. They also have a higher chance of Aortic Dissection than genral population. 4-MYCOTIC:result from localized infection , Its mc in Femoral artery and 2mc in Ascending aorta. The mc pathogen is S. Aureus and 2mc is Salmonela. 5-SYPHLYTIC:Occur in Ascending Aorta. Pt presens with fever,chills, spliner hemorrhages. CT is dx.

Angina, Prinzmetal or Variant - 2
Classis picture is a pt with absence of risk factors of CAD, night pain waking her up, transient ST elevation, absence of Q waves and negative cardiac enzymes. The disease results from coronary vasospasm of the artery that causes "Transmural Ischemia" and hence ST elevation on EKG. Other things to know is: "Subendocardial ischemia" in Angina pectoris causes ST Depression. "Transmual Infarct" causes ST elevation followed by development of Q waves and increased cardiac enzymes. "Subendocardial infarcts" cause ST Depression that are not followed by Q waves and elevation of cardiac enzymes. Summary: TM-IS=Elevate ST-Q-Enz. TM-IN=Elevate ST+Q+Enz. SE-IS=Suppresed ST-Q-Enz.and SE-IN=Suppresed ST+Q+Enz. *** Propranalol and Aspirin are CI in these pts. The initial tx is with Nitrates and Calcium channel blockers. Second drug is only added when there is no response to the first drug. DOC for initial mgmt is Calcium channel bloker, Diltiazam.

Angina, Stable - 4
EKG stress test is the initial test for dx. Rbbb is not a CI. But when a pt has Lbbb,WPW, ST depression >1mm at rest then stress testing with imaging is done. Dobutamine stress test is for those pt who cant exercise sufficiently. Coronary Angiogram is done if stress test fails. Myocardial Perfusion is for those who are at risk of develoing complications with excercise or Dobutamine. *****Medications that has to be withheld prior to EKG Exercise test are Anti ischemic mdeciation, Digoxin and medications that slow the heart (B-Blockers, Atenolol).****In pts with stable angina and HT, B-blocker is tx of choice. CCB(Verapamil) is indicated if BBs are CI or dont work. They both have BOTH anti HT and anti anginal effect. Enalapril has ONLY anti HT effect. ****Stress EKG or an Excercise Echo should be done for risk stratification in pts with stable angina. Pharmocological stress testing is an alternative if pt cant do excercise. Coronary angiography is done when pt is refractory to medical tx or when excercise tesidentifies pt as high risk.

Angina, Unstable - 2
Ishcemic chest pain only partially releived by Nitroglycerin, T wave inversion, and negative cardiac enzymes. Tx for unstable angina and NON-Q wave infarction is with IV heparin, aspirin, B-blobker and nitroglycerin is indicated. Thrombolytic tx is asso with mortality in these pts. Thrombolytic therpay is indicated in MI with ST elevation after sublingual Nitro rules out coronary vasospasm. Another indication for Thrombolytics is LBBB. ****Give CLOPIDOGREL not Ticlopidine for platelet de aggregtion.

Angiodysplasia or Vascular Ectasia

The two mc causes for pianles GI bleeding are diverticulosis and Angiodysplasia. DDX is that Angiodysplasia is asso w Aortic Stenosis. Other asso is renal failure. Also Sigmoidoscopy reveal Diverticulosis and not Angiodysplasia. MERK:Angiodysplasia is an acquired submucosal AVM, which may cause lower GI bleeding in elderly patients. When the bleeding is massive, it is usually from either angiodysplasia or diverticulosis. Typical angiodysplastic lesions are 0.5 to 1.0 cm, bright red, flat or slightly raised, and covered by very thin epithelium (see Plate 22-3). Treatment is indicated for angiodysplasia that has bled because of its tendency to cause chronic recurrent hemorrhage. Active, severe bleeding may be controlled quickly by intra arterial or IV administration of vasopressin when the patient is stabilized, but results are variable. The lesions then may be treated more definitively by endoscopic coagulation or surgery. The most difficult aspect of treatment is to eliminate other potential causes for the GI bleeding and to locate all of the angiodysplastic lesions. If the lesions are not large or numerous, endoscopic coagulation with hot biopsy forceps or laser photocoagulation is preferred. The usual surgical treatment is a right hemicolectomy because of the propensity for angiodysplasia to involve the right colon.

Angioedema - 2
ACE inhibitors are notorius for producing Angioedema in ER. Pt presents with non-inflamatory edema and laryngeal edema that could be life threatening. Angioedema occurs due to proinflamatory action of substance.P which is stimulated by Bradykinin. Bradykinin can be broken down by angiotensinogen converting enzyme. When an ACE inhibitor blocks this enzyme, the levels of brady kinin increases leading to angioedema. Tx is Anti histamine.

Angiofibroma - 3
A benign vascular tumor found in adulescent male. Present with frequent epistaxis(Epistaxis is the major symptom), nasal obstruction, HA & conductive hearing loss. In PE hay greyish-red mass in the posterior nasopharynx. CT is Dx, TX is medical and surgery, depending on stage. ****Any adulescent boy with epistaxis and has localized mass with bony erosions on the back of the nose has an Angiofibroma unless proven otherwise.

Anion Gap Metabolic Acidosis 3
T9Q3. First see pH<7.4, then see HCO3<24 then we know its MA. To calculate compensation use Winter's Formaula PaCO2= 1.5 (HCO3) + 8, this is what CO2 would be after compensation. Normal AG is b/w 6-12. MERCK: When metabolic acidosis results from inorganic acids (ie, hyperchloremic or normal anion gap acidosis), HCO3 is required to treat the acid-base disturbance. However, when acidosis results from organic acid accumulation (ie, increased anion gap acidosis), as in lactic acidosis, ketoacidosis, most experts still recommend judicious use of IV sodium bicarbonate in the treatment of severe metabolic acidosis (pH < 7.20)******AG formula is (Na)-(Cl+HCO3), normal is 6-12.

Ankylosing Spondolytis - 3
Asso with IBD.****regular exercise is the only tx that halts progression of the disease. Pt is young, presents with insidious onset of back pain for more than 3 months, positive family hx, reduced back motion and chest expantion, also HLA-B27. Xray shows scoliosis. NSAID is for pain control. Sulfasalazine is for peripheral joint involvment. Surgey is recommended when dis is sever and refractory to medical tx. ****Dx cant be made unless there is evisence of sacrolitis. So when pt has symptoms of AS, the next step is to do Xray of the sacro iliac joint. If Xray is inconclusive then MRI is done.

Anorexa Nervosa - 2
Elevated carotene gives the skin a yellow color. Carotene is also elevated in DM and Hypothyroidism. Pregnant women with current or previous AN are at risk for Miscarriage, intrauterine growth retardation, hyperemeis gravidarum, premature birth, cesarean delivery, & post-partum depression. Osteoporosis is a common finding in women pregnant or not. Also elevated cholesterol and carotene levels, euthyroid sick syndrome, cardiac arrythmias (prolonged QT). The FIRST step of MGMT is Hospitalization. ****There Ammenorrhea and body weight is below normal. In bulemia weight is normal.****Once the dx is made the first step in managementis hospitalization.

Anserine Bursitis
Pain over medial tibial plateu, hx is asso with trauma and cxr is normal.

Anterior Cord Synd
Usually occurs due to motor vehicle accident injury. There is Paralysis and analgesia below the level of injury and preservation of posterior column function like position,touch and vibratory. Pts trearted with High-dose Methyl prednisone w.I 8 hrs of injury have significant neuorologiccal improvement. All trauma pt do 2 things, 1-Immobalize, 2-ABC.*****Asso with burst fracture of the vertebra, characgterized by total loss of motor function (Paraplegia) below the lesion, with loss of pain and temperature on both sides below the lesion. MRI is the best initial Dx procedure.

Anti Psychotc drugs
Work by blocking Dopaminergic receptors. Typical ones are Haloperidol, Chlorpromazine, Fluphenazine. Atypical ones , add Serotonin blocks as well, so block EPS SE. Atypical is Clozapine, Risperidone. ****Dystonia, an extrapyramidal SE of Haloperidol is treated with Benztropine or Diphenhydramine.

Anti-D Ig
After events that are asso with maternal-feto hemorrhage (placenta abruption) the failure to correct the dose of Anti D can result in maternal Alloimmunization (T22Q39).

Anti-depressants - 2
Are SSRI, MAO inhbx,TCA. SSRI causes sexual dysfunction. If it does, discontinue and give Bupriopoin (inhibit Nepi, and dopamine reuptake) it doesn’t cause impotence. TCA also causes sex dysfunction. Trazodone is good for antidepressant in those with Insomnia, but it too causes sex dysfunction. *****In pts with terminal dis, when severly depressd with active suicidal thoughts antidepressnt should be given immediately, not lectures about accepting the fact and the feeling being normal and .blahblahlah...

Antiphospholipid Antibody Synd
Recurrent arerial or venous thrombosis or recurent fetal loses in the presnece of Antiphopholipid antibodies. There are 3 types of APLA, first one is responsible for false VDRL, Second is LUPUS and falsly elevates APTT, the Third is Anticardiolipin. The tx is Heparin+Aspirin.

Antisocal Personality
Is dx in those older than 18 yo who engage in illegal activites and abuse others. They show CONDUCT disorder when they are minors.

Aortic Aneurysm
MCC is ascending aorta and cuase is cystic medial necrosis. Descending aorta is asso with atherosclerosis.

Aortic Aneurysm, Abdominal -3
After AAA repair (surgery) , diarrhea and blood in stool should raise the question of Ischemic Colitis. . If CT is inclusive, a sigmoidoscopy/Colonoscopy is recommended. DDX is Pseudomembraneous Colitis due to antibiotics will present with same symptoms but not the ischemic changes in the colon. CT shows ruptured aorta and blood around aorta, tx is exploring the abdomen.***The study choice of Dx and folow up is abdominal USG. ****When pt presents with pulsatile mass and hypotension , a presunptive dx must be entertianed. and pt should be taken directly ro surgery, NO USG OR CT.*****spinal cord ischemia with lower spastic paraplegia is a rare complication of AAA. Its due to loss of blood during the operation.*****When there is ruptured AA confirmed with CT, then the next step is Explore abdomen not Laparoscopy (not used in acute conditions).

Aortic dissection - 6
Htn and BP difference in two arms. First thing to do is admit to ICU, IV Nitropruside(reduce BP), Beta blocker(Esmolol,reduce heart rate). Any delay maybe fatal, don’t even give pain killer first, just do the above. So first tx is antihypertensive agent, before CT,MRI,TEE or Cxr. Intense retrosternal pain that radiated to subscapular area, also check for Aortic regurgitation (decresendo diastolic murmur in the left sternal border, also Hypertension. Dx w TEE. ******The mCC of AD is HT, if given no info pick this as the cause.****Acute AD is a risk factor in Marfan pts. Tearing pain and raddiation to the back and a difference BP of 30mmhg b/w two arms are impotant clinical clues. TEE or CT ar the dx studies of choice.

Aortic Regurgitation
Presents with Water hammer or collapsing heart and pistol shot femoral pulses. These occur due to hyperdynamic circulation and early diastolic runn off of aortic insufficincy.****Tx is Diuretics+ACE inhibitors+Digoxin, are given first to releive congestive sysmtoms for LV dysfuntion and then we need to change valve is indicated. Pt must undergo Echo for diagnosis.

Aortic Anreurysm Rupture
Aortic rupture should be rulled out in ALL trauma pts with Severe chest trauma, pulsatile mass and hypotention. Its best done with Cxr. The signs are:1-widening of mediastinum>8cm, 2-Depresion of L main bronchus >140degrees, 3-Deviation of nasotracheal tube, 4-Fracture of 1st&2ns rib,sternum,scapula, 5-L apical hematoma. Immediate surgery is very important but do confirmatory CT or angiogram. BUT REMEMBER if question gives you option b/w Surgery and CT go with surgery.

Aortic Coarcation:
The tx for RECURRENT AC is Baloon Angiography.

Aortic Stenosis - 4
Age dependant idiopathic sclero-calcific changes are the mfc of isolated AS in elderly. Pt presents with exersional syncope. PE shows increased intensity of point of maximal impulse. Auscultation reveals ejction-type systolic murmur. With radiation to carotid arteries. NOTE:Bacterial endocarditis may lead to Aortic insufficiency not aortic stenosis. ****Pt presents with Anginal chest pain, dyspnea or syncope (The classic triad of symptoms is syncope, angina, and dyspnea on exertion.). Pain is ischemic in origin and occurs due to increased O2 demand 2ary to LV hypertrophy. ECHO is the study of choice to Dx AS. Its also used in follow ups****Key to Dx is Harsh systolic murmur over the right sternal edge, know that only left sided murmurs increase on exspiration. S4 results from forceful atrial contraction against the thick non-compliant ventricle. The classic indication for surgery in pt with AS is SAD (Syncope, Angina, Dyspnea). Dyspnea results from CHF. Presentation of either indicates valve replacment surgery. The indications for Aortic valve replacment are:1-All Symptomatic pts, 2-Pt with severe AS undergoing CABG. 3-Asymptomatic pt with severe AS either poor LV systolic function , LV hypertrophy > 15mm.****In ALL AS pt who are SYMPTOMATIC, IE prophylaxis and repeated regular follow ups are recommended.

Aplastic Anemia, Acquried
Injury to the bone marrow by drugs, radiation, toxins or insecticide. Pedaitrics disese. Pallor,fatigue, loss of appetite, easy bruising, petechia, mucosal hemorrhage and fever. Lab shows, anemia, leukopenia, thrombocytopenia. BM biopsy is essential for Dx and shows hypocellular BM and fatty infiltration. MERCK:Tx: Equine antithymocyte globulin (ATG) has become the treatment of choice for older patients or those without a compatible donor. Combined ATG and cyclosporine is also effective.. Bone marrow transplantation from an identical twin or an HLA-compatible sibling is a proven treatment for severe aplastic anemia, particularly in patients aged < 30.. DDX:Fanconi syndrome, familial, pancytpenia, brown pigmentation, cafe au lait, short stature, upper limb abnormality, skeletal abnormality, it starts w thrombocytopenia then neutropenia and then anemia. DDX2 Diamond-Blackfan Anemia, or congenital RBC aplasia presents in the first three months of life w pallor and poor feeding. WBC and platelet counts are normal.

Appendicitis:
If a 62 yo on WARFARIN comes in with appendicits and requires emergent surgery, first step is to reverse the effect of Warfarin not by Vitamin K infusion (takes 1-2 days for effect), but with infusion of FFP. Then proceed with the surgery. ****Acute Appendicits may be complicated by pelvic abscess that presents with lower abdominal pain, malaise, low grade fever, and tender pelvic mass on rectal exam. Most of pelvic abscesses are due to perforation of appendix. Pt could have had appendicits that resolved with rupture and abscess formation. The diarrhea is reactive due to irritation. Drainage of the abscess is the tx in these cases. ****Experiecne has shown that right hemicolectomy with ileotransverse anastomosis has best postoperative results, when resection of part of ascending colon is required, when hay appendicits with cecum inflammation and pus.******Complicated appendicitis is when the pain is ignored for days and pt presents with high fever and localized pain to RLQ. Tx is with IV hydration, Antibiotics and bed rest. Non-operative management is curative, CT may reveal abscess that can be drained percutaneously Antibiotics should cover Gram negative and Anaerobics, Cipro+Vancomycin.

ARDS - 4
Could happen secondary to Acute pancreatitis. Dx:PaO2/FiO2 must be <200 (PaO2=55, Receiving O2 by mask
is 60%) in the setting of absence elevation of left atrial pressure (PCWP<19mmhg). Sepsis is the mcc of ARDS. Other causes are Herion, acute pancreatitis, severe burn and near drawning.****Goal of mechanical ventilation is to improve oxygenation and is best doen with increased PEEP. PEEP keeps the alveoli at end expiration and keeps them open for a longer duration to inprove oxygenation. PEEP is the major factor in improving oxygenation in ARDS.***In a postoperative pt who prsents with dyspnea and tachypnea one must exclude MI,PE,Pneumonia and ARDS. If pt presents with Bilateral fluffy infiltrates on a chest xray and hypoxia with diffuculty with oxygenation Its most probabely ARDS. Its not Atelectasis so there is NO FEVER.

Arm Fractures
1-Colles, outstreched hands in elderly. 2-Smith, my injury, 3-Bartion, intraarticular fraction of distal radius. 4-Chauffer's, fracture of radial styloid process in drivers. 5-Galazzi, isolated fracture anywherea long radius with asso injury to the distal radial joint.


Arrest Disorder - 2
Midpelvic contraction which is indicated by prominent ischial spines is an important cause of Arrest disorder or dialation. DDX:Inlet Dystocia, Descent of the presenting part at +1 indicates that fetus is engaged, so ID is unlikely.****Arrest in dilation more than 2 hours, and arrest in descent more than 1hour is the definition. Can be caused by hypotonic contraction, anesthesia, cephalopelvic malproportion or malpresentation. If arrest is in midpelvic contraction, indicated by prominent ishcial spines, then the next step is to do a C-section. Forcetps cant be used until cervix is fully dialated (10cm). Oxytocin might cause uterine rupture due to pelvic prevention of birth. Now in case of Shoulder Dystonia, a last resort tx is 'Zavanelli' maneuver (pushing back the fetus in uterus & doing a C-section.

Arthritis, Reactive
It’s a form of seronegative spondyloarthropahty. Enthesopathy(A disease process occurring at the site of insertion of muscle tendons and ligaments into bones or joint capsules ) causes heel pain and sausage digits. Enthesopathy is quite specific to spondyloarthropathy. Tx of choice for Reactive Arthritis and Reiter's is NSAIDS. Tetracycline is added if UTI with Chlamydia is suspected and IM Ceftriaxone if Gonoccocal is suspected (but NIsseria does not cause RA).

Asbestosis
The hx of shipyard worker with cxr with pleural plaques is dx. DDX1:Sillicosis, hx of glass and pottery making. DDX2:Berylliosis, hx of high tech industries.

Ascending Cholangitis
Characterized by Triad of RUQ pain, Fever and Jaundice (charcot triad). Its an infection of the CBD, generaly 2ndary to obstruction of CBD with a stone leading to dilatation of CBD. Broad spectrum antibiotics should be started immediately, however, its very important to decompress the billiary duct and provide their drainage. ERCP (Endoscopic Retrograde CholangioPancreatography) is the method of choice. ERCP can be used to do a sphingtrectomy with the stone removal and drain the bile via the sphingter or by placement of a stent. Early drainge can significantly reduce mortality and morbidity.

Ascites - 3
Management starts wirh sodium, water and protein restriction, spironolactone, furesamide. If given a choice for only one drug tx b/w spironolactone and Furesamide, pick spironolactone. If that didn’t help then slow tapping of up to 2L of ascites fluid a day balanced with infusion of 10gr albumin per liter tapped. If that didn’t work then do surgery. The vascular shunts are indicated after first bleeding. Distal spleno-renal shunt will not improve and it might worsen it. Side to side porto-caval shunt might improve the ascites but worsen encephalopathy. Peritoneum-Jugular shunt is designed for tx of Ascites only. *If pt 's ascites is so much that is compromising other systems, the next step is Paracentesis which is both therapeutic and diagnostic. ****Spironolactone is the DOC in tx of Cirrhotic Ascites. Tx of ascites in Cirrhotic pts should be as followes: 1-All the pt shold have Dx paracentesis done. 2-Salt Restrcited Diet is the coner stone of the therapy, in 10-20% of pt thats all you need to do. 3-Pts not controlled with SRD, Spironolactone is next. Its an Aldosterone antagonist, and it works because Ascites is only due to 2ary Hyperaldosteronism. 4-Recalcitrant (difficult to manage) ascites should be tx with TIPS. 5-Very severe ascites should be tx with paracentesis initially.

Aspergilosis - 3
An opportunistic infection in South East USA. A mobile cavitary mass in the lung, which prestns with occasional hemoptysis. DDX1:Lung abcess, due to anerobic organism, with an AIR FLUID level on Cxr. Medical mgmnt is antibiotics, postural drainage and bronchoscopy. DDX2:PE, from lower legs, presents with Dyspnea,Tachpnea, Chest pain and collapse. ECG may demonstrate RV Hypertrophy, RBBB, Right Axis and T inversionin antreior leads. Cxy may show decreased pulmonary vascular markings. DDX3:Histoplasmosis:The mc fungal infection in US. Acquired by inhalatin. "Calcified Nodes" in lung , mediastenum or spleen. Cxr shows central or target calcification. Ocasionaly causes mediastinal lymph node enlargment.***Allergic Broncho Pulmonary Aspergillosis (ABPA), finding of central bronchiectasis on the cxr and elevated AgE and Eosinophilia is characteristic. Next do a skin test for Aspergilosis antibody and you ahve your Dx. Tx is Prednisone oral. Itraconazole may reduce the need for steriod but its not the main therapy.****It occurs in immunocompromised pt (those taking cyclosporine, chemotherapy). Pt prestns with fever, cough, hemoptosis, and dyspnea. Cxr may show cavity lesion. CT shows pulmonary nodule with a 'halo' sign. ****Aspergiloma is the “fungus ballâ€
 in preexisting cavities, mc presentation is hemoptysis, TX is Lobectomy.****4 types of infection: 1-ABPA (tx is Prednisone), 2-Aspergious Colonization, 3-Aspergiloma (surgery), 4-Invasive Aspergilosis (tx ix IV Amphotericine)

Aspirin intoxication - 3
!-ADULTS: Initially increased respiration leads to respiratory alkolosis and then uncouples oxidative phosphorylation and leads to met acidosis. So they have mixed metabolic acidosis and respiratory alkalosis. 2-CHILDREN: Initially causes Metabolic Acidosis and then compensatory Respiratory Alkolosis. Aspirin can cause acute erosive gastritis and upper GI bleeding. , alcohol can aggrevate this effect.

Aspirin Sensitivity Synd
Pathogenesis is 'Pseudo-allergic reaction'. Accumulation of leukotriens and changed leukotrien/progtaglandin balance triggers bronchoconstriction, nasal polyps in suseptible individuals. Tx are Leukotrien receptor inhibitors(DOC), topical steriods and aspirin desensitization therapy.

Asthma - 9
before and after administration of a bronchodialator (Beta-2 agonist). Significant improvement in FEV1 after bronchodialator indicates reversibility of destruction, which is more consistant with Asthma. ****Mast cell stabalizers (Sodium Cromolyn) are doc for pts who have other allergic disorders, so give this to a boy who started to have night time cough and wheeze with hx of allergic rhinitis.**** Exercise induces Asthma (not to be confused with post excercise asthma) presnts with chest discomfort, wheezing cough, breathlessness, fatigue and abdominal discomfort. Beta Agonist and Mast cell stabalizers (Sodium Cromolyn) are the best tx for these pts. **** When an asthmatic pt presents with Subcutaneous Emphysema, which is face becomes all swollen and palpation reveals crepitans all over face and neck, then the first thing to do is to do Cxr to rule out Penumothorax. Once that is rules out just observe the pt, it needs no tx.****Inhaled corticosteriods are indicartd in pt with persistant asthma symptoms. The agents are beclomethasone. In adults SE of low-dose drug are limited to are Dysphonia and Thrush. In high-dose systemic toxicity may occur. ****Its a common illness in childhood. 10% of children come ro ED with un-remitting asthma (continues wheezing despite tx with neubelizers and een steriods). This is called Acute Status Asthmaticus. Of these pts 10% require MECHANICAL VENTILATION, however hospitalization is mandatory. If on auscultation there is no air entry bilateraly, the child has 'silent chest' or absent air entry and continues to desaturate despite Prednisone therapy, therefor the best option is mechanical ventilation and hospitalization.*****For pt who have asthma accompanied with other allergic disorders, mast cell stabilizers like Sodium Cromolyn are the agent of choice. *****1st neubelizers, 2nd IV steroids, 3rd mech ventilax.Normal PCO2 is one of the indicators of a severe attack. During an attack, pt is tachpneac so he hyperventilates which should cause decrease in PCO2. So if PCO2 seems to be normal that means the obstruction is getting worst or respiratory muscel are getting too tired. Other signs of severity are broken speech, diaphoresis, cyanosis, altered sensorium and "silent lung". Inhaled corticosteriods improve long term quality of life in Asthmatics. Initial hypertensive therapy for pt with asthma is Hydrochlorothiazide.***To differentiate b/w Asthma and COPD (Emphysema) the best test is a bronchodialator response test. , its conducted by measuring FEV1

Astrocytoma
Occurs in Parietal lobe, supratentorial. It’s the mc tumor in both infra and supra tentorial. Medulablastomais the 2nd mc tumor in posterior fossa, 90% occur in vermis. Craniopharyngioma arise in sella torsica, visual field defect, Its characterized with cystic structure with calcification.

Asymptomatic actriuria of pregnancy:
When everything is normal but a routin clean catch urine culture grows 100000 colonies of E choli. Untreated pt have increased risk for cystitis and acute pyelonephritis. So they should be treated with 100mg Nitrofurantoin or Ampicillin for 7-10 days.

Atelectasis
its common after surgery in smokers. Bronchoscopy needs to be done to remove mucus Plug.Pt prestns with tachycardia, tachypnea, low grade fever. Once Bronchopscopy is done, cxr is repeatd and coughing is encouraged.

AtheroEmbolic Disease
It immitates Gout, but does not say red toe but its Blue toe. DON’T FALL FOR GOUT TRICK. Pt has cyanosis and circulation problems like pain in calf, pulses are fine.

Athlete foot
Pt presents with sever itching, fissure, thickness of the nail in a swimmer. Best tx is antigungal medicine, Tolnaftate.

Atpoic Dermatitis:
Edema and erythema of the skin. Skin is Itchy. Tx is Pimecrolimus, like Tacrolimus, its MOA is through inhibition of T cell activation.

Atracurium:
Is the neuromuscular blocking agent of choice for pt with renal and hepatic problems, because its metabolized in plasma.

Atrial Fibrillation - 4
Control rate and rhythm.**** AF along with WPW tx of choice is Procainamide or Disopyramide. Drugs that slow AV conduction (Dixogin, Verapamil) are CI in these pts, they may lead to malignant arrythmais. Lidocaine might als worsen the situation. Cardioversion is used in pts that are HemoDynamicalt unstable (very rapid vent rates with hypotension).*****When AF (absent P waves and irregular heart rate) is asso with HemoDynamic compromise, tx is only Cardioversion. If AF is not asso with hemodynamic compromise, Amiodarone is used. Amiodarone causes hypotension so its CI with HD compromise (hypotention already in pt), but once cardioversion stabalizes pt then Amiodarone is an excellent choice to maintain the pt. Calcium channel blockers are also ONLY excellent choices for AF when there is no HD compromise.

Atrial Flutter
shows with saw tooth EKG. Unstable AF is best tx with cardioversion. ACUte AF with stable hemodynamics is tx with cardioversion or rate control. Chronic stable AF is best tx with rate control with Ca Chanel blocker(VerapamiL) or Betablocker.

Aut Dom Polycystic Kidney Dis
5-7% asso w. Berry aneurysm. Routine screening is not recommended.

Autism - 3
Mainstay of tx is special education and behavioral modification techiques. *Have special interests.Usuallt starts before age 3. DDX:Childhood Disintegrative Disorder, is a rare pervasive developmental disorder, mc in males. Chracterized by a period of NORAML development for atleast 2 yrs, followed by a lost of already acquired skills. They have autism symptoms. Prognosis is poor and they are disabled for life.****Rett Synd: characterized by an initial period normal development until 6mo, followed by loss of hand coordination and sterotype hand movments. Almost exclusively in Females.

Autoimmune Hemoytic Anemia
SEE Spherocytosis

Avascular Necrosis of femur
The well known causes of non-traumatic avascular (aseptic) necrosis are chronic corticosteriod therapy, alcoholism. Pt presents with progressive hip pain w/o restriction of motion and normal Xray. MRI is the gold standard.

Avoidant personality:
Shyness and feeling of inferiority, and desire to make friends.

B12 Deficincy, Hem&Onco, 6/2
DDx b/w Anemia and vegeterian diet is the duration. We store 3-4 years of B12 in the body, so if you’re a vegeterian <3>4 years then we do Abody test for intrinsic factor.****DDX b/w Folate and B12 is increase in Methylmalonic level. Folate will cure anemia but neurological problems will progress.

Bacillary Angiomatosis:
Is caused by Bartonella species, gram negative bacilli. Cutaneous lesions are round papules or nodules, vascular and asso with fever malaise and headache. It occurs in HIV pts.


Bacterial Meningitis
CSF : elevated protein, decreased glucose, and elevated WBC. Plus skin lesions of Purpura and petechia.****Acute Bacterial Meningitis:the 3 mf causes in communit acquired ABM are S. Pneumonia, H. Inf and Meningococcus. Pneumococci have become resistant to penicillin and cephalos, so empirical therapy in adults and children include Vancomycin in addition to Ceftriaxone. Listeria Monocytogenes is a fc in pt older than 55, so we add Ampiccilin for these pts. Other pts at risk for LM are immunocompromised and lymphoma pts. In children >3 yo, LM is a risk so empiric regimen inclused Ampicillin in addition to Cerotaxime. Now S. Aureus and pseudomona are agents in meningitis in hospitalized pts, so empiric therapy is with Vancomysin (for aureus) and Ceftazidime (pseudomona).

Bartter Synd - 2
The DDX of normotensive pt with hypokelemia and metabolic alkolosis include: 1-Diuretic abuse, 2-Surrepticous vomitting, 3-Bartter synd, 4-Gitelman synd. Classis Barter usualy presnts early in life, as polyuria,polydupsia,growth and mental retardation. However this can occur later. The underlying pathology is defective sodium and chloride reabsorption. in the ascendign loop, thereby resulting hypovolemia and consequent activation of renin-angiotensinogen aldosteron system. This then causes increase in K & H ion secretionleagin to hypokalemia and alkolosis. DDX:Primary hyperaldosteronism and Renin secreting tumors are charcterized by HT, Met Alk and Hypokalemia. Measurment of Plasma renin activity and aldosterone is used for DDX b/w the two. in Primary hypoer Aldosteronism, PRA is suppresed and aldosteron is elevated, but in renin tumors, both PRA and Aldosterone are elevated. Now remember the mcc of Hypokalemia in clinical practice is Diuretics, which is hard to ddx with Barter.

Basal Cell Carcinoma - 2
Is the mc malignant tumor of the eyelid. Lesions are slow growing, pearly and indurated. Invasive clusters of spindle cells surrounded by palisaded basal cells. It rarely might appear on upper lip but NEVER on the lower lip. The mc location is the lower eyelid. They rarely metastasize. Squamous CC is much less commn and faster growing, It presents as plaque nodule or inverted wart, its ddx is Actinic Keratosis.****sun is Bad for Basal cell carcinoma. Five warning signs are 1-Open sore tht bleeds, oozes and remain open for >3weeks. 2-Redish patch,3-Shiny bump, scar like area, 5-Pink growth with rolled border. BCC is the mcc of skin in US. Never metastasis. Its removed using by 1-Cauterization(burning), 2-Surgical (excision with 1-2mm margin). 3-Cryosurgery(freez) and 5-radiation.

Basilar skull fracture
Signs are rhinorrhea, raccoon eyes (black eye), ecchymosis behined the ear. A way to see if hay CSF mixed w blood is to drop a drop on a cleansing tissue, if hay csf there would be a yellowish spreading on the paper. In this pt head fracture has to be ruled out w CT of head and spine. Expectant therapy for all uncomplicated cases. Anterior packing is not necessary to control CFS loss. If CSF leak continues for >4 days, spinal drainage and acetazolamide is used to reduce CSF production and reduce ICP.

Bechet - 2
Is a rare multisystem disorder that affects males <20yo, in mediteranean area and east asia. An AUTOIMMUNE mechanism is suspected. It manifest with Ulcers in mouth and genital area and asso with Uveitis. Oral lesions are Aphtha like but genital lesions are more destructive leading to fenesterated vulva. No specific tx yet. ***Its a Multisystemic Inflammatory condition with recurrent oral and genital ulcers, skin lesions, mc in Turkey, Asian and middle east.Corticosteriods offer releif but dont prevent progression to Dementia and Blindness.

Beckwith-Wiedemann synd
Infant with macrosomia, macroglossia, visceromegally, omphalocele, hypoglycemia & hyperinsulinemia. iT might be asso with duplication of CH 11p, this region has the gene for IGF-2, which may explain macrosomia. DDX1:congenital hypothyroidism has umbilical hernia instead of omphalocele, and there is no hypoglycemia and hyperinsulinemia. DDX2:Macrosommia due to maternal diabetes, however these infants dont have the dysmorphic features of omphalocele, prominant occiput and macroglossia. The common congenital problems in these infants are Caudal progression synd, Transposition of great vessles, Duodenal atresia and small left colon, Anencephay and neural tube defects. DDX3:WAGR synd, Wilms tumor, Aniridia, Genitourinary anomaly, and mental Retardation. Its related to deletion of CH11involving the gene WT1.

Bells palsy
Is the PERIPHERAL seventh nerve palsy, Its dx with absence of forehead furrows and thus ruling out the CENTRAL Facial Paresis. Pts with Central lesion still have furrows because contralateral motor inervation of forehead rremians intact.

Benign Intrahepatic Cholestasis
It can develop after a major surgery in which hypotension, extensive blood loss in tissues, and massive blood replacement are noted. Jundice develop due to pigment load from transfusion. Jaundice becomes evident 2nd day post operative. Alkaline phosphatase is markedly elevated but ALT & AST are only mildly elevated. DDX1:Acute hepatic failure, has increased PT, low albumin and neurologic signs due to hepatoencephalopathy. DDX2:Hepatitis, presents with marked elevation of ALT & AST. DDX3:Halothane Hepatotoxicity, type-1 has mild elevation of liver enzymes and NO jaundice, type-2 is characterized by Acute Liver failure.

Bernard_SoulierSynd
The hallmark is GIANT platelets.Its auto recessive. There is mild thrombocytopenia but the major defect is of membrane glycoprotein Ib. This defective membrane lacks the receptor for VW atachment so platelet cant adhear to endohtelium. Plateles don’t aggregate in presence of normal VWF and Ristocetin. Vigniet prsents a 16 yo girl who's periods last 6-10 days and her brother also had bleeding problems.

Beta blocker Toxicity
Overdose causes hypotension and bradycardia. Sever overdose may result in cardiogenic shock. If bradycardia or AV abnormallity is found Atropine is indicated to oppose unopposed vagal tone. Isoproterenol is given if Atropine fails and if both of them fails then Glucagon is the DOC. If medication fails then a temp pacemaker is indicated.

BIASES: SEE STEP UP book
1-Selection: loss of people to follow up in prospective studies.
2-Observers and Ascertainment: result in misclassification of the outcome due to flaw of the design of the study.
3-Recall: Misclassification of the exposure status, its potential problem for case-control.
4-Confounding:
5-Generalizability: when th epopulation you study does not include all the population where the topic of the research is covering. Like studying just men when ALL people are targeted.
6-Reliability:
7-Validity.
8-Leadtime: Its prolongation of apparent survival in pts whom this test was applied, w/o changing the prognosis.

Bicuspid Aortic Valve
Is the mcc of aortic stenosis in middle age adult. Both AS and HCM produce a midsystolic (Ejection systolic murmur) murmur, however murmur of HCM is best heard at left lower sternal border and it doesn’t radiate to carotids. Valsalva Attenuates AS murmur but Accenuates HCM murmur. Murmur of AS is best heard right second intercostal space and radiates to the carotids. Slow rising puls is seen in HCM.

Billiay Cholic:
Is symptomatic for CheledoCholelithiasis. If there are no signs of acute cholecystitis (Murphy sing, elevated WBC and fever) then there is no need for hydration, antibiotics or emergent chlecystectomy. There could be Emphesematous cholecystitis that presents with gas in gall bladder. DM pts are at increased risk. For an uncomplicated billiary cholic just do spasmolytic and analgesic therapy and elective surgery is done at a later time.

BioPhysical Profile
BPPis a scoring system to evaluate baby's well being. Its indicated when there is Decreased movement or a non-reactive NST. It includes NST in addition to 4 things, 1-Fetal tone, 2-Movment, 3-Breathing(30/10min), 4-Amniotic fluid inxed(5-20). Each has a score of 2, when present and 0 when absent. 8-10 is normal, and should be repeated once or twice weekly, until term.In presence of OlygoHydramnions (AFI<5) delivery is considerd. If score is 6 w/o OH, Contraction stress test is ordered. If it gives non-reassuring results then Deliver, if it gives suspicious results then repeat the next day. If its 4 w/o OH and fetal lung are mature, Delivery is considered. If lungs are not mature, steriods is gven and BPP accessed w/i 24 hrs. If score is <4, deliver now.

Black widow spider:
Presents with Acute abdomen and best treated with calcium Gluconate and muscle relaxant. Brown Recluse spider causes skin necrosis localized, resembles pyoderma gangreosum. Deep skin ulcer develops. Local excision is tx of choice for the ulcer. Dapsone is used for pts with G6PD def.

Bladder cancer - 2
Up to 80% of pts who go through a urinary diversion procedure, specially an ileal conduit, can develop hyperchloremic metabolic acidosis due to exchange of Cl for HCO3 in the intestinal mucosa, leading to loss of HCO3 and increase Chloride

Bladder Rupture - 2
Hematuria, suprapubic tenderness, non palbable bladder and lower abdominal and perineal edema. The best Dx method is retrograde cystogram with voiding films. Remember for Urethral injury we do Retrograde urethrogram. ****Intraperitoneal rupture is more common in pts in trauma accidents.

Blastomycosis - 2
Anyone in Wisconsin, Ohio, Mississippi with chronic respiratory problem is suspected. Another dx triad is Skin,Lung Cavity and Bone Lytic lesions.

Blood Transfussion
Femur fracture can acct for 1L blood loss, if Pelvic is also fractured the blood loss could be several liters. General guidelines are start iv crystalloids initially, 2L in 10 min, if pt continues sign of hypovolumia then Blood transfusion is started. So the best indicator for transfussion is blood loss of >1500ml.****Washing of RBC washes off antigens asso with transfusion. Its used for IgA def pts. Leukoreduced RBC reduces the risk of allosensitization

Blunt Chest Trauma:
When it happens with wide mediastinumon CXR, aortic injury must be suspected. Either a CT scan or Echo is Dx.

Body Dysmorphic Disorder
Woman thinks her nose is 'enormous'.

Boerhaave's Syndrome
Complete tear of distal esophagus that leads to pneumomediastinum, vs incomplete tear in Malory Weiss and no Pneumomediastinum. Xray shows subcutaneous emphysema. Dx w barium swallow. Tx give antibiotics and thoracotomy and repair of esophagus immediately.

Borderline Personality
Spliting characteristic. You are the best and the other doctor was terrible.

Bordetella Pertusis:
For preventin, all close contacts (houshold and daycare) get 14 day Erythromycin, regardless of age, immunization or symptoms.

Botulism - 2
we have two types: 1-Infantile type, organism gains entry through the food and prduces toxin in the intestinal tract. It’s a protease that blocks Ach release. 2-Adult type the toxin is ingested pesay, produces the effect.****Infntile botulism, tx is supportive only. BUT if Children get it then administer equine derived botulism anti toxin right away.

Bowel Ischemia
Always consider it as an early complication of operation on the abdominal aorta . Pt presents with bloody diarrhea and abdominal pain. Its due to infarction of Inferior Mesenteric artery, 1-2 daya post surgery. DDX:Pseudomembraneous colitis, takes 2-3 weeks after drug therapy. ****** Unrecognized bowel ischemia is one of the mc causes of lactic acidosis with severe atherosclerotic disease. Pt complains of abdominal pain after meals.

Bowel Obstruction
If a pt comes with constipation and no flatulus, even then, unless strangulation or perforation is suspected, bowel obstruction is treated conservatively. So dotn do surgery first. 1st thing to do afyer IV is nasogastric suction and barium enema.

Bowel resection:
In pt who goes under bowel resection the mc type of kidney stone is Oxalatedue to excessive absorption of Oxalate from GI tract. Increased intestinal fat binds calcium which is then unavailable bind oxalate. Therefore increased absorption of oxalate occurs in GI and precipitates in kidney.

BP criteria
BP should be kept below 140/90. But BP in DM and chronic renal pts should be kept under 130/80 to prevent end-organ damage.

BPH - 7
Starts in the center of the prostate. Cancer starts in periphery. The mcc of overflow incontinence in elderly male. Finasteride acts on epihtelium and alpha-1blocker acts on smooth muscles of prostate. Alpha blockers (Doxazocin) are prefered in pt with BPH and dyslipidemia and glucose-intolerance. If Creatinine is elevated do US of kidney, bladder and ureter to check for damages. ***** Tamsulosin, an Alpha-1 receptor blocker has the least SE of all alpha one blockers used for Tx of BPH.****The two initial tests that are recommended in ALL possible bph pts are serum creatinin and urine analysis. Urine analysis rules out infection and creatinin rules out kidney problems.*****US of the kidney, ureter and bladder should be done in pts whose creatinin level is elevated.

Brain Abscess
A pt with acute onset of HA and focal neurological symptoms (cant walk right) after an episode of acute otitis media or sinusitis most likely has brain abscess. CT and MRI show ring enhaning lesion. Fever pesents with only 50% of cases so its not a reliable sign.

Braindeath
is defined as irreversible cessation of brain activities. It’s a clinical dx. The characteristic findings are absent cortical and brain stem functions. The spinal cord may still be functioning, therefore DEEP TENDON REFLEX are intact. EEG can confirm but is not necessary. In brain dead people, pupilary reaction & oculovestibular reaction are absent, Atrpine doesnt accelerate heart since vagal is gone, and there is no spontaneous respiration.

Brainstem Infarction
1-Medial Medulary Syndrome:Occlusion of vertebral A. Contralateral paralysis of limbs, contralateral loss of tactile, vabratory and position. Tongue deviated to affected side. 2-Lateral mid-pontine synd:A lesion in Lateral Pons. Impaired sensory and motor function of CN V (trogeminal) and limb ataxia. 3-Medial midpontine Synd:A lesion in Medial Pons. ipsilateral limb ataxia, and contralateral eye deviation and paralysis of the face,arm and leg. 4-Wallenberg synd: A lesion of laterla Medula. ipsilateral horner synd, loss od pain and temperature of the face, weakness of the palate, pharynx and vocal cords, and cerebellar ataxia. Also loss of pain and temperaturein cotralateral side of the body.

Breaking Bad News Protocol
1-Comfortable environment, 2-Ask pt how much he knows about his symptoms, 3- Ask pt how much he wants to know, 4- Give him a warning shot ( ie its worst that we thought, do you want someone with you), 5-Break the news if he wants that. 6- Tell him of prognosis but also of the option to make hi slife as comfy as possible, 7- Try to explain everything clearly and simple as possible.

Breast Carcinoma - 4
Inflamatory beast cancer: Erythema and edema of non lactating breast could be due to locally advanced cancer, biopsy should be done first to rule out that dx. Tx is 2-3 weeks of combination chemotherapy to shrink the tumor allowing sybsequent extended resection. ****Metastatic Breast Cancer has a poor prognosis. with little chance of cure. Its importnat to choose Local (Surgery) vs Systemic (Systemic Chemotherapy) tx in pt with metastatic breast cancer. ****Tumor burden, based on TNM staging, is considered the single most important prognostic consideration in treating pts with breast cancer. ER+ and PR+ are good prognostic factors. Over expression of Her2/Neu oncogene is worst progosis. ****Breast cancer is the leading cause of metastatic skin disease in women. These lesions are erythematous that present as erosions covered by necrotic skin. Tx is palliative radiation therapy with aggressive wound care. *****Two proibitions when hay breast cancer in pregos: No chemo in 1st trimester, and no Radiotherapy anytime in pregnancy. Also Lumpectomy is not a good choice for 1st trimester cuase it needs Radiation afterwards. So the only Tx for 1st trimester is Modified Radical Mastectomy.

Breech presentation - 2, 6/24/06

If prior to 37 must be left alone. After that External cephalic version may be attempted PRIOR TO onset of labor, given no CI (Hypertension).

Bronchiectasis - 2
An irrevesible widening of medium size airways in the lung. Characterized by destruction of bronchial walls and chronic bacterial infection. They migh thave life threating Hemoptysis. Bronchiectasis is due to formation of large collateral vessels, which have a very fragile wall. Hemoptysis could be very extensive and ALL pt should be admited. ****Any pt with fever, night weats, copious foul smelling sputum has one of the following: 1-Bronchiectasis, 2-Lung Abscess, 3-Anerobic Pneumonia. "Copious foul smelling" sputum is the KEY word. Cxr shows characteristic "Tram Track Appearance" (increased vascular markings) ring shadows, peribeonchial thickening. CT is the confirmatory investigation, it has REPLACED Bronchography. After that Sputum for AFB is done.

Bronchiolitis:
Is defined as the first episode of wheezing asso with an URT infection. The infection is usually caused by CMV and is common in winter. In affects 50% of children in the first two years of life especially those prone to airway reactivity, and there is an increased inidence for Asthma later in life. WBC in nl and Cxr shows air trapping or atelectais. Tx is supportive care and humidified oxygen and bronchodilators.

Bronchogenic Carcinoma. Pulm. 6/3
BGC is the mc lung cancer asso with asbestos exposure, while Malignant Mesothelioma is almost exclusively asso with asbestos exposure but its not the mc malignancy after asbestos exposure. Pleural involvment is HALLMARK of asbestos exposure. Cigarete smoking acts synergicaly with asbestos exposure in increasing risk factor for BGC.

Brown Sequard synd
Asso with damage to lateral spinohtalamic tract, causing contralateral loss of pain and temperature beginning TWO LEVELS BELOW the lesion. Therefore, a lesion of right sided Laterla SPT at T10 will result in left sided loss of pain and temperature at beginng at T12.

Bruton's Agamaglubinemia
T9Q14. X-linked. MERCK: Panhypogammaglobulinemia of male infants characterized by levels of IgG < 100 mg/dL and other Ig levels low or absent, low or absent B cells. onset of infections sometime after age 6 mo. These infants have recurrent pyogenic infections of the lungs, sinuses, and bones with such organisms as pneumococcus, haemophilus, and streptococcus. A defect of the Btk (Bruton's tyrosine kinase) gene at Xq22 prevents differentiation of pre-B cells to B cells. Lifelong IG given IM or IV in the lowest dose that prevents recurrent infection is essential.

Bud Chiari Syndrome

Buergers disease
Triad of occlusive dis of arteries, migratory superficial thrombophlebitis[Thrombi+Phlebe(vein)+Itis(inflamation)], and Reynaud phenomenon in a smoke male. DDX w atherosclerotic disease is that in those Pulses are normal.

Bulimia Nervosa - 2
Outpatient tx include: SSRI antidepressant, cognitive therapy, interpersonal pshychotherapy, family and group therapy. If pt failed this and/or has metabolic problems or is suicidal then Hospitalize. *Pt bing eat and then feel guilty. They might even feel sad about their situation. But they maintian their BMI and are NOT Amenorrheic. They bing eat at least TWICE per WEEK. If they dont do that, they are dx as "Eating disorder, not otherwise specified".

Bullous Pemphigoid- Dermo, 6/3
Is characterized by tense blisters in the flexural areas. Commonly in elderly (>60). The precipitating factors are ultra violet rays, NSAIDS, antibiotics. Autoantibodies are formed against basement membrane. Immunofloresence microscopy reveals diagnostic findings of IgG & C3 at the epidermal-dermal junction and Prednisones are tx of choice. In Pemphigus vulgaris IgG deposits are intercellulary in the dermis. In Herpes there is C3 at the basement membrane zone.

Bupropion - 2
Its used for Major depression, ADHD, cigaret craving. It ma be used in conjunction with Nicotin patches, but such combination requires frequent BP monitering. It reduces weigh gain that comes with cigarett smoking. Although it might cause seizure, you dont stop the drug due to this rare SE, unless pt has a Hx of seizures. ***Pt has hx of epilespy and wants Bupropion, Dont give him Bupropion its CI for his Hx of epilepsy, give SSRI

Burns
Tx of superficial and deep burns. * For inhalation injury which may take a few days to manifest, Dx is best done with bronchoscopy. *****For calculations only consider 2nd and 3rd degree burns. Kids: head=18, lower ext 27. *****when circumferencial full thickness burns involving extremities or chest is present, Escharectomy maybe the best option.*****Parkland formula for ressecitation is 4ml/kg/ % of body burned, half in 8hrs and rest in 16hrs. ****Early excision therapy is indicated for extensive partial thickness and full thickness burns.*****mcc of death in burn pts in hospital is infection.*****Inhalation injury is commonin burn pts and may take several days to manifest. Dx is best done with a bronchoscopy. Beta agonists along with steroids, endotrachial intubation and antibiotics have all been used in pt with inhalation injury.*****1st degree: confined to epidermis, erythomatous skin. Heals w/o scarring. Example is sun burn. 2nd degree:Involves entire epidermis, red and blisters. Its partial thickness burn. 3rd degree: is full thick ness burn, epidermis and ermis completey destroyed. Not painful. Debridment and grafting is required.

Calcanium Fracture: Surgery 6/2
If due to fall, evaluate for other injury, plain film of head, neck, abdominal, lumbar & pelvic. Its asso with compression fracture of thoracic spine.

Cancer Drugs
1-Anorexia: doc for anorexia asso w cancer is Megesterol Acetate. 2-Nausea&Vomit: Metoclopropamide and ondansentron

Carcinoid syndrome –3, 6/3
Triad of flushing, diarrhea and valvular heart disease. Its asso with carcinoid tumors and hepatic metastasis. Isolated tumors w/o metastasis do not produce carcinoid syndrome. These tumors produce serotonin. Elevated serotonin and its metabolite (5HIAA) are in plasma and urine. Tryptophan is the precursor of Serotonin . Tryptophan is the aa used in synthesis of Niacin. Pts of Carcinoid synd are at risk of Niacin deficiency due to increased formation of serotonin from tryptophan. As a result supply of tryptophan is decreased and 3 Ds of Niacin def (Diarhhea, dementia, dermatitis) of Pellagra develops. ****Carcinoid tumor when symptomatic is in Small bowel, when asymptomatic its in Appendice.***Triad of flushing,diarrhea and wheezing.

Cardiac Contusion
Is asso with arrythmias, so the first thing to do if you suspect it, is Continues ECG NOT Echo.

Cardiac Temponade - 2
Characterized by 1-hypotension, 2-Sinus tachycardia. 3-Pulsus paradoxus. 4-Prominent JVD with 'Y' descent. US shows blood in pericardial sac. DDX is Medicastinal hemorrhage, which is the same as CT except that US shows no blood in pericardium and the blood is in mediastinum. It could happen in pts who are taking warfarin and cause coagulation abnormality.

Cardiomyopathy, Dialated - 3
Characterized by 1-Impaired systolic function of left and right ventricle leading to progressive cardiac enlargement. 2-Cxr shows marked or moderate enlagemnt of cardiac silouette. 3-Echo shows systolic dysfunction and left ventricle dilatation with Normal wall thickness. Pt should refrain from drinking alcohol. Viral infection is the mcc of myocarditis that results in DCM, and the mc virus is Coxsackie virus. .****DCM is the end result of myocardial damage produced by toxic, infectious, or metabolic agents. Viral or idiopathic cause is mc by Coxsackie virus. The dx is by Echo, shows dilated ventrilces with diffuse hypokinesia resulting in low EF (systolic dysfunction and CHF). Concentric Hypertrophy is seen in Aortic stenosis. Eccentric in Valvular regurgitation. Hypokinesia is due to MI inferior wall. MS has Left atrial hypertrophy. HCM shows Asymetric vent septum hypertrophy.

Cardiomyopathy, Hyper - 5
Characterized by 1-Asymetric left ventricular hypertrophy. 2-Harsh systolic Diamond shape murmur best heard at the left sternal border. 3-Cxr shows mild enlargement of cardiac silouette. 4-Echo shows vigorous systolic function, Asymetric septal hypertrophy and Systlic anterior motion of the mitral valve. 5-Due to hypertrophy of left ventricul there is Diastolic dysfunction. * Beta blockers are the tx of choice for isolated ventricular diastolic dysfunction. MERCK:Systolic murmurs are usually present, but patients with apical and symmetric hypertrophic cardiomyopathy may have no murmur. Most common is a crescendo-diminuendo ejection-type murmur that does not radiate to the neck; it is best heard at the left sternal edge in the 3rd or 4th intercostal space. This murmur is caused by obstruction of left ventricular ejection (produced in systole when the hypertrophied interventricular septum and the anterior leaflet of the mitral valve approach each other). A mitral regurgitation murmur due to distortion of the mitral apparatus is heard in some patients. It has a characteristic blowing quality and is best heard at the apex, radiating toward the left axilla. Rarely, early or midsystolic clicks are heard. In some patients with right ventricular outflow tract narrowing, a systolic ejection murmur is heard in the second interspace at the left sternal border. An S4, almost always present, indicates a forceful atrial contraction against a poorly compliant left ventricle in late diastole. Mitral regurgitation is as a result of anterior motion of the mitral valve leaflet. Mitral regurgitation in "Infective endocarditis" or "trauma" is caused by rupture of chordae tendinae. Mitral degeneration in "elderly women" can be caused by mitral annulus calcification. Mitral valve prolapse is the mcc for "isolated mitral trgurgitaion" in north america. * Echo is dx of choice, shows asymetrical ventricle septal hypertrophy. *in 25% of HCM pt there is obstruction of LV ourflow tract (echo shows anterior motion of mitral valve) , becauseof this filling preffure is furthur elevated and out is compromised. This outflow gradient is incresed by manuvers that reduce cavity size of left ventricle. Valsava and standing after squatting, decrease LV Vol thus increasing the gradient and intensify the murmur. But Handgrip, increases systemic arterial resistence and so decreases gradient and the murmur. Leg elevation also decreases the murmur.because it increases LV vol. *****To screen young athlets for HCM Echo is non sensitive. Do detailed personal,family Hx and PE.****Tx for HCM is Beta blockers.***HCM is Auto DOMINANT. Pt is young, dyspnea on exertion, harsh diamond shape systolic murmur at the left lower sternal border.Echo shows Asymetrical septal hypertrophy.

Cardiomyopathy, Restrictive - 4
Characterized by 1-Severe Diastolic dysfunction due to a stiff ventricular wall. 2-Echo shows symetrical thickening of the ventricular wall. 3-Kussmal sign. 4-Apical impluse palpable. DDX: Constrictive pericarditis no no 4, cxr shows calcification, and normal thickness of ventricular wall. ****Tx of most causes of RCM is useless except Hemochromatosis, Phlebotomy and Iron chelation with subcunatous defroxamine may result in substantial improvement.*****Since heart cant relax filling is compromised so both Liver and Lung are congested.****Xray shows mild cardiac slouette. Echo shows symetricly thickened vent wall and near normal systolic function. "Speckled Pattern" is specific for Amyliodosis .

Caroli Syndrome
Is a rare congetnital disorder characterized by intrahepatic dialatation of billiary tree, asso wi APKD.

Carotid Artery Stenosis & Endarterectomy
Asymp pt with 66-99% are considered for surgery, 100% is CI for surgery.

Carpal tunnel syndrome - 2
most likely location of pathologic process is the wrist. Pt presents with pain and burning sensation of the first three fingres and atrophy of thenar eminence, poor 2-point discrimination over the thumb, and they keep dropping things. Its seen in asso with RA, Myxedema, Sarcoidosis, amyloidosis and Leukemia. Most specific test is Nerve conduction study. Tinel test (tapping on Median nerve) Phalen test (90 degree flextion of both wrist and pushing them together dorsally), Carpel compression test (applying pressure over carpal tunnel) are not specific.

Cat Bite - 2
Should be tx prophylacticaly with Amoxicillin/Calvulanate for 5 days, due to fear from Pasturela Multicoida.****1-Pasturella Multicoida, occurs after cat or dog bite, and there is an intense inflamatory reaction w/I 24 hrs of the bite. Pain, swelling, purulent discharge are features. 2-Bartonella Hensalae, occurs after cat scratch or bites. Clinical features occur after 3-10 DAYS. . They include papular or vesicular lesion, at the site of injury and proximal Lymphadenpathy.

Cat Scratch Disease
By Bartenella Hensalea. It most commonly presents with localized cutanous and lymph node disorder near the site of inioculation. A local skin lesion evolves through vesicular erythematous and papular phases, but can be postular or papular. Dx is by clinically and antibody to B. Hensale or a positive Warthin-Stary stain on the tissue specimen.Most people resolve gradualy w/o therapy. However, tender lymphadenopathy and systemic symptoms require five days of Azithromycin.

Cataract congenital -2
Its due to progresively thickening of the lens. In "Congenital cataract" the retina CAN’T be visulized properly, exam reveals bilateral white reflex, the mcc of white reflex in the pediatric population is Congenital Cataract. Tx is extraction of the lens.

Caustic Poisoing - 2
Upper GI Endoscopy is the dx study of choice when a person comes in with ingestion of Alkali (oven cleaner) in the first 24 hr to assess the damage.

Cavernous sinus thrombosis
Presents with severe headache, followed by fever and periorbital edema. Also CN involvements in the form of opthalmoplegia, lateral gaze palsy, ptosis and dilated pupils. Nasal discharge and blood should be cultured. CT scans of the cavernous and air sinuses, orbit, and brain should be performed. Treatment with high-dose IV antibiotics, nafcillin or cefuroxime should be started, pending culture results. Surgical drainage of the infected air sinus may be indicated, especially if there is no response to the antibiotics in 24 h. The prognosis is grave; the mortality rate remains about 30%, despite antibiotic therapy. DDX:Orbital Cellulitis:Its unilateral, and more common in children. Presents w abrupt onset of fever, proptosis, restriction of extraocular movements and swollen red eyelids. there is NO CN dysfunction or visual disturbances in the early stages unless it spreads to cavernous sinus.Treatment with antibiotics, cephalexin should be started, pending culture results. Incision and drainage are indicated if suppuration is suspected or if the infection does not respond to antibiotics.

Celiac dis - 2
Pt present swith malabsorption, loss of muscle or subcutanous fat, pallor due to iron def anemia, bone pain due to osteomalasia, easy bruising due to vitK def and Hyperkeratosis due to VitA def. Hay fatigue and weight loss. Dx is with ELISA for IgA antibodies to gliadin and immunoflorescence for IgA antibodies to endosomysium. ALso antibodies against rtansglutaminase. But CONFIRMATION is small intestine biopsy. ****A 15mo old girl with dermatitis herpitiformis (erythematous vesicles symetrically distributed over the extensor surfaces of elbows and knees.) and chronic non-bloddy diarrhea with malabsorption (foul smelling stool) and distended stomach, is suggestive of CD.

Cellulitis - 2
Is an inflamation of skin that could extend into deeper tissues. In majority of pt is caused by Strep hemolytic or Staph aureus. Clinical symptoms can be Systemic as well as Local. Local findings are swelling, erythema, warm and tender and less well demarcated than Erysipela. The systemic signs are high grade fever chills and rigors, malise and confusion. When systemic signs are present IV Nafcilin or Cefazolin is preffered. **** The senario is usually a lady with painful leg DDX are:1-Cellulitis, high fever and chills. 2-DVT, cellulitis of calf is the one when there is high fever and no risk of DVT. 3-Necrotizing facitis is a deep seated cellulitis, suspect it in pt with bulla or crepitus. 4-Erysipelas, is a superficial cellulitis, it usually attacks cheeks, area is erythematous, painful and raised, with vesicles or bullae. No lymphangitis. 5-Erysipeloid is an edematous, purplish plaque with central clearing. Its caused by Erysipelothrix incidiosa. Usually at hands of fishermen and meat heandlers, its not as painful as cellulitis and there is no fever.

Central Cord Syndrome:
Characterized by burning pain and paralysis in the upper extremities with relative sparing of lower extremities. It is commonly seen in elderly 2ndary to forced hyperextension of the neck.

Central line complication:
Include pneumothorax, sesis and temponade occurs in 1-5% of pts. Cxy confirms that the tip is proximal to cardiac silhouette.

Central retinal Artery occulsion
Sudden painless loss of vision in one eye, however opthalmoscopy reveals pallor of the optic disk, cherry red fovea, cotton woolspots, retinal hemorrhages.

Central retinal Vein occulsion 2-
Sudden painless loss of vision in one eye, however opthalmoscopy reveals disk swelling venous dilation, tortuosity, retinal hemorrhage and cotton wool spots.

Cephalohematoma
Is a benign bleeding of newborn's scalp. It’s a sub-periosteal hemorrhage. It shows like a swelling. No tx is necessary. DDX:Caput succedaneum is a diffuse and ecchymotic swelling of the scalp. It may extend across the midline

Cerebral Hemorrhage
If its due to excess warfarin (PT is icreased) then FFP reverses the effect.Pt on anticoagulants should be on INR measure check. So if 1-INR <5,>9 DO stop warfarin and give oral VitK. Pts with serious intracranial bleeding cant wait for VitK, give them FFP right away to bring INR<1.5. Now to reverse Heparin give Protamine sulfate.

Cerebral infarction
Hypodense on CT ( white area over the cerebral surface).

Cervical cancer – 2.OBGYN 6.2
Risk factors: Young age at first coitus (<20). Young age at marriage and first pregnany. High parity, multiple sex partners, smoking, and low socioeconomic status. ****If pap is dysplasia, perform colposcopy. If it shows inflamatory Atypia then repeat after 4-6 weeks. ****If pt comes in with spotting, and you see the cervix having a gross lesion that bleeds by touching, dont even bother for PAP, go straight to Punch biopsy to rule out cancer. **** Once pt had the cancer check for cytology every year not every 2 year for normal people.

Cervical Spondylosis
It affects 10% of people >50. Hx of neck pain is typical. Osteophytes are the mc findings in cervical radiography in pt with CS. Bony spurs are the mc findings.

CGD
A defect of phagocytic cells Due to NADPH oxidase def. Leading to recurrent infection with catalase positive organism, Aureus, seratia, Klebsiela, Aspergillosis. Not suseptible to catalase neg (strep, influenza, Pyogenes). The MC clinical findings are lymphadenopathy, hypergamaglobolenemia, hepato and splenomegaly, anemia of chronic causse, short, gingavitis and dermatitis. NitroBlueTetrazolin is dx. Tx is prevention with trimeta-sulfa and Gamma interferone 3 times a week. BMT is curative. DDX1:Wiskot aldrich:Eczema,thrombocytopenia,recurent infection with encapsulted organism. Manifest at birth, petechia, bruises, circumcision bleeding, bloody stools. DDX2:Chediak Higashi, decreased granulation,chemotaxis and granulopoesis.Finding of neutropenia, ginat Lysosome in neutrophil will confirm Dx. Tx includes prevention with Trimeta-Slfa and daily ascorbic acid. DDX3:Jobs syn (Hyper IgE), chronic pruritic dermatitis, recurent staph infection, marked elevated IgE, eosinophilia and coarse facial features.

Chaga's dis
Caused by insect borne Trypanosoma Cruzi which is a common form of carditis in Centerl and South America. Pt presetns with Cardiomegally, conduction anomalies. Almost all pts have a hx of Megacolon or Megaesophagus.

Chalazion
a small swelling(NODULE) of the eyelid. It results from obstruction of the glands in the eyelid. First try to cure it by putting hot compress on it if it comes back again and again then do histopathology. It often requires surgery. Recurrent chalazion requires histopathologic exam because there is a risk of underlying Squamous cell carcinoma. DDX Hordeulom(Stye). Occurs at the edge of the eyelid(pretty red)

CHD risk factors:
Age (men>45, Women>55. Family Hx of premature CAD (<55 in first degree male parent, <65 in females relatives). Hypertension up to or equal to 140/90 even controlled w medication. Cigarret smoking. DM. HDL <35. HDL up to or equal to 60 negates one risk factor.

Risk factor LDL goal LSM Meds
CHD or Equiv <100>=100 >=130
> or =2 <130>=130 >=160
0 or 1 <160>=160 >=190

Chediack Higashi:
Is a storage granulocyte abnormality resulting in hepatosplenomegaly, lymphadenopathy, anemia, thrombocytopenia, and susceptibility to infection in childood.

CHF - 6
CHF is a common cause of Pleural Effusion. Pleural analysis is consistant with transudate effusion. The effusion caused as a result of systemic factors (CHF) is Transudate. The effusions caused as a result of local factors is Exudate effusion. Existence of at least one of the following indicates exudate, if non exist its a transudate. 1-Pleural/Serum Protein is >0.5, 2-Pleural/Serum LDH is >0.6. The determination of pH is important in parapneumonic effusion in which a value of <7.2>100 is dx for CHF. BNP is like ANP but BNP is released from Ventricles vs ANP from Atria.Also remember in CHF pt sodium is reabsorped in kidneys in response to renin-angiotensinogen -Aldosterone system, therefre sodium in urine would be low .****ACE inhibitors increase survival rate in CHF pts, so cosider it in a pt with E<40%. Also out of all diuretics, Spironolactone is the only one that improves survival. *** Drugs that improve survival are B-blocers, Spironolactone, ACE inhibitors (Captopril and Losartan), and Aspirin. Digoxin helps the situation but NOT survival.

Child Abuse steps
A19-

Choanal Atresia
Suspect it in a infant who presnts with cyanosis that is aggrevated by feeding and releived by crying. Failure to pass a cathater through the nose is sugestive of dx. Dx is confirmed by CT with intranasal contrast. The first step in mgmnt consist of placing an oral airway and lavage feeding. Definite tx is repairing the obstruction with surgery.

Cholecystitis, Acute - 3
MM-402. Dx stages: 1-USG(initial workup), 2-HIDA (very specific, for confirmation. Its particulary useful in dx of Acalculus cholecystitis). ****Triad of acute RUQ pain, fever and leukocytosis. Pain radiates to scapula. Billirubin is normal and Murphy (pain on deep inspiration) exist. Its most commnly 2ary to gallstones. , in these pts it mc due to impaction of stones in a cystic duct. The inflamatory response results from any of the following, 1-Mechanical(increased intraluminal pressure), 2-Chemical(release of tissue factors) 3-Bacteria(2ary to stasis), this occurs in 50-70% of cases. REMEMBER although in 50-70 of cases there is infection due to bactreia, but the cause for AC is mc due to impaction of the stone in cystic duct.**** If pt still has pain after cholecystectomy, and ERCP shows sphincter of Oddi dysfucntion, then ERCP with sphincterectomy is the procedure of choice.**** After ERCP and shingterectomy, if pt has normal LFT and no dilation of biliary tree with US, then Oddi and CBD can be ruled out and pt is having FUNCTIONAL PAIN. Give analgesics and reassurance.**** Acalculus Cholecystitis occurs in CRITICALLY ill pts and imagin studies show thickening of gall blader wall and presence of pericholesistic fluid. Etiology might be stasis of bile ducts and ISCHEMIA of the gall bladder (after accident with loss of blood).

Cholecystectomy: Hepatology, 6/2
Post cholecystectomy pain most commonly is due to either Common bile duct stone, Sphincter of oddi or Functional pain. If LFT is normal and no dilatation of biliary tree then its functional pain, tx is symptomatically with analgesics and assurance. If pt has abnormal Alkaline phosphatase and dialation of billiary tree on US, then we do ERCP to confirm and treat, by stone removal or sphincterectomy.

Choledochal Cyst
congenital abnormalities of the billiary tree characterized by dialation of intra and extra hepatic billiary ducts. Presentation vary with age. An infant presents with jaundice and passage of acholic stools. In children it causes abdominla pain, jaundice and attakcs of recurrent pancreatitis, which maybe evident by increase inamylase and lipase. Adults present with vague epigastric or RUQ pain or Cholangitis. Choledochal cyst could degenerate into cholangiocarcinoma. Initial investigation of choice is US followed by CT or MRI. DDX1:Caroli's Synd, congenital disorder of intrahepatic dialation of bile ducts. DDX2:Biliary Atresia presents in infancy with marked obstructive jaundice and acholic stool.

Cholelithiasis - 3
Tx: 1-Asymptomatic pt, leave it alone. 2-Symptomatic pt, if ok with surgery choice is Laparoscopic Cholesystectomy, if surgery is CI or pt declines surgery then Ursodeoxycholic acid 10 mg/kg/day reduces biliary secretion of cholesterol and decreases the cholesterol saturation of bile, resulting in gradual dissolution of cholesterol-containing stones in 30 to 40% of patients.****There are 3 types: 1-Cholestrol, 2-Pigment stones(mostly calcium bilirubinate, 20%) and 3-Mixed stones. Water insoluble cholestrol is secreted in bile where its converted into soluble miscles by bile acids and phospholipids. If too much cholestrol and too little bile then cholestrol crystals precipitate. Predisposing factors are Fat,Femlae,Forty,Fertile(OCP), cloFibrate. Remember 80% of stones are radiolucent so xray cant see them.

Cholesteatoma, ENT 6/2
Causes acquired conducting hearing loss in CHILDREN .Its not a tumor. Its an Epithelial Cyst that contain desqumated Keratin. It could be acquired secondary to Otitis media or Eustachian tube dysfunction. Infection is usally due to Pseudomona. Pt presnts with recurrent infection. MC sign is drainage and granulation tissue and debris unresponsive to antibiotics & marginal tympanic perforation. They destroy bone. CT can detect defected bone. Tx is surgical removal. DDX Chronic Otitis media where there is no debris and granulation.

Cholesterol embolizarion
Or Atheroembolic disease, follows surgical or manipulation of arterial tree ( ie Angiography), due to showering of cholesterol from aorta or other major arteries. Its mc seen in elderly pt with evidence of diffused Atherosclerotic dis.. Renal failure, Livedo reticularis, sstemic eosinophilia, and low complement levels, should ake you think of this. Tx is conservative, antocoags should be stoppedsince it may prevent healing of the ruptures Plaque. Physical exam shows painless, redish blue mottling of the skin of the extremities.

Cholesterol lipid profile - 3
For CHILDREN the recommendation is: A child with a parental hx of elavated total cholesterol (>240mg/dl) or a chld with risk factors for CAD should be screened for total cholesterol level. If its >200mg/dl then we do a fasting lipid profile test. For screening we use HDL and Total Cholesterol. For Tx guidelines we use LDL levels.

Cholestyramine:
A bile acid sequesterant that lowers LDL and mostly increase HDL when combined with statin. In addition to binding bile acids in gut it also binds other drugs and reduces their bioavailibility so the pt needs higher doses ( ex hypothyrism )


Chorioamnionitis
Pts present with fever >38, uterine tenderness, irritability, elevated WBC and fetal tachycardia.Its asso with preterm or prolonged rupture of membrane. Fetal tachy could also be caused by Beta-2 agonist for tocolysis. Elevation of WBC could also be caused by steriods admin. Amniotic fluid cultures are gold for Dx (Nitrazine paper test). Once Dx is established samples are taken for culture and then Ampicciline and Gentamcin are given. Labor should be induced. If cervix is unfavorable C-section is done.

Choriocarcinoma
It’s a malignant tumor of the trophoblastic tissue. Clasically prestns w Hemoptosis, but it could also present with shortness of breath and chest pain. In any postpartum female you should suspect Choriocanrinoma. Quantitative Beta HCG is important in Dx. So once you have postpartum woman with hemoptosis chest pain and shortness of breath then we need to do Cxr, pelvic exam and BetaHCG.

Chronic Liver Disease
Do Merck. Asso with respiratory alkolosis.

Chronic Mesenteric Ischemia
suspect it in pt with chronic abdominal crampy pain, weight loss and people who don’t eat food because of pain and other malabsorptive symptoms. Evidence of Atherosclerotic dis is present. Abdominla exam might reveal bruit in 50% of pts. Dx needs angiography and Doppler US.

Chronic renal failure - 2
Secondary hyperparathyroidism with resultant renal osteodystrophy (loose calcium and retain phosphate) is almost universal in CRF.****Normochromic normocytic anemia due to eryhtropoetin def is a very common I pt with End Stage Renal Failure. Recombinant Eryhtropoetin is the tx of choice, however, Iron supplemnt should be given BEFORE erythropeitin. All Chronic Renal Failre pts hct<30,>3-3.5 mg/dl.

Churg Strauss
A20. MERCK:Its one of the group of diseases of known or unknown etiology characterized by eosinophilic pulmonary infiltrates and, commonly, peripheral blood eosinophilia.allergic granulomatosis (Churg-Strauss syndrome), a variant of polyarteritis nodosa with a predilection for the lungs.

Circumstantiality
is a thought disorder that answers in un-necassary details that deviate form the topic but eventually goes back to the topic. DDX is Tangentiality which is an abrupt permanent deviation from the topic. DDX2:Loose asso which there is no asso b/w sentences.

Cirrhosis - 3
Could happen 2ary to alcoholism. Pt has ascites, and esophageal varices 2ary to portal hypertension. Prophylactic tx of pt wth large varices who have never bled with propranolol significantly decrese the risk of futur bleeding. *In a pt with refractory ascites , refractory hydrothorax, and recurrent variceal bleeding, TIPS (Transjugular Intrahepatic Portosystemic Shunt). is used.****Alcoholism is the mcc of cirrhosis in US. 33% alcoholics, 10% HBV, 20-30% HCV(the mcc of liver transplant in US).****Pts with cirrhosismay have upper GI bleding due to : Erosve gastritis, PUD, Mallory-Weiss tear. Sclerotherapy isindicated for first varices, but not prophylactically. For PUD do consertavie mngmnt, if that didnt work then we do surgey, Excision of ulcer and vagotomy and pyloroplasty.

Clavicle Fracture
In bew borns it presents with irregularity,crepitus and fulness over the fracture site and decreased movemnt of the arm. Predisposing factors ar shoulder dystonia, traumatic delivery, large infant. No tx is needed.****For Adults do a figure of 8 bandage. *****Clavicle fractures that are displaced can damage subclavian artery, Artriogram is needed to rule out injury. Next step would be nerve donduction studies to rule out Brachial plxes injury. If fracture is in distal third then may require open reduction and internal fixation. Proximal and middle third are treated with closed reduction and figure of eight brace.

CLL (Chronic Lymphocytic Leukemia - 3
MC Leukemia in Western countries. In older pts. Mostly asymptomatic and discovered accidentaly. Smudge cells. In general don’t need to do lympb node biopsy to confirm dx, but if you want to a highly specific biospy is available to confirm dx. ****DDX:CML, presents with LEUKOcytosis with left shift Imyelocytes, neutrophils) not LYMPHOcytosis.****Smodge cells (leukocytes that break down because of theri greater fragility) are charcteristic. Staging is directly related to prognosis, stage 0= Lymphocytosis only, Good; StageI=Lymphocytosis+Adenopathy, Fair; StageII=Splenomegally present, Fair; StageIII=Anemia present, Intermediate; StageIV=Thrombocytopenia,Poor. Mean survival is 8-10 years.*****To CONFIRM dx do lymph node biopsy.

Clomiphene Citrate
Is an antiestrogen acts by competitive blocking of receptors of hypothalamous, inhibiting the negative feedback that estrogenhas on GnRH and consequently insreasing Lh & FSH and improving ovulation. Along with hMg and hCG itsindicated for chronic ovulation. SE are hot flashes, breast discomfort, spotting. DANAZOL is an androgen derivative that has gonadotropin inhibitory effect . Its indicated in Endometriosis, Fibroids and Fibrocystic breast disease.

Clonazepam toxicity
Clonazepam is used for insomnia. In elderly pt it could cause memory disruption. The next step in mngmt is to discountinue it.

Clozapine se
Agranulocytsis.

Club foot
Or Talipus Equinovarus. Calcaneum and talus are in equines and varus position. Initial mgmnt involves non-surgical methods (stretching and manipulation of the foot, followed by serial plaster casts, splint or taping). Surgical tx is indicated if that didn’t work , its performed b/w 3 and 6 month of age.

Cluster headache - 2
Tx for acute attack is 100% oxygen & subcutaneous Sumatriptan. *Presents with acute , sever retroorbital pain that wakes pt up at night. Maybe accompanied with redness of ipsilateral eye, tearing , runny nose, and Ispilateral HORNER synd (Ptosis, Myosis, Anhydrosis). Prophylaxis is key to mgmt, with verapamil, lithium and ergotamine.

CML
There is increased mature granulocytes like segmented neutrophils and band forms. BM shows hypercellularity with prominent granlocyte hyperplasia. When pt is in Crisis phase, IMATINIB is DOC. It’s a tyrosine kinase inhibitor that block signals w/I cancer cells. SE are mild naseau, diarrhea, leg cramps and swelling of the face and itchy rash. It has chenged the prognosis with CML.

CMV Pneumonitis
Is seen in 15-20% of Bone Marrow Transplants with case fertality of 84-88%. Pt presents with dypnea, cough and fever. Cxr shows multifocal diffused pathy infiltrates, and ground glass attenuation, parenchymal opacification or multiple small nodules on high res CT. BAL is dx in most cases. IT IS NOT SEEN IN IMMEDAITE post transplant period, wich is DDX with bacterial and fungus pneumonitis. PCP is also seen in immediate post transplant but its occurance has decreased dramatically due to routine prophylactic use of tri.sulfa in pre-transplant period.

Coarcation of Aorta
Present with rib notching (the 3 sign). HA is a presenting sign. Hay HT in upper extremity. Cxr shows dilatated ascending aorta and subvlavian artery. Indentation of aorta at site of coarcation and pre and post stenting dilation is called the '3' sign.

Cocaine Tox - 3
Pt presents with EKG abnormality of st depresion (ischemia and infarction), HT and excrutiating chest pain. Tx is Benzodiazepine, Nitrate and aspirine.If pt has MI then first line is Cathaterization. ****Fetuses exposed to cocain abuse exhibit intracranial hemorhage., nerotizing enterocolitis and cardaic defets and GU malformations. ****Could cause MI due to causing vasospasm, threre is blood in narises and dilated pupils . He has no risk for MI and is only 27.

Coccidiomycosis
Is endemic in California Arizona and new mexico and texas. Primary Pulmonary infection has non-specific features life fever, fatigue, dry cough weight loss. Cutaneous Erythem multiform and erythema nodosum anf arthralgias might be. Blastomycosis cutaneous dis is verrucous or ulcerative.

Colon Cancer
FOBT is the mc used screening test for colon cancer. Pts should be followed with colonoscopy.

Colorectal Cancer
MC presenting symptom is bleeding!

Communicable dis
If pt's dis could harm others he should be tx against his will. Senario is a man with Meningitidis and fever 104 for 2 days who wants to be tx at home. Answer is treat him in hospital agains his will since he will be harmful to others at home.

Compartment Synd
Dx is made clinically with pallor, pain, pulselesness, paralysis and paresthesia. PAIN on passive extension of fingers is the most sensitive marker of CS. Pain is persistant,progressive, unrelieved with imobilization and out of proportion to initial injury. CS is cused by increased pressure w/i an anatomical space.

Complex Patial Seizure
Breif episodes of impared consiousness, failure to respond to varius stimuli, staring spells, AUTOMATISM( Lip smacking, swollowing), and post-ictal confusion. EEG is usually normal. DDX1,Typical Absence seizure might have lip smalcking but they have n post-ictal confusion.

Conduct disorder
Charcterized by disruptive behavior that violate basic social norms for at least one year in pt <18 yo. Like stealing, setting fire, fighting, animla abuse. DDX is Antisocial disorder is when these boys become adults.
.


Congenital Adrenal Hyperplasia- 2

Hyponatremia, HyperKalemia, Hypoglycemia, and metabolic acidosis. Its due to 21-Hydroxylase deficiency. Its auto recessive. Deficiency of both glucocorticoids and mineralocorticoids. Male infants will NOT have ambigous external genitalia unlike female infants, thats why male infants go on un-noticed until 2-4 weeks when they present with salt wasting. Treatment of 21-hydroxylase deficiency is with glucocorticoid replacement. *****Adolescent onset of hirsutism and virilism with normal mensturationand elevated 17-OH Progesterone.

Congenital Diaphragmetic Hernia:
In all emergency cases remember TX supercedes Diagnosis. The 1st step if oyu suspect CDH is to place orogastric tube and connecting it to a continues suction in order to prevent bowel distension and further lung compression. Bag-and-mask is to be avoided because this can cuase the stomach and intestine to become distended with air, further compromising lung funx.

Congenital heart defects
1-TOF: VSD. 2-Down: Endocardial cushing defect. Also ASD ( L to R shunt). 3-Turner synd:Coasrcation of aorta. 4-Congenital rubella: PDA.

Congenital Hypothyroidism:
The mcc is thyroid dysgenesis, 85% of cases. Infant has apathy, , weakness, hypotonia, constipated, sleeps a lot, large tongue, umbilical hernia. Screening is by T4 and TSH levels, Tx is Levothyroxine

Congenital Rubella;
Triad of sensorineural deafness + Cardiac malformation (PDA & ASD) + Cataracts. There could also be thrombocytopenia and purpule skin lesions ( Blueberry muffin spots). If transmission occurs in first 4 week of pregnancy the risk of developing CR is 50%, it drops to 1% in third trimester. The child might show symptoms when he is 2 years old.

Congenital Toxoplasmosis:
Triad of Chrioretintis + Hydrocephalus + Intracranial calcification. Look for pet in the picture somewhere.

Conjunctivitis, Neonatal - 2
Hay 3 causes for NC in US. 1-Chemical is the mc of the red eye presents at 1st 24hr of life. About 80% who receive prophylaxis w silver nitrate (to prevent gonococcal) experience mild conjuncitivis and tearing that resolves w/I 24 hrs. 2-Gonococcal: is acquired through contact with infected vaginal secretions, it occurs 2-5 days after birth, it presents as Copious purulent eye discharge with swellen eyelids & Chemosis(conjunctival edema), dx is by obtaining a smear and culture of the discharge, tx is a single intramuscular dose of ceftriaxone. 3-Chlamydia:Trachoma,presents with mild hyperemia and scant mucoid eye discharge and pannus (neovascularization) formation, it occurs b/w 5-14 after birth. TX is Systemic Erythromycin to decrease risk for Chlamydial pneumonia.

Constrictive Pericarditis. - 2
Asso with TB in immigrants.****Etiology could be early TB in life. The early third hear sound, called Pericardial knock and the respiratory increase in JVD (Kussmal sign) are important findings. Kusmaul is also present in right side hear failure, sever tricuspic regurgitation ,RV infarction and cardiac temponade. ******CP will lead to inability of ventricles to fill during diastole and would furthur cause the signs of decreased cardiac output (fatigue) and signs of venous overload like JVD, dyspnea, ascites, Kussmaul, pedal edema tender hepatomegaly. Sharp ‘x’ and ‘y’ descent on central venous tracing is the sign of CP as is Pericardial Knock (early sound after S2).

Contraception
Lactation is a contraceptive methid in itself because prolactin inhibits GnRH thus preventing ovulation. However, it is not a reliable methis. If a woman wants contraception right after giving birth, give her Minipill (Progestin only pills), Don’t give her Combined OCP because Estrogen may cause decrease in milk production.

Conversion disorder - 2
Tx is psychotherapy. Pt looses eye site in stressful situation. *** If pt comes with bilateral leg paresis, give him Sodium Amytal and he will dramatically improves.

COPD - 8
If pt is not crashing, the first line of tx is non-invasive positive pressure ventilation before intubation. All pt with PaO2<55>55 should be started with home therapy if PaO2 falls <60mmhg. Smoking cessation and home oxygen therapy are two modalities that can reduce mortality in pt.****Acute exacerbation of COPD is tx with combination of inhaled /nebulized bronchodialators and systemic steriod (Methyprednisone).*****In a pt with multifocal atrail tachycardia due to exacerbated CODP Theophyline and Beta Agonist (Albuterol)(remember beta agonists decrease potasium and worsen arrythmia) will WORSEN tachycardia. Give Oxygen to stop the arrythmia.****COPD comprises lung OBSTRUCTIVE diseses, which includes Chronic Bronchitis and Emphysema. The airflow obstruction is irreversible. A chronic smoker with a productive cough and dyspnea on excertion, with decreased FEV1/FVC is suggestive of COPD. Chronic Bronchitis is characterized by chronic cough for at least 3 months for two consequetive years.The presence of chronic productive cough, prominant bronchovascular marking, mild flatening of diaphragm, and normal DLCO is suggestive of CB rather than Emphysema. These are called Blue Bloaters (for Bronchitis) they have heart failure sings and profound oxygen desaturation. Emphysema is destruction of alveolar wall, Pink Puffer (emPhysema) , hyperinflation of the chest, Decreased vascular marings, Decreased DLCO(due to destruction of alveoli) and moderate oxygen saturation. Now remember Sarcoidosis, Silicosis, Asbestosis, Scleroderma and rheumatoid lung are all RESTRICTIVE lung dis characterized by NORMAL Fev1/FVC. ****The most impact on COPD pt is by Oxygen home tx. The guidline is PaO2<55 and O2 saturation <90%.*** chronic pt comes in with severe sypnea and confusion and profuse sweating. Cxr shows complete collpase of L lung, possibly by a mucus plug (atelectasis is the same after surgery). Tx is emergency Bronchopscopt to remove the plug. And that will improve PaO2.*****Non Invasive Positive Pressure Ventillation is the best option for pts with COPD exacerbation. It should be tried before intubation and mechanical ventilation in COPD pts with CO2 retention.

Core Pulmonale
Combination of Elevated JVP, Hepatomegaly,ascites, and lower extremity edema w/o evidence of pulmonary congestion is suggestive of isolated right heart failure. If there are no RALES it means there is no pulmonary congestion. The mcc of Right side heart failure is Pulmonary disease, and its known as Core Pulmonale. CP is most likely caused by COPD(Smoker) , lees common causes are pneumoconiosis, pulmonary fibrosis.

Court order
T9Q38

Craniopharyngioma - 2
Althought mc in children, they are bimodal and could happen in adults too. In children retarded growth is the mc presentation. In adults hyposexuality. Bitemporal blindness is a classic sign. Dx is MRI or CT. Tx is surgery or radiothreapy.****A young boy with symptoms of increased ICP (HA,Vomit) , Bitemporal anopsia and a calcified lesion above sella has Craniopharygioma until proven otherwise. Presense of a Cystic Calcified parasellar lesion on MRI is almost Dx. DDX is Pituitary Adenoma, where its more frequent in women and Prolactinoma is an important part of it, and there is no CALCIFICATION of the gland.

CREST Synd - 2
Anticentromere antibody is Dx.*****Calcinosis, Reynauds, Esophageal dysfunction, Sclerodactyly and Telangiectasia. It has a better prognosis than diffused Scleroderma.

Creutzfeldt-Jacob - 2
Pt is b/w 50-70, with rapidly progresive dementia, myoclonic and periodic synchronous bi or triphasic sharp wave complex on EEG.Brain biopsy shows cortical spongiform changes. CSF is normal, Death w/I 12 months, NO TX. Spongiform encephalopathy is caused by prion.

Crohn's disease - 2
DDX with UC is non-caseating granulomas.

Croup
Also known as Laryngotrachiatis or LaryngoTrachioBronchitis, is characterized by laryngeal inflammation that results in hoarseness, a barking cough and respiratory distress. Typical pt is <3 and the mcc is Parainfluenza virus. Dx is clinical and lateral xray shows subglotial narrowing. Always give Epinephrine before any invasive tx like intubation. This is ONLY for croup not Epigolitis. Tx is 1st O2, 2nd Epi, 3rd intubate in ER.

Cushing's Syndrme - 3
Due to ectopic ACTH.***Pt with lung cancer and ectopic ACTH production can have Cushing's. ****Dx procedure.

Cutaneous Larava Migrans
Is a cc of dermatological disease in tropical travelers. Its caused while "Sand box handling" and its characterized by serpiginous lesion in the skin. Tx is Applying thiabendazole or mebendazole

CVA
Occurs in middle and late years of life. Could be either Ishcemic (85%) or Hemorrhagic (15%). Ischemic CVA orignates from aortic arch, carotid bifurcation, and obstruct arteries. Clinically atherothrombotic stroke occurs at rest and have a gradual onset. Pt experiecne successive strokes. Babinsky indicates UMN due to major cerebral artery obstruction. Ischemic CVA could also be caused by thrombi from left heartIts asso with sudden onset and preexisting cardiac disease. EKG is characteristic. The mc site islaterl astriae arteries (arteries of stroke) which are branches of MCA, they supplt internal capsule,putamen. So if a pt has normal EKG and cardiac enzyme, this kind of stroke is unlikely.

Cyclical vomitting
recurrent self-limiting episodes of vomitting and nausea in children. Tx is antiemetis and reassurance.

Cyclosporine side effects
1-Nephrotoxicity:the mc and serious side effect. It manifest as acute azothemia or irreversible progressive renal disease. 2-Hypertension:due to vasoconstriction and sodium retension. Ca chanel blokers are doc. 3-Neurotoxicity:Often reversible. Tremor, headache, nasea, seizure, visual problems. 4-Glucose intolerance. 5-Infection: 40% of pt get infection chronically. 6-Malignany: Risk of squamous cell carcinoma. 7- Gingival hypertrophy and hirsutism. 8-GI, mild. Tacrolimus has the same se except hirsutism and gum hypertrophy. Azothioprine se is dose related diarrhea, leukopenia,hepatotoxicity. Mycophenolate se is Marrow suppression.

Cyclphosphamide SE
Bladder carcinoma is a SE. Also alopecia, sterlity, amenorrhea, acute hemorrhagic cystitis.

Cystic Fibrosis - 8
Bronchiectasis due to pseudomona and infertility and recurrent respiratory infections. Cxr showing "Tram Track pattern" and opacities is dx for Bronchiectasis. CF is due to abnormal chloride transportation in all exocrine glands. Sweat chloride concentration >60 is DX. Since there is fat malabsorption, fat soluble vitamins are deficient. So vit K is deficient and since vit K is a cofactor for the enzyme gama-glutamil carboxylase which adds carboxyl group to glutamate residue of factor II,VII,IX & X, and protein C & S, those coagulation factors will be also deficient.*****A routine influenza vaccine is indicated in all CF pts, but not Pneumococcus vaccine. There is asso b/w Pseudomona pneumonia and CF, use Gentamycin+Pipercilline. *****Tx of an acute severe exacerbation of lung dis in CF is IV Pen/Cephalo+Gentamycin. *****Aut rececive, we need to know both parents DNA status to determine child's possibility of having CF. *****The mc mutation is a DELETION of a three base pair coding for Phenylalanine (DjO8) in the CFTR gene in CH7.****Pts with CF present with Meconium Ileus characterized by bilious vomiting, failure to pass meconium at birth, and ground glass apperance on abdominla xray.*****Suspect it in a pt with Bronchiectasis(cough productive of sputum for 3 months) and Malabsorption (foulsmelling stool) due to pancreatic insufficeny. One clue if the pt starts to show in his 20s is a hx of Meconium Ilueus (intestinal obstruction) as a neonate. Whenever hay acute exacerbation of pulmonary infection in a pt with CF, think Pseudomona and treat it with Pipercillin + Gentamycin.

Cystinuria:
An inherited disease causing recurrent renal stone formation. Look for positive family history and stones since childhood. Stones are radiopaque and Hexagonal. The urinary cyanide nitropruside test is a screening procedure.

Cystitis - 2
Most commonly caused by Ascending infection. Pt presents with UTI symptoms and suprapubic tenderness.**** In an Uncomplicated Cystitis, where pt presents with suprapubic discomfort and signs of UTI, then there is NO need to do culture. Just give Oral Trimeta-Sulfa. If there is resistance to it then give Cipro or Nitrofurantoin.

Dacryocyctitis
Inflamatory changes in medial canthal region. Staph aureus and Strep are common causes. Acute dacryocystitis is treated by frequent application of hot compresses; cephalexin or cefazolin; and incision and drainage if an abscess has formed. Chronic dacryocystitis may be treated by dilating the nasolacrimal duct

De Quervains Tendonitis
(abductor or extensor tendons of the thumb) is usually diagnosed by localized tenderness, if not mild swelling, along the course of the tendon .Sharp pain is elicited or accentuated when the ipsilateral thumb is flexed across the palm, enclosed by the fingers, and the wrist is deviated ulnarly to stretch the tendons and surrounding sheath (Finkelstein's sign in stenosing tenosynovitis). Symptomatic relief is provided by rest or immobilization (splint or cast) of the tendon, application of heat for chronic inflammation or cold for acute inflammation (whichever benefits the patient should be used),and NSAIDs.Surgery for release of fibro-osseous tunnels.

Dehydration - 2
Mainstay of tx is IV sodium containing crystaloids (0.9% NaCl=Normal Saline)***Elderly pts are sensitive to dehydraion and even mild Hypovolemia can lead to orthostatic syncope, especially upon getting up in the morning. INCREASED BUN/Creatinin is a good indicator of dehydration.

Delirium Tremors
Tx is Chlordiazepoxide.

Delirium vs Dementia
Delirium has:Acutenes, impaired conciousness, fluctuating course, reversible symptoms and global memory impairment. In the absence of any focal neurological signs, even if there is evidence of carotic bruit (Vascular Dementia) Delirium is the most like Dx.

Delusional Grandiosity
Pt thinks she has special powers, extraordinary accomplishments, or specila relationship with God. There are three types of Delusion: 1-Grandiose (religious in nature) 2-Paranoid 3-Somatic.

Dependant personality
When ot is so agreeing and depends on whatever the doctor suggests.

Dermatitis Herpetiformis-3. Dermo. 6/3
Pruritic papules, vesicles over the knee, elbow, buttocks. Immunofloresence shows granular IgA desposits along dermal papillae. Asso w Celiac sprue. Tx is Dapsone. Suspect DH in a pt with Malabeosrtion and pruritic papules and vesicels over the extensor surfaces. Anti-Endomysial anribodies are charcteristic. Pt also suffers from Gluten sensitive enteropathy or Celiac Sprue. Tx:Strict adherence to a gluten-free diet for prolonged periods (eg, 6 to 12 mo) may control the disease in some patients, or Dapsone.

Dermatomyocytis:
Is an autoimmune disorder involving muscles and the skin. Skin eruption is dusty red in color. Edema around the eye and the helitrope rash of the eyelid are more specific. Gottron’s sign is highly suggestive of this disorder in which skin over the back of knuckles show non-scaly violacious erythomatous eruption.

DES toxicity
If given to pregnant women causesClear cell ADENOCARCINOMA of vagina in their duaghters. In the old days it was the best tx for threatened abortion. With erly dx and tx survival is 80%.

Development Dysplasia of Hip
DDH is characterized by subluxatable and dislocatable proximal femur and acetabulum. Early dx and tx is important because failure will result in sigificant morbidity. On inspection uneven gloteal fold are seen. Its mc in femlae cockasian females. Dx is confirmed by USG in infants <6mo. Positive Barlow and ortolani are highly suggestive. Tx is surgical reduction.

Developmental Milstones
1-LANGUAGE:Social smile=2mo, Bables=6mo, 2words and obey one step command=12mo, 2-3Phrase& 2 step command=2yr. 2-GROSS MOTOR: Hold head=3mo, Rolls back to front=4mo, sits unsuported=6mo, walks alone=12mo, walks staris=2y. 3-FINE MOTOR: Raking Grasp=6mo, Throw objest=12mo, Build tower of 2 block=15mo, build 6 blocks=2yr. 4-SOCIAL: Recognize parents=2mo. Recognize strangers=6mo, Imitates/comes when called=12mo, play with other kids=18mo, Pararel play=2yr.

Dextrometorphan- Poisenining 6/3
A cough medicine. has drung toxicity with MAO inhibiors, causes hyperthermia

Diagnostic Peritoneal Lavage
Is the best dx procedure for intraperitoneal organ laceration. Like a guy being hit in the stomach. Angiography is never done.

Diabetes Incipidus - 3
Presents with polyuria and polydypsia, due to ADH def or resistance. Pt prefer cold beverages and their urine osmolarity is < aldosterone="saves"> serum osm. DDX3:SIADH, results in hyponatremia, low serum osmolarity and inapropriately high urine osmolarity. sually seen in pt with lung cancer and abnormal brain pathology like trauma or stroke. DDX4:Primary polydypsia, is a disorder where pt drinks fluid in excess of 5L/day and both plasma and urine osmolarity are low (diluted).****Administration of DDAVP (desmopressin) ddx b/w CDI and NDI. Pt with CDI will have increase of urine osmolarity following admin of AVP (arginin Vaso Pressin, or DDAVP), but NDI pt wont have that increase. Tx for CDI is intranasal Desmopresin.

Diabetis Melitus - 34
1-Normal anion gap metabolic acidosis in a diabetic pt with Gastroenteritis could be either due to loss of bicarbonate due to diarrhea, or defective NH4 sunthesis due to nephropathy. So next we need to calculate urine anion gap. [Urinary (Na+K) - Urinary Cl]. If its positive value problem is Nephropahy, if its a negative value its due to Gastroenteritis. 2-Diabetic Osteomyelitis (due to arterial insufficiency) that involves bone adjasent to the foot ulcers is explained by contigous spread of infection. 3-Acanthosis nigrans is a complication of DM. Although its associated with both DM and Addison's disease insulin resistance is the mcc in young population, and its asso with malignany in older individuals. 4-Somogi effect. 5-Diabetic neuropathy tx is Gabapentin and TCA (imipramine). 6-Diabetic Cystpathy tx is Bethanechol. 7-Diabetic retinopathy. 8-Infection in diabetes. 9- Diabetic Nephropathy, detection of microalbunemia is the best detection. Fasting blodd glucose is now test of choice for screening high risk individual for DM. When fasting G is 126 or greater, repeat it, and if its still elevated the Dx is made. Dx could also be made if pt is SYMPTOMATIC and G after 75gr tolerance test its 200 or greater. Ketones responsible for DKA are Acetone, acetoacetate and beta hydroxy butyrate. Diabetic Neuropathy leads to denervation of bladder resulting in urinary retention, Overflow incontinence day and night, aside from strict glucose control tx includes intermittent cathaterizarion and Bethanechol, avoiding alcohol maybe helpful. **** Antibiotics dont cure ulcers, do a debriment of the wound. ****Glomerular Hyperfiltration is the earliest renal abnormality seen, as early as several days w/i dx of DM. Its the major pathophysiologic mechanism of glomerular injury in these pts. Thickening of the glomerular basement membrane is the first change that can be quantified. Effectiveness of ACE inhibitors is related to their ability to reduce intraglomerular hypertension and decrease glomerular damage. **** In pt with Diabetic Nephropathy add ACE inhiitor even if BP is under control, it slows progression of nephropathy and keeps glomerualar bp reduced. ****Glycosylated Hemoglobin (HbA 1-c) is the best way to monitor DM control. HbA1c is fomred by non-enzymatic glycation of Hb. Its reflective of the average glucose blood levels w/i the precedding 2-3 months, which corresponds tothe life of RBC. Every 1% increase in HbA1c correspodes to 35mg increase in glucose. Remember measurement of C-peptide is done to determine if the insulin use is internal or external. DIABETIC KETOACIDOSIS:Anion Gap Metabolic Acidosis observed during DK is accompanied by HyperKalemia, its called paradoxial because body K reserves are actually depleted. Hay hyperKalemia for 2 reasons: 1-extracellular shift of K in exchange for H with resultant intracellular K depletion. 2-Impaired insulin-dependant K entry. So in Tx for DKA, after insulin and diuresis administer K. ****Suspect it if pt presents with rapid breathing, hx of weight loss, polydipsia nd polyuria.END OF KA. ****Fasting bloog glucose measurement is now the screening of choice. A FBG of 126 or more on two occasiona is Dx. FBG b/w 100-125 is categorized as impaired FBG or pre-DM. If pt has symptoms, poluuria, polydyspsia, and obesity FBG of >200 may confirm the dx but its not appropriate for screening. The 50gr glocose tolerance test is used for screening gestational DM, while 100gr is used to confirm it.***DM Neuropathy seen in 50% of pts. Pt manifest with poplyneuopathy,mononeuropathy or Autonomic Neuropathy. AN is related to the duration of disease and glycemic control. Any part of GIT can be affected. Involvement of small intestine causes diarrhea, and Colon causes constipation, and stomach causes gastroparesis which presents as nausea, vomit, bloating, anorexia, and early satiety. Due to delayes gastric emptying, glucose control is difficult to achieve. Pt have post meal hypoglycemia after insulin injection. Nuclear Medicine Scintigraphy after ingestion of radio-labeled food is the best method to document Delayed gastric Emptying. Management includes:1-improved glycemic control, 2-small,frequent meals, 3-Dopamine agonist(Metachlopromide,domperidone) before meals, 4-Bthanechol, 5-Erythromycin (reaction with Motilin promotes emptying), 6-Cisapride. *****The most beneficial therapy to reduce progrssion of DN in presence of renal insufficiency is to control HT. *****Nonketotic Hyperosmolar synd occurs in DM2 pt because level of insulin in these pt is enough to prevent ketoacidosis but not hyperglycemia. Hyperglycemia occurs, with hyperglycosurea and dehydration. They will present with semicomatose state. So first thing you do is to check blood glucose.*****Diabetes screening in pregnant women is performed b/2 24-28 weeks of gestation. If urine dipstick reveals glycosuria then the next step is Fasting Urine samples if its positive then do a 1hr-50g oral glucose tolerant test. If its <140,>140, 3hr-100gr OGTT is used for confimation.****Diabetic Neuropathy can present with ulcer in the foot. Risk factors for development of diabetic foot ulcer are: Diabetic Neuropathy, peripheral vascular dis, poor glycemic control, bony abnormalities of the foot, male sex, smoking, chronic DM (>10ys), and a hx of previous ulcer or amputation. Neuropathy is found in 80% of diabetics with foot ulcer.****Symetrical distal sensorimotor polyneuropathy is the mc type of diabetic neuropathy. Characterized by "stocking glove" pattern or sensory loss. ****DM is a risk factor for Non alcoholic fatty stetosis.*****Hyperglycemic, hyperosmolar, non-ketioc coma is characterized by very high blood glucose, plasma hyperosmolarity, normal aion gap and negative serum ketones. DDXKA is sugested by 1-blood glucose >250, 2-pH<7.3, 3-Bicarbonate<15-20,4-Plasma ketones.DDX2:Alcoholic Ketoacidosis is ketoacidosis with increased anion gap BUT near normal glucose levels. ****Always consider candida albicans as a casue of infection in a pt with uncontrolled DM.****Diabetic mothers babies are often born with clavicle fracture that heals spontaneouly w/o any tx.***The Dx procedure of choice for Diabetic Polyneuropathy is Electromyography and conduction studies. ****Poorly controled pt with low grade fever,bloody nasal discharge,nasal congestion,involvement of the eye and chemosis ,proptosis and diplopia is more likely suffering from Mucor Mycosis and maxillary sinus due to Rhizopus. Involved turbinates usually become necrotic. DDX is Pseudomona which causes Malignant Otitis Externa, it my also cause black necrotic lesions. DDX also H.inf and Moraxella are the mcc of bacteria sinusitis, they dont cause NECROTIC infections*****Non-ketoic Hyperosmolar coma presents with gllucose 1000, and normal pH. Tx is normal saline initially and then replaces with 0.45% saline. Once glucose is down to 250, then we give 5% dextrose that prevents cerebral edema. **** Diabetic Cystopathy usually secondary to diabetic autonomic neuropathy. It begins with inability to sense s full bladder & failure to void completely. With time bladder size increase leading to signs of BPH & recurrent UTI. Dx is made with Cystometry and Urodynamic studies. Initial mgmnt is strict voluntary urinary scheduling couplded with Bethanechol. If there is no response intermittent catheterization is recommended. ****Emphysematous Pyelonephritis: is a life threatening condition caused by E. Coli. Dx is confirmed by CT. TX is IV antibiotics and possible Emergency Nephrectomy.

Diamond Blackfan Anemia:
Also called “congenital hypoplastic anemiaâ€
. Suspect it in a child with macrocytic anemia, low reticulocyte count and congenital anomalies. Primary path is an intrinsic defect of erythroid progenitor cells which results in increased apaptosis. Over 90% are dx w/I the first year of life. Macrocytic anemia is distinct from megaloblastic anemia because hay no hypersegmentation of the nucleus in neutrophils. Pt presents with anemia, short stature, webbed neck, shielded chest, triphalangial thumbs. Tx is mainly corticosteriods, if unresponsive then transfusion therapy.

Diaphragmatic Herniation
Occurs in accidents and Cxr sign of elevated left diaphragm could be the only sign.*****One dx often missed in er is traumatic rupture of the diaphragm. Usually on the left side. Pt comes back months later with breathing difficulty. Cxr shows deviated mediastinum with a mass in the left lower chest. Barium Swallow is dx. In acute cases surgery is done via abdomen and in chronic cases via chest.

Diarhhea - 6
Campylobacer Jejuni is the mcc of bloody diarrhea in US. Its from undercoocked pultery****Vibria Parahemolyticus:by ingestion of sea food. Bloody diarrhea,abdomnal cramps,nasea and feve. Incubation 12-24hrs. Shigela diarrhea occurs in day care and institutional settings. Yersinia diarrhea is by eating undercooked pork. Campylobacter is the mcc of diarrhea in US due to uncooked infected poultry. could be watery or hemorrhagic. ***Staph causes toxin induced gastroenteritis mostly emetic type that starts w/i 6 hours.Salad,meat and egg.***Travelers diarrhea, due to E Coli, is the cause of diarrhea w/i blood,mucus,explosive,rice watery diarrhea even in Mexico travelers. DDX Giardia is endemic in Nepal. ***MCC of diarrhea in children is Roto virus that causes acute gastroenteritis.Most causes are self limiting but maintain hydration Ther is now a vacine for it but it was withdrawn due to risk of Intussusseption*****Types of diarrhea are: 1-Inflamatory, where ESR is elevated and there is anemia and blood positive stool. 2-Osmotic, caused by meds or hormonal disturbance. 3-Motor, exemplified by Hyperthyroidism. 4-Factitial, is psychologic.

DIC:
Tx is FFP if pt is bleeding only. If pt is not bleeding and has sepsis (high T and low BP) first step is IV antibiotics plus Activated Protein C.

Diffuse Esophageal Spasm - 2
Manifest with chest pain and dysphagia. Etiology in unknown but its related to emotional stress. Unlike Achalasia LES has a normal relaxation response. Esophagogram might show Corkscrew. Tx is with antispasmic drugs, dietry modulation and psychiatric counselling. for USMLE know 1-pathophys, 2-present or absence of perstalsis, 3-LES tone. MERCK:A generalized neurogenic disorder of esophageal motility in which phasic nonpropulsive contractions replace normal peristalsis and, in some cases, lower esophageal sphincter malfunctions occur. Esophageal manometry shows: contractions are usually simultaneous, prolonged or multiphasic, and possibly of very high amplitude.*****Esophagography may not show the corkscrew, so do Manumetry, if revealed "repetitive,nonpeistoltis,high amplitude contraction either spontaneoud or after Ergonovin stimulation then its Dx.

Di-George synd:
Infants have cyanotic heart dis, cranofacial anomalies, thymic dysplasia, cognitive impairment and hypoparathyroidism. Asso with Ch 22/11 deletion. In Surgery keep an eye on Ca levels.

Digitalis Toxicity - 2
Some of the toxicities occur in therapeutic range (AV block, ST depression, T inversion) and there is no need to discountinue the drug. Some occur in Toxic serum levels (Atrial Tachycardia and AV heart block) and we need to discountinue the drug. Digoxin also causes Nasea and Anorexia.

Diphenhydramine Toxicity
Prduces seizure as well as anti-cholinergic effects.

Dipyridamole
used during myocardial perfusion scaning to reveal areas of restricted myocardial perfussion. It shows "Coronary steal phenom"

Disk Herniation
Once you know its DH and straight leg is positive, then NSAID and early mobilization s the tx of choice.

Displacement - 2
An immature defense mechanism, in which individual displaces negative feelings asso with unacceptable situation onto a safer one.

Disseminated GonoCoc infx
Persents in menturating women with tampon, many partners, occasional condom, presents with high fever, rash, tenosynovitis and migratory arthralgia. DDX with TSS which presents with Fever, macular erythema of palms and soles,, vomit and diarrhea nad hypotension.

Dissociative Fugue
Pt get lost in another city.

Diverticulitis - 2
the dx test to evaluate the abdomen during an acute episode of diverticulitis is a CT scan. Colonoscopy and Sigmoidoscopy can cause perforation.*****Acute diverticulitis complication may be bowel perforation where xray shows air under diaphragm and rigidity and guarding. Next step is Laparotomy with surgical resection of perforated bowel and proximal colostomy.

Diverticulosis
Pseudodiverticulum can erod a penetratinf atrery. This leads to perfuse arterial bleeding of bright red blood. Diverticulosis is the mcc of bleeding in elderly pt. Chronic constipation is the single most predisposing factor to develop Diverticulosis. Normal xray does not rule out diverticulosis if its negative. You need xray with contrast ( Barium ) to be able ro see it. DDX1: Colon Cancer, presetns with chronic,occult bleeding NOT BRIGHT RED. DDX2:Ischemic Colitis, Asso with Abdominla pain, feverand vomit and atherosclerosis, xray shows thickening of colon wall. DDX3:Mesenteric Thrombosis, Pain out of proportion is a classical symptom, Bloddy diarrhea rather than bright red blood is charcteristic, Bowel sounds are diminished.

Down Synd in Pregnancy
T14Q12 explains how to test for DS in older women. Decreased MSAFP and Estriol and Increased B-HCG is the best test. SEE FIRASR AID.. Know heart defect, and also that they have Duedenal atresia. Learn this.***** Hay ASD and endocardial cushin defects. *****Duodenal Atresia is the mc anomaly asso with Down, in xray you see a double bubble sign. Other anomalies are Hirshsprung, Esophageal atresia, Pyloric stenosis, malrotation. Congenital heart disease is the mcc of death in childhood, like endocardial cushin defect, VSD, PDA.

Dressler Syndrome:
It’s a post MI Pericarditis. Non specific ST elevation. NSAID is tx of choice.

Drug induced Pancreatitis
1-Diuretics, furesamide and thiazide. 2-IBD, Sulphasalazine and 5-ASA. 3-Immunosupresants, azathioprine. 4-Seizures, Valprioc acid. 5-AIDS, Didanosine, Pentamidine. 6-Antibiotics, Metronidaole, tetracycline. CT is dx with inflamed pancrease. Tx supportive.

Drug induces Interest Nephritis - 2
Caused by Cephalosporins, Penicillins, Sulfa drugs, NSAID, Rifampin, Phenytoin and Allopurinole. Pt presents with Acute renal failure+Arthralgia+rash.*** 70% of cases are induced by drugs, discountinue the drug and it will be OK. Pt present with fever, and urine analysis shows RBC, WBC and white cell casts, eosinophelia and proteinuria.

Drugs CI
1-Beta blockers: peripheral vascular dis (pt presents with worsening intermittant claudication), asthma, copd, Raynaus. 2-ACE inhibitors: Hyperkalemia, pregnancy. 3-Calcium channel blockers:Second&thrid degree heart block and CHF.

Dubin-Johnson
A familial disorder of hepatic bile secretion. Leads to conjugated Hyperbilirubinemia. May be aggrevated by women taking OCP. Liver biopsy reveals cells with DARK granular pigments. DDX1:Rotor, like DJ but no DARK granule pigments.

Duchenes MD
Muscle biopsy will reveal dx.

Dumping Synd
Is a common post-gasterectomy complication. Pt with recent Gasterectomy presents with postprandial abdominal cramps, lightheadedness, diaphoresis. First thing to do is to Modify diet, small frequent meals and avoid simple CHO. Dx is made clinically but occasionaly Contrast xray (barium swallow) is used.

Duedenal Atresia:
Bilous vomiyying few hous after the FIRST eating, usually asso with congenital anomalites and Down’s Synd.

Duodenal Hematoma, isolated
If pt is hemodynamically stable, she needs nasogastric succion and parenteral nutrition (food ) not IV fluid.

Duedenal Injury:
Isolated duodenal injury is easily missed. They occurin accidents due to the belt or steering wheel. Present with epigastric or RUQ pain. Retroperitoneal air or obliteration of right psoas margin on xray is very suggestive. Best dx with CT scan of the abdomen with oral contrast or an upper GI study with gastrograffin, followed by barium if necessary.

DVT - 3
OCP is a well known risk factor for DVT. TX steps:Anticoag therapy has serious Ses so accurate dx must be made before anticoag is started. Test of choice for DVT is Compression US. Impedence Plethysmography is for recurrent DVT. Venography is the Gold standard for dx of DVT, but it causes discomfort so its not the initial test for suspected DVT. Its only done when other tests are impossible or inclusive.***Surgical pts can be categorized according to their risk of DVT. 1-Low risk, Minor surgery in a pt <40>40, one or more addiiotnal risk, minor/non-minor surgery, risk is 2-10%. 3-Hihg risk, pt is >40, additional risk factors, major operation, risk of DVT is 10-20%. In Low risk pt, prophylaxis other than early mobilization is not recommended. In Moderate risk pt, LMWHeparin or Unfractionted Heparin is recommended. Pts in whom bleeding risk are unacceptable (intracranial.spinal cord injury) should receive intermittant pneumatic compression. In High risk pt, undergoing general surgery can be given LMWH, those pts going under Orthopedic surgery of lower extremity (knee replacemnt) LMWH or Oral Warfarin.***INR (International Normalized ratio) is used to monitor tx response to Warfarin. Therapeutic Range of INR for most pts is 2-3, which is for venous thromboembolism, valvular heart dis. 3-4.5 is for Proshtetic valves.*****The besr DVT prophylaxis for high risk surgery ptgoing under orthopedic surgery includes either warfarin or LMWH.

Dysfuncx Uterine Bleeding- 2
Heavy unremiting endometrial hemorhage throught menarche and perimenopause requires Estrogen (conjugated) to supress the bleeding to ensure CV stability. Once that is achieved D&C should be performed. The MCC of DUB in adulescent is anovulation. Therefore endometrial biopsy is not required in these pts. Once bleeding is stopped , advise pt to take the following: conjugated estrogen for 25 days , then add methoxyprogestrone for the last 10-15 days and then allow 5-7 days for withdrawl bleeding to mimic menstural cycle.****In which pts with CUB do you perform endometrail biopsy to rule out endometrial carcinoma? When a pt is >35, obese, DM or has chronic HT.

Dysthymia
Depressed mode for more than twoyears.

Dystonia
from antipsychotics, tx is Benztopine or Diphenhydramine.

Eaten Lambert
Is asso with small cell carcinoma. And antibodies against the voltage gated calcium channels in presynaptic motor nerve terminal. It is presynaptic, resulting from impaired release of acetylcholine from nerve terminals.The diagnosis is confirmed by finding an incremental response to repetitive nerve stimulation: Amplitude of the compound muscle action potential increases > 200% at rates > 10 Hz. Treatment is first directed at the underlying malignancy and sometimes induces remission. Guanidine facilitates acetylcholine release.

Eating disorder not otherwise sp
If the senario shares features of both bulemia and anorexia, its this disorder.

Echinococcus
Due to close cntct with SHEEP. Pt presents with hepatomegally, forms hydatid cyst in liver after US. Hydatid cyst has an inner germinal layer and an outer acellular laminated membrane. DDX is Neurocysticercosis, due to PIG farming. With cysts in Brain, kills fast.

Eczema Herpeticum
T9Q23. A form of Herpes simplex that is asso with atopic dermatitis. Numeric umbilicated vesicles around The healing area is typical. In infants tis could be life threating, start acyclovir asap.

Edward synd - 2
Microcephaly, prominent occiput, micrognathia, closed fists, index finger overlaping 3-4-5 bilaterllay, rocker bottom feet. 95% die by first year.*** Pts have hear mumur due to VSD. This is trisomy-18 (E-lection age)

EKG abnormalities
1-T wave inversion, in ischemia of myocardium. 2-ST depression, subendocardial infarcts and unstable angina. 3-Ptoonged PR, first degree heart block. 4-Delta waves, WPW. 5-New RBBB, seen in PE. 6-Electrical Alterns, seen in pericardial Temponade.

Embolus, limb
If pt presents with cold hand due to embolus, immediate antocoag with heparin and surgcal intervention is indicated.

Emphysema
In a non smoker should raise the suspicion to Alpha-1 anti trypsin def. Its also asso with Neonatal Jaundice in the hx of the pt. Dx is made by estimating alpha-1 trypsin levels.

Emphysematous Cholecystitis:
Is a common form of acute Cholecystitis in elderly diabetic males. It arises due to infection of the gallbladder wall with gas forming bacteria. Dx is confirmed with abdominal xray showing air-fluid level in the gall bladder or US showing gas line. Lab shows moderate unconjugated hyperbillirubinemia or small elevation of aminotransferases. Tx includes early fluid resuscitation, early cholecystectomy, and parenteral antibiotic therapy effective aginast gram positive Anaerobic Clostridium sp. (Ampicillin- Sulbactam, or combination of aminoglycoside or quinolone with clindamycin or metronidazole.

Empyema
Can occur from parapneumonic effusions(In parapneumonic effusions, the visceral pleura overlying a pneumonia becomes inflamed; often, an outpouring of serous exudative fluid accompanies acute pleurisy. The fluid contains many neutrophils and may contain bacteria. Parapneumonic effusions are usually caused by bacteria. If the body's defenses do not control infection in a patient with pneumonia and parapneumonic effusion, the number of neutrophils and bacteria increases, and the fluid takes on the gross appearance of pus. The result is empyema of the thorax (purulent exudate in the pleural space). Fluids with > 100,000 neutrophils/µL, bacteria seen on Gram stain, and pH < 7.2 may be presumed to be empyema). Pt presents with low gade fever, dyspnea and chest pain. Dx with CT. When its localized, complex and has a thick rim best tx is surgery to remove the clotted blood.

Endometria hyperplasia
A28. Printed out

Endometriosis - 4
Pt presnts with painful periods. Bimanual exam shown a few firm nodularities in pouch of douglas.The first line of tx is OCP. They cause a state of pseudopregnancy and causing an "exhaustion atrohpy" of the endometriomas. If OCP fails or not tolerated then we give Danazole, its an androgen deivative that causes Pseudomenopause state. SE are acne,hirsutism deep voice.GnRh agonists have an inhibitory action of LH & FSHwhen given continuslywhich produces temporary castration. Its also a 2nd line of choice. ****Typically pt prestns with Dysmenorrhea, Dysparunea(when endometriomas is in cule-de-sac), Dyschezia(Pain on defecation), hematochezia, hematuira, and pre post menstrual spotting. Laparoscopy is GOLD standard which shows powder burns. The hemorrhage of endometriomas into the ovaries results in formation of cystic cavity filled with blood with dark color, hence the name 'Chocolate cyst'. ****The '3Ds' are Dyspareunia, Dysmenorrhea and Dyschezia (defecation pain). Tx is OCP. DDX1:Vaginismus, use Vaginal dialators. DDX2:Pain disorder, pain in one or more anatomical sitetxis pain managment training. DDX3:Somatization disorder, tx is follow up visitsregularly scheduled. *****Endometriosis is the location of tissue outside uterine cavity so hysterosalpingogram cant see it.

Endometritis:
It usually occurs on 2nd-3rd day postpartum. Predisposing risk factors are prolonged labor, prolonged and premature rupture of the membrane, manual removal of placenta, and repeated pelvic exam. Clinically it presents with fever, uterine tenderness and foul smelling luchia. Antibiotic start asap to conver both aerobic and anaerobic. Clindamycin with aminoglycoside or ampicillin. MC pathogen is anaerobics.

End stage lung dis:
PFT is the best test to to determine if the pt can benefit from lung resection surgery. Predicted postoperative FEV1 is very useful for this. Blood tests don’t reveal any good info in this regard, they’re more useful in determining the level of respiratory compromise and appropriate ventilator settings for pts undergoing lung resection surgery. Results of split function quantitative lung scans and exercise testing are useful in pts in whom the potential benefit is doubtful even after determining the results of the predicted postoperative FEV1.

Enterobius Vermicularis
Or Pin worm is the MC helminthic infection in US. Most commonly seeb in school children 5-10. Larva goes to perineal area to lay eggs, which gives characteristic Nocturnal perianal pruritis. Dx is made by "Scotch tape test". Albendazole or Mebendazol is the first line of tx. Pyrantel Palmate is an alternative.

Enuresis - 2
Tx is low doses of Imipraimne or desmopressin.***It should go away begore school age. The first step is reassurance. Then wet alarms and walking the child to bath room is tried. If persiss, then Desmopressin (ADH) is first line, Imipramine is the 2ns line tx. ****Its important to rule out treatable causes like UTI. The initial evaluation is urine analysis.

Epididymitis, Acute

Epidemiology:
Mean is average, Mode is the number repeated mostly, Median is the number in the middle given by vigniette, don’t put them in order treat it as is. Reliable test is one that gives similar results repeatedly. Accurate is when the results are on the target.**** Sensitivity curve movements and its effect on PPV.

Epiglotitis - 3
MC by H. inf and 2nd mc by Strep. Tx is antibiotics, antipyretic, racemic epinephrine, steriods and immediate intubation. Dx is by Fiberoptic Laryngoscopy in the operating room, once its made then nasotracheal intubation secures the airway. If intubation is CI then Emergency Tracheostomy is performd.. *****you dont need epinephine before intubation, intubation is the first thing to do here.

Erectile Dysfunction
Types are 1-Neurogenic:A pelvic fracture with an urethral injury is usually accompanied with ED. The cause is nerve injury and altered blood supply. 2-Venogenic:After penile fracture and disruption of tunica albuginea. 3-Endocrinologic:Prolactinoma. 4-SystemicM can cause Ed through many systems (neuro, vascular). 5-Situational:Psychogenic, where night and morning eection is preserved.****If pt is taking Nitrate drugs, Sildenafil is CI so the next step is Penile prosthesis devices or Vaccum devices.

Erysipelas
Is a specific type of cellulitis in which there is superficial inflamation of epidermis producing prominent swelling. The characteristic finding is a sharply demarcaded , erythomatous, edematous tender skin lesion with raised borders. Onset is abrupt with systemic signs. Group A strep is the mcc. Penicillin V or erythromycin 500 mg po qid should be given for >= 2 wk. See pic on desktop.

Erythema Multiform. Dermo. 6/3
Onset is usually sudden, with erythematous macules, papules, wheals, vesicles, and sometimes bullae appearing mainly on the distal portion of the extremities (palms, soles) and on the face. The skin lesions (target or iris lesions) are symmetric in distribution and often annular. Stevens-Johnson syndrome is a severe form. EM Usually folows infection with Herpes Simplex. Erythema multiforme associated with mycoplasmal pneumonia should be treated with tetracycline. If frequent or severe erythema multiforme is preceded by herpes simplex, acyclovir

Erythema Nodosum
Pink to redish painful subcuataneous nodules that usually develop in pretibial region. Most often in woman 15-40. Lesion resolve w/o scaring w/I 2-6 weeks. Histologically there is paniculitis involving inflamation of septa in the subcutaneous fat tissue. There are other conditions that could cause EN, like TB and Sarcoidosis. So the initial work up is include Antistreptolysin O (ASO) titer, a TB test and chest CXr. Its also asso with IBD. Sarcoidosis pt often presents with EN as an initial symptom, cxr will show bilateral hilar adenopathy. An inflammatory disease of the deep dermis and subcutaneous fat (panniculitis) characterized by tender red nodules, predominantly in the pretibial region but occasionally involving the arms or other areas. Bed rest helps to relieve painful nodules. If an underlying streptococcal infection is suspected, antibiotic therapy is beneficial (eg, penicillin for >= 1 yr).

Erythema Toxicum
Is a benign self imited condition in newbors characterized by rash with red haloes, and eosinophils in sin lesions. Neonate presents with No fever, no infectious risk factoe, looks healthy, with erythematous papules and vesicles surrounded by pathes of erythema.

Esophageal Atresia:
It’s the mc esophageal anomaly w esophagotracheal fistula. It leads to gastric distention. It results in drooling and regurgitation due to incomplete esophagus. In addition food gets into trachea and lungs and cuses aspiration pneumonia. Inability to pass tube is suggestive.

Esophageal cancer - 2
It mimics Achalasia. Short hx , rapid weight loss, and inability to pass esophaguscope isindicative of cancer. The next step is biopsy. Ofcourse BS followed by endoscopy should be done first.

Esophageal Varices, Acute - 2
Variceal bleeding is a life threatning emergency. FIRST step is fluid replacement with two large bore IV needles followed by fluid resucitation. SECOND step is control of bleeding medically with vasoconstrictors (Octreotide, somatostatin) THIRd step is Endoscopic Sclerotherapy or Band Ligation (which is better due to less SE). If endoscopic therapy is not available then Baloon temponade with S-B tube is done. If all this fails then surgery is indicated (TIPS). ****In case of EV, need for 5 or more units os blood transfusion in a period of 24 hours is considered an indication for surgery and Transjugular Intrahepatic Portosystemic Shunt. Remember both ligation andmeso-caval shunt have high mortality rate in ER setting, TIPS has less mortality rate.*****Varices are submucosal veins dilated due to portal HT.

Estrogen Replacement Therapy:
Affects metabolism of thyroid hormones. The requirement for L-Thyroxine increases, although the exact mechanism not known it could be due to induction of liver enzymes, increased level of TBG. In pregnancy also thyroid hormon requirement will be increased and the pt should be monitored for dose adjustment.


Essential Tremors - 3
DOC is Beta blockers.***Another drug is Primidone, Its SE is Acute Intermittent Porphyria (Abdominal pain, neurologic and psychologic abnormalities, it can be dx by urine prophobillinogen. ****Propranalol is the DOC for pts with benign essential tremors + HT.****Its famililal, its worse with action and resolved at rest. Rule out Thyroid problem before starting therapy.

Ethyline alcohol poisening
Presents with anion gap metabolic acidosis with Rectangular envlope shaped crystals (calcium oxalat).

Eustachian tube dysfunction
Is a common cause of conducting hearing loss in children. Aurul fulness, pop when swallow, hearing loss, intermittant ear pain. Usually following URT infection or allergic rhinitis. Retraction and decreased mobility of tympanic membrane. Hallmark is a middle ear effusion. "Acute Otitis media": Otalgia, hearing loss, fever and dysequilibrium, bulging membrane. "Serous otitis media": Due to prolonged blockage of auditory tube, common in children, membrane is hypomobile and dull, air bubbles in the middle ear. "Otitis externa": Purulent dischatge, common in swimmers, pain with tenderness is the hallmark. "Foreign body in children": Foul-smelling discharge and signs of infection.

Ewing sarcoma
Highly malignant tumor of lower exremities in children. With early metastasis. Presentation is pain and swelling for weeks. Oftern confused with Osteomyelitis due to intermittent fever, leukocytosis, anemia, elevated ESR. CXr shows "ONION SKINING" peroosteal retraction. Lesion is Lytic and central. Onion skin is followedby 'moth eaten' appearance. Tx includes surgery , radiation, and multiple drug chemo. DDX is Osteomyelitis:Pt presnts with feve, malaise, local pain in joints and swelling. Xray in chronic osteomyelitis shows Lytic bone defect with surounding sclerosis termed as "Brodie's Abscess".

ExtraPyramidal Synd (EPS)
Is seen as SE of antipsychotics (Risperidone). 1-Tardive dyskinesia, lip smacking, tongue protrusions, chewing,biting. It occurs b/w 4mo-4ys. Tx is discontinue Risperidone and give Clozapine. 2-Akathesia is the feeling of restlessness, beta blocker gives some releif. 3-Dystonia, occurs b/w 4h-4d, there is muscle spasm, stiffness, twisting, opisthotonus. Antihistamine (diphenhydramine) or Anticholinergic (Benztropine) releif.

Factitious diarrhea
Laxative abuse, profuse diarrhea. 10-20 times a day. DDX with IBS diarrhea is that IBS does not happen nocturnaly but factitious does. FD is usually done by women of high socioeconomic status and Nurses. There is also characteristic dark brown discoloration of the colon with lymph folicles shining through as pale patches that confirms the dx.

False Labor
Characterized by painless and irregular contractions for 5hrs or more. In the last month these contractino may become rhythmic occuring every 10-20 minutes mimicking contraction of real labor. The main characteristic is however they are not accompanied with progressive cervical changes, so cervix is closed shut. All the pt needs is reassurance.

Familial Colonic Polyposis
Pt hace 100% risk of colon caner, so when they are dx (colonoscopy reveals 100s polyps), then then next step is elective proctocolectomy.

Fanconi’s Anemia:
An auto recessive dis, progressive pancytopenia and macrocytosis. Deformities include, café au lait spots, microcephaly, micropthalmia, short staure, horseshoe kidneys and ABSENT thumb. Dx agerage age is 8 YO

Fantacy Defense Mechanism
An immature defense mechanism, that does not exist in the real world, like an angel telling you things are gonna be OK.

Fat embolism
Dyspnea, confusion and petechia in the upper part of the body, After multiple fractures of long bones. Tx should include prompt respiratory support. Use of heparin, steriods is controversial.

Fat Necrosis
Biopsy shows foamy macrophages and fat globules. Coarse calcification is indicative of benign, and microcalcification indication of malignant tumors. FN is asso with hx of surgery or trauma and it mimics breast cancer.Exisional biopsy is dx and no tx is needed and standard follow up and mamogram is sufficient.

Febrile Neuropenia - 2
A neutropenic pt with sustained fever of >100.4 for one hour. Its an emergency, admit to hospital and obtain blood cultures and IV Cefepime, or Ceftazidime, or Imipenem. Vancomycin is added IF pt is hypotensive, evidence of skin or line infection, Hx of resistant to S. Aureus or pneumococcus, or recent prophylaxis with flouroquinolones.

Femor Shaft fractures
Most can be managed with closed intra medulary fixation of the shaft. This allows for early mobilization improved hip and knee function and less hospital cost. I this technique, closed reduction of fracture segment is followed by inrta medullary nail insertion through small skin insertion over the greater trochaner.Closed nailing is preferedover OPEN nailing due to reduce risk of infection. Internal fixation with plates and screws are used in NECK fractures of femur.*****Interochanteric fracture of the femur is mostly an extracapsular fracture in elderly. Occurs along the line b/w greater and lesser trochanter, Extremity is shortended and internally or externally rotated. Dx is xray. Operative tx is indicared asa pt is stabilized. Do internal fixation with sliding screw with plate and early mobilization.

Femoral Neck Fractures
They are seen in Elderly. The limb is shortended and rotated and painful with limited motion. Unstable fractures (complete neck fractures), need Open reductionb and internal fxation or Primary Athroplasty(surgical reconstruction of the joint) as soon as pt is stabalized. If surgery is CI the pt should be mobilized asap and eventual malunion can be dealt with later.

Fetal Alcohol Synd - 2
Irritability, mild to moderate mental retardation, hpoplastic maxilla, lng philtrum, thin upper lip border and microcephaly, and epicanthal folds.

Fetal Demise – 6/24/06
if pt comes to you due to not feeling any mvemnt and you cant hear any beats with Doppler, then the first thing to do is to do a Real Time Sonogram.. It’s the most appropriate test to confirm fetal demise. BhCG might be lower but it doesn’t make it a dx tool.

Fetal Distress (Repetative Late Decelerations
Is an indication for C-section. Remember Tocolysis means not delivering.

Fetal Hydantoin Synd. - 2
Caused by using Phenytoin in pregnancy due to seizure. Infants presents with small size, microcephaly, hypoplasia of distal phalanx of fingers and toes, nail hypoplasia, low nasal bridge, cleft rib and rib abnormality and cardiac mmurmur. Its also asso with Neuroblastoma. Karytype and TORCH should be measured.***Diphenylhydantoin (phenytoin) is metabolized to Epoxide Metabolite, which is eliminated inturn by enzyme Epoxide Hydroxylase. The genetic expression of EH is different from one subject to other and its substrate EMis the agent incriminated in the syndrome.

Fever, Post operative:
Fever after the first day is due to Atelectasis. Pt might have pain at the incision and not take deep breaths causing atelectasis. Fever 3-5 days post surgery is due to UTI. Fever 3-7 days post surgery may be due to Pulmonary Embolism. Pt is Tachypnea,tachycardia and Hypoxia. Dx requires Duplex US to look for clots in extremities. Eventhough pt are given anticoags, 200,000/y die of PE. Fever one to two months post durgery is due to post spleenectomy sepsis. All pt are given Pneumococcal vaccine after surgery to prevent this. Fever due to Pneumonia can occur 3-6 days post surgery. Pts are most likely, smokers,obese, elderly and fail to ambulate. Pt will have sputum and leukocytosis. Fever due to wound infection is 4-7 days post surgery. Redness, pain and induration.

Fibroadenoma:
1x1 cm firm rubbery freely mobile round mass in a 35 yo women w/o axillary nodes palpable. Best initial step is Mamogram.

Fibrocystic Dis of Breast
A 4x5x6 cm moveable rubbery mass that will go away after poking the needle and secretion of clear discharges. The best approach after aspiration of fuid is to wait 4-6 weks . In FCD the mass goes away and doesn’t come back. It it recure or doesn’t go away, then a biopsy is indicated. If the fluid (initially) was bloody or foul smelling, cytology is needed at that stage.

Fibroid uterus
Presents with Dysmenorrhea, heavy menses, and enlarge uterus is almost dx of either Adenomyosis or FU. Submucosal fibroids often imterefre with rmbryonal implantation and infertility. Fibroids are the mc benign uterin tmors in women and the mc indication for hysterectmy. Tey are estrogen-dependent tumors, therefore they increase in csize with OCP and pregnancy. and often regress after menopause. DDX is Endometriosis which presents with Amenorrhea. Make sure you can DDx the above conditions with Adenomyosis.

Fibromuscular Dysplasia - 2
Is the mcc of 2ary HT in childern. It responsible for 20% of the cases of renal HT. Its also seen in premenopausal women. PE shows a hum or bruit (soft to-and-fro bruit) in the right costovertebral angle due to well developed collaterals. The right renal artery is more affected than the left. Angiography shows 'string of beads ' pattern in the renal artery****Pt presents with Occipital HA, HT and renal bruit, suggestive of renovasculat HT due to Renal Artery Stenosis. The usual cause in youner pt (30) is FMD. In older pt its atheroslcerotic plaques. Goal of tx is decrease BP and restore perfusion to kidneys. Interventanl therapy is better than medical mgmnt alone, so Angioplasty with stent replacemnt is best tx. If it fails then Surgery is indicated. Ace inhibitors are reserved for Elderly pts who are not good candidate for surgery. Remmber Ace inbitors are CI in bilateral renal stenosis.

Fibromyalgia- 4. Rheumo 6/3
Occurs mainly in females and may be induced or intensified by physical or mental stress, poor sleep, trauma, or exposure to dampness or cold .Pt presents with diffuse musculoskeletal pain, multiple tender points, with no joint swelling/pain. Initial work up is 1-CBC, 2-ESR, 3-TFT, and 4-CK enzyme. Normal spine movement makes "Ankylosing Spodylitis" unlikely. Normal ESR with pain rather than stiffness makes "Polymyalgia Rheumatica" unlikely. "Polymyocytis", usually presents with weakness rather than diffuse pain and increased CK. *Characterized by muskuloskeletal PAIN and presents of 11-18 tender points. Pt prsents with recurrent HA, IBS, Reynauds, The most importnat ddx of this diseases is masked depression and somatoform disorder. Tx is excercise & Antidepresssants.****DDX1:Chronic Fatigue Synd: pt prestns with extremem fatigue and not body aches. It must be there for 6mo. DDX2:Polymyalgia Rehumatica:Pain and stiffness of shoulder and pelvic. Very unlikely in pt <50yo. ESR is elevated. Complain of stiffness rather than weakness or pain. Asso with fever,weight loss and HA. DD3:Polymyositis:Proximal weakness of muscle in upper and lower extremity. Pt complains of difficulty raising from chair position or climbing stairs. No pain just weakness***** Fibromyalgia is not an inflmatory disease so NSAISs like Naproxen & steroids are not helpful. Tricyclic antidepressant like Amitriptine are tx at bedtime. For daytime pain use acetaminophen. You can also use Cyclobenzaprine. So the initial tx is either Amitriptalin or cyclobenzaprine. If refractory to the above medicine then SSRI are added. When pt feels better then exercise is initialed. Other refractory tx is trigger point injections

First degree heart block


Fluphenazine SE:
Hypothermia by causing vasodialation and inhibition of shivering.

Focal segmental GN
Asso with HIV.

Fragil X syndrome - 2
Pt can be tought to take care of himself with and perfor simple task with close observation, like down's. *** Low to normal IQ, with learning disability. , general language disability, short attention span, autism, Mutation of FMR-1 gene caused by increased number of CGG trinucleotide repearts. Large head, prominent jaw and large low set ears.

Friedrisk Ataxia
Auto rec. excessive number of trinucleotide repeats resulting in abnormalilty of topopheral transfer protein. Poor ptognosis. Tell the parent to seek genetic counseling for future pergnancy. MRI of the brain and spinal cord shows marked atrophy of cervical spinal cord and minimal cereberal atrophy. *****Auto rec dis. Begins before 22 years of age. Neurological manifesration gait ataxia, falling, dysarthria) result from degeneration of spinal tracts (spinocerebellar, posterior tract, pyramidal tracts). Non neuro symptoms include concentric HCM, DM and skeletal abnormality (scoliosis annd Hammer toes). Median survivial is 20 years. Mcc of death is CV, 90%.

Frost bite injury
Warm up the body with warm water is the tx.


Frozen Shoulder
There is stiffnes and limited motion due to glenohumeral joint stifness. In PE range of motion in all directions is limited whether passive or active movement. This is as a result of Pericapsulitis. Majority are idiopathic. Arthroscopy establishes the dx by showing decreased joint space volume and loss of normal axillary pouch. Tx involves NSAIDa, steriods injection in the joint and physical therapy. Rotator Cuff Tear or Rotator Cuff Tendinitis presents with severe pain and weakness of shoulder abduction. Positionng arm above shoulder aggrevates the pain. Range of motion is only limited in active movement but is normal in passive flextion. A positive drop arm sign, with inability to actively maintain 90 degree of passive abduction maybe present in large tears. Tendinitis can be ddx from Tear by injecting Lidocain that would result in improvment in motion in Tendinitis but not in Tear.

Furosemide Tox:
Causes oto-toxicity. Aminoglycosides, vancomycin, quinine, and chloroquin also cause oto-tox. Aspirin causes Tinnitus.

Galactosemia
In a newborn or young infant with failure to thrive, bilateral cataracts, jaundice and hypoglycemia. Early dx and tx by removing galactose from diet are mandatory. It’s a metabolic disrorder causd by Galactose-1Phosphate Uridyl Transferace def. That leads to increse level of Galactose. DDX: galactokinase def, only have cataratct and otherwise asymptomatic.

Gallblader Carcinoma
It’s a rare tumor, found in pts with chronic Choledocholithiasis usually diagnosed intra or post operatively after cholecystectomy.*****Chronic Cholecystitis predisposes to carcinoma, in xray we can see Porceline gall bladder, due to calcium deposition in gall bladder.

Gastric Carcinoma
Pt might present with blood vomiting and Acanthosis Nigrans. Don’t be fooled into thinking that its Aspirin tox, becaseu aspirin tox doesn’t cause AN. AN is a diease that causes dark,thick areas on the skin. MC in arm pits and other folds. Its associated with being over weight and other tumors like GASTRIC carcinoma. so pt needs complete work up like endoscopy and bioposy.

Gastric MALT
H. Pylori has an important role in parhogenesis of Gastric Mucosa-Asso Lymphoid Tissue Lymphoma. These Lymphomas may regress after eradication of H. Pylori, IF THERE IS NO METASTASIS. If Pylori eradication fails then Chemotherapy is the choice (CHOP). In the old days they used to do radical gasterectomy. Tx Pylori with Omeprazole+Clarithromycin+Amoxicilline.

Gastroschisis:
Bowel protrudes through a defect. Bowel not covered with protective membrane. First thing is to cover the exposed bowel with sterile wrapping, then iv access for nutrition and then iv antibiotics, then surgery to fix the defect. Omphalocele: Bowel protrudes through unbilicla ring. Bowel covered with amnioperitoneal membrane. Asso with other congenital abnormalities. Management is first wrapping, then orogastric tube to decompress stomach. If <2cm>

Gaucher’s Dis:
Sphingolipidosis due to deficiency of glucocerebosidase. Characterized by Hepatomegaly, anemia, leukopenia, and thrombocytopenia, but NOT chery red macula.

General Anxiety Disorder
First line of tx is Buspirone, because it does not show the dependence and withdrawl symptoms asso with Benzodiazepines. If sexual performance is a problem use Nefazodone.

Gentamycin
causes nephrotoc and Ototoxicity, vestibular toxicity that causes the pt to feel dizzy w/I a couple of weeks of use.

GERD - 6
In neonates is regurgitation after eating and failure to thrive. The child assumes the position of tilted head and arched back. Dx is 24hr pH monitoring of esophagus. *****When its unclear whether the pt has nocturnal asthma or GERD, a trial of proton pump inhibitor (Omeprazole) before breakfast is both Dx and therpeutic. ***** Esophagoscopy is indicated when a pt fails to respond to antibiotics and or theye are signs of implications (weight loss in cancer).****Dx Process: once pt presents with cough, and the vigniette says esophagoscopy is normal, the next thing is 24 hour pH monitoring. Then Manometry will confirm dx.****GERD can happen due to hiatal hernia. Chronic GERD can lead to metaplastic change in lower esophagus called Barret esophagus and is a risk for Adenocarcinoma of esophagus.****When pt comes in with symptoms of GERD you need to differenciate b.w Barrets,PUD,Gastritis, or tumor. Endoscopy is the most informative procedure for all these. Now if the vingette says "he has no Dysphagia" then you can skip the normal Barium test that precedes endoscopy and go straight to endoscopy.*****Indications to endoscopy are: 1-Nausea/vomiting, 2-weight loss, anemia or melena, 3-Long duration of symptoms (>1-2 yr), 4-Failre to responde to PPI. So here is the order, if there if Dysphagia first do BS, then EC then Manometry. If no dysphagia, first EC and then Manometry, you can skip BS.

Giant cell arteritis - 3
Don’t wait for biposy start high dose Prednisone right away.***Lab may show elevated ESR and normochromic normocytic anemia. Thoracic aortic aneurysm is a complication of this dis, maybe due to disruption of collagen and elastin.

Giardiasis
Adheres to mucosal surfaces by adhesive disk and cause malabsorption, may lead to weight loss. The MC symptom is foul smelling sttol, fatty stool, bloating and flatulence, nausea, malaise, abdominla cramps. Empiric tx should be given w a course of Metronidazole.

Gilbert synd
Partial absecnce of enzyme leads to : 1-mild uncongugated hyperbilirubinemia, 2-More elevated values happen with stress, fasting, alcohol abuse. 3-normal cbc, 4-normal liver enzyme, 5-complete reversal of hyperbilirubinemia. DDX1:Crigler-Najar-I: cgaracterized by 1-Unconj.Hyperbilirubinmeia 8-30, 2-Normal liver enzymes, 3-High rates od Kernicterus, 4-No response to Phenobarbital. DDX2:C-N-2: characterized by 1-Unconj HyperB, 2-Normal liver enz, 3-No Kernicterus, 4-25% decrease of Bilirubin with Phenobarbital. DDX3:Rotor synd: Conjugated Hyperbilirubinemia.

Glascow coma scale - 2
EYE opening:Spontanous 4, to verbal command 3, to pain 2, none 1. VERBAL response:Oriented 5, confused 4, Inappropriate 3, Incomprehensive 2, non 1. MOTOR respopnse:Obeys 6, localization 5, Flexion 4, abnormal felxtion 3, extenstion 2, none 1. Total 15. Minor injury is GCS of >14. Moderate <13>9. Severe <8.>

Glaucoma
it’s the leading cause of blindness in North America. Its characterized by elevated ICP. Symptoms are sudden onset of photophonia., eye pain, HA nasea. Eye is hard to touch. A non reactive mid dialated pupil suggest Acute Glaucoma.The best Dx is Tonometry. OPEN angle glaucoma: has incidious onsert with gradual loss of peripheral vision and consequence tunnel vision. Intraoclar pressures are high. Opthalmoscopic exam shows cupping of the optic disk.*****once you suspect it do MAPPING VISUAL FIELDS to find out defects.

Glaucoma, Acute closure angle 2
Age 55-70, Acute onset of severe pain and blurred vision, nausea and vomiting. Anterior chamber is shallow with inflammatory changes. Tonometry reveals increased IOP. IV Acetozolamide, Manitol, Pilocarpin with subseq oral. Permanent cure is laser peripheral iridotomy. Avoid Atropine.

Glaucoma, Acute Open angle:
Its more common in blacks. Gradual loss of peripheral vision, tunnel vision. IOP is high. There is cupping of the optic disk with loss of peripheral vision. Beta blockers, Timilol are effective in initial mngmnt. Later on Trabeculoplasty. Its asso with Diabetes.

Glioblastoma Multiform
GBM or HIGH grade astrocytoma. CT shows butterfly appearnce in the frontal lobe of alesion (surrounded by white calcification, looks like cyst membrane but not round). DDX is Brain Metastasis, pt will have a duration of symptoms of <2months,>

Glucagonoma
Triad of Hyperglycemia, necrotizing Dermatitis, and weight loss. It’s a tumor of islet cells in Pancrease . The excess glucagon causes symptoms like glucose intolerance, and hyperglycemia. It also causes a distint skin lesion called 'Necrolytic migratory erythema'. Dx is confirmed by fasting glucose elevation, elevated glucagon, and pancreatic tumor by CT. Surgery is the prefered tx. It doesnt respond to Chemotherapy.

Glucose 6 Phosphatase deficiency:
Also glycogen storage type 1 or Von Gierke Dis. Presents with hypoglycemia, lacic acidosis, hyperuricemia, & hyperlipidemia. Hypoglycemic seizure occurs. Hay hepato and renal megally. Doll face, fat cheeks, thin legs, and a protuberant abdomen.

Glucocorticoud SE
Long term use in asthmatic pt can lead to Neutrophilia by increasing BM release and mobilization of marginated neutrophil pool. Eosinophils and lymphocytes are decreased.

Glycogen storage diseases
1-Von Gierke, 2-Pompe, type 3 and type 4. Read on them.

Gonococcal arthritis
Urethral cultures have the highest yeled than synovial or blood or urine cultures in cases of suspected purulent gonococcal arhtritis.

Gonorrhea - 2
Commonly causes cervisitis, urethral discharge. It also causes Pharyngitis. *It also causes GA(arthritis), which is asymetric migratory polyarthralgia, followed by monoarticular arthritis and rash, palms and soles have multiple necrotic pustules.

Goodpasteur's synd - 3
Involves lung and kidney. There is lower respiratory problems (hemoptosis) and proteinuria (renal failure), there is antibodies against glomerular basement membrane. Sputum shows iron in form of hemosiderin . There is anemia and RBC cast in urine. DDX1:Idiopathic pulmonary Hemosiderosis: Like GP, but there is more copious hemoptasis and its for children. DDX2: Wegner's, involves Upper (rhinorrhea) and Lower (hemoptasis) and Kidney problems. And Granular granulomatosis. Cresent formation.****Caused by circulating antiglomerular basement membrane antibodies. Early removal is imperative in order to minimize the damage tokidney. Emergency Plasmaphoresis is indicated.***Pt presents with massive hemoptosis, weightloss, hematuria and proteinuria. To confirm measure anti glumerulat basement membrane antibodies.

Goughers Dis
Is due to deficient activity of the lysosomal enz, acid beta glucosidase. The typical pt is Ashkani Jew adulescent with chronic fatigue, easy bruisability, bone pain and pathological fractures. The dx is conformed with radiologic (Erlenmeyer flask deformity of the distal femur) and bone marrow studies (Gaucher cells with wrinkled paper appearance).

GOUT- 2
Fluid joint aspiration for dx.*****In pt with frequent attacksof gout not controlled by Colchicine, a 24hr uric acid level in urine is determined. This evaluates whether hyperuricemia id due to over production or under secretion. A value of <800>800 indicates over production, so add a xanthin oxidase inhibitor (Allopurinol).

Gouty arthritis - 2
Very painful, mostly in toe but could happen in long term RA pt in the hands. Alcohol is metabolized to Lactate which competes with Urate for renal excretion leading to accumulation of urate in the body and gout.Cessation of alcohol is important. Acute attack give oral indomethacin, colchicine or steriods. Chronic Cholcicine.

Graft Vs Host Disease
By activated donor T lymphocytes. Targeted at Skin (maculopapular rash in palm,sole and face), Intestine (bloody diarrhea, Liver.

Granuloma Inguinale
Is an STD caused by Donovane granulomatis. Its uncommon in US and is mc in Balck population. It starts with a papule that rapidly develops into a painless ulcer characterized by irregular border and red beefy granular base. Inguinal lymphadenopathy occurs w/o Buboes. Advnced stage has scaring, depigmentation and keloids. Fibrosis may lead to vaginitis and elephantitis. Dx is by identifying Donovan bodies, visulized by giemsa or wright stain of tissue smears, and appear reddish, encapsulated bipolar bacteria found within Monocytes. Tx is tetracycline, 500mg, every 6hr, 10-21 days. DDX1:Ulcer of primary syphlis has roled edges and punched out base. DDX2:Ulcer in Chancroid is very painful and foul smelling, buboes form and are painful. DDX3:in Genital herpes many vesicles appear before they turn into ulcers. DDx4 Ulcer in Lymphogranuloma Veneruem is also painless but its shallow and asso w non-specific systemic symptoms, also lymphadenopathy is inflamatory and does not appear at the same time as ulcers.*****The first thing to do is stop Heparin. However, most pt need anticoag so two options are offered: -Danaparoid and a direct thrombin inhibitor such as Lepirudin or Argatruban.

Granulosa Cell Tumor
Are SOLID tumors. Bimodal distribution. If occur before puberty , Precociouspuberty is presented. It produces excess estrogen and causes pubic hair, hpertrophy of brest and hyperplasia of uterus. Usually removal of tumor reverses the problem. If its in postmenopausal women it causes bleedingand uterus shows myohyperplasia. DDX1ysgerminoma, in young women and children, unilateral and go under torsion. It doesnt produce any hormones. DDX2:Sertoli-Leydig,produces androgen and DEFEMINIZATION, followed by masculinization in childbearing years. DDX3:Mature teratoma or Dermoid cysts, benign and dont produce any hormones. DDX4;Serous cystadenomas, are the mc CYSTIC ovarian neoplasm. 25% are malignant,half cases are bilateral. They dont produce any hormones. Ovarian mass and abdominla pain are presenting features.

Grave's Disease - 3
Tx is radioactive iodine. Might cause hypothyroidism *** Sudden onset of Atrial Fibrilation (irregularly irregulat rhythm with tachycardia) in pregnant women should alert the doctor to look for Hyperthyroidism, GRAVES is the MCC of hyperthyroidism in pregnancy. Remember Dx is made by TSH, T3 & T4, BUT the best SCREENING is just TSH. So next step is ordering TSH. Now the vigniette may not even indicate other signs of Graves, but you must recognize it as well. DDX is Hydatiform Mole, but sonogram will easily show it.

Green stick fracture
Commonly seen in children because the bone is less brittle. Also see torus fracture and plastic deformation.

Growing Pain
Common in children from 2-12. Mostly at night, awakens the pt, responds to massage and NSAIDs. Obs/Reasur.

Guillain Barre - 4
The best way to monitor respiratory function is to measure serial bedside vital capacity, to make sure it stays above 15ml/kg.***DDX is Tick borne paralysis charcterized by rapidly progresive ascending paralysis, absence of feve , absence of sensory abnormality and normal CSF. ****Campylobacter Jejuni is the most frequent precipitant in GB. ***GB is characterized by Ascending paralysis , previous hx of infection, CSF shows albumino-cytologic dissociation(Elevated protein despite normal cell count). Tx is IVIG and plasmophoresis. DDX is Botulism which presents with desceing paralysis, tx is antitoxin.

Gun Shot wound:
If its below 4thrib, level of nipples, then exploratory laparatomy is done.

Hairy Cell Leukemia - 2
It’s a type of B-lymphocytic derived chronic leukemia. Tartrate resistant acid phosphatase stain is Dx.****Picture looks like hairy projections of large cells. BM may become fibrotic so BM aspirate are frequently unsuccessful (Dry Tap). Tx DOC is a purine analog, Cladribine.Its toxic to BM, it causes neurological and kidney damage. Remember CHOP is for Nonhodgkins, Chlorambucil+Prednisone are for CLL.

Hashimoto Thyroiditis - 2
Transient thyrotoxicosis can occur in initial stages due to thyroid-stimulating antibodies. Positive anti-thyroperoxidase antibodies with an enlarged rubbery goiter are virtually diagnostic. The risk of thyroid Lymphoma is 60times greater in these pts. CT shows elragemnt of thyroid around trachea, "Doughnut sign". US shows "psudocycst pattern". RAIU is decreased. Since FNA might miss dx, Core biopsy is prefered. * Anti Thyroid Peroxidase Antibodies are present in >90% of pts.

HCV
If pt is + but no sign and symptoms, just follow up w yearly tests. No Tx.

Head trauma - 2
always do spine xray in pt with falls or accidents leading to head trauma. **** Pt with increased ICP should be treated with hyperventilation , head elevation, and IV Manitol and diuretics. Hyperventilation works by causing vasoconstriction and decreasing the ICP by decreasing cerebral blood flow volume.

Heart Block - 2
1-First Degree: prolonged but constant PR >0.2s . No tx is neede. 2-Mobitz Type-1: or Wenkeback, narrow QRS and progressive increase in PR until a ventricular beat is dropped. It may occur with Digoxin, or Inferior MI. If pt is symptomatic Atropine is gven. 3-Mobitz Type-2: Fixed PR with occasional drop . QRS is wide. Its seen after MI. Its dangerous and all pts have to be monitored in ICU. It can progress to complete heart block and needs Pace maker. Atropine must be by bed side at all times until a permanant pacemaker is inserted. *****Third degree heart block, no atrail impulses will travel to ventricles. , Atrail rate is 80 and Vent is 30. Pts are at risk of suden cardiac death and they should be transmitted to ICU and PERMANENT pacemaker should be placed. Always have Atropine by bed side. B-blocker WILL KILL HIM IMMEDIATELY.

Heat Exhaustion
Characterized by volume depletion under the conditions of heat stress. DDX:Heat Stroke, has CNS symptoms and T>40. DDX2:Heat Syncope:breif syncope after exposire to heat. DDX3:Heat cramps: painful muscle cramps due to depletion of salt in muscles.

Heat stroke
Characterized by body T of >40.5, due to filaure of thermoregulatory center. Rapid Evaporation cooling is the tx of choice. Immersion in ice water is also useful but makes it difficult to monitor the pt.

HELLP Syndrome
Hemolysis & Hypertension (preeclampsia), Elevated Liver enzyme, Low platele(thrombocytopenia). DDX are DIC, Preeclampsia, TTp, HUS, and Acute Fatty Liver of Pregnancy. In AFLP tx is supportive with early dx and rapiddelivery, there is incresed PT & PTT. In HUS, thrombosis of the glomerular arteries, happens in children. Often preceeded by infection. . TTP is wide spread of HUS and it occurs in Adults and Asso with NEUROlogical symptoms. Tx is exchange transfusion or plasmaphoresis with FFP****Delivery is definitive tx for HELLP in women beyond 34 weeks, give Mg SO4 to reduce BP & VAGINAL deliver.

Hematochezia, scant
Pt see fresh blood on toilet paper. Most common causes are hemorhhoids, fistulas or even cancer. If pt is <50>

Hemi Neglect Synd
Involves the Right (Non dominant ) parietal lobe.

Hemochromatosis - 3
Genetic disorder of iron absorption, increased aborption from intestine . Liver, pancrease, heart and joints are mc affected. Pt prsents with weakness, loss ob libido, skin pigmentation, weight loss, abdominal pain and symptoms of DM (polyuria& polydypsia). Trensferin saturation (>=50%) and Ferritin (>1000) is a simple reliable sreen test. Hepatoma(Hepatocellular Carcinoma) is the most serious complications. ***Hay increased Fe and Feritin. Its Auto Recessive causing increased iron absorp and deposition in skin (pigmentation), Testes(decreased libido), pancrease (Diabetes), Joints (Arthralgia), and liver (cirrhosis).*** Pts are at increased risk for Listeria Monocytogenes, maybe due to impaired phagocytosis. Iron overload is also a risk for Yersenia Enterocolitica and sepsis from Vibria Vulnificus which are iron loving bacteria.*** IF pt presents with Joint pain and Hepatomegally and no other sysmptoms suspect it and FIRST thing you do is CBC to check Fe level. It presents with hepatomegaly,hyperpigmentation,diabetes (BRONZE diabetes), arthropathy, heart failure and hypogonadism.

Hemolytic Uremic Synd - 4
When you get Thrombocytopenia (decreased platelet 56000), a MicroAngiopathic Hemolytic Anemia(MAHA) (Increasesd bilirubin, and schitzocytes and RBC fragments and abnormal renal function) in a pregnant woman DDX include HEELP(Hemolysis, Elevated Liver enzymes, Low platelet count), HUS, TTP, DIC, preecclampsia and Acute Fatty Liver of Pregnancy (AFLP). Oliguric renal failure+MAHA+Thrombocytopenia points to HUS. Its characterized by thrombosis of the glomerular arterioles and capilaries. It occurs more frequesntly in childhood and presents with fever, thrombocytopenia, MAHA, HT and renal failure. Usually preceded with viral infection and asso with E. Coli. TTP is a variant of HUS, the thrombosis is systemic and it obstructs microvascular of several organs. It presents with fever, altred level of consciousness, focal neurologic signs, renal failure, MAHA and thrombocytopenia. tx is exchange transfusion, or plasmophoresis with FFP. HELLP is a variant of preeclampsia, in women older than 25, multipara, before 36 weks gestation. DIC is a comsumptive coagulopathy, Presents with elevated PT, PTT, fibrin degradation product and decreased fibrin. ****Occurs in young children. Preceeded by a diarrheal disease. Hallmark is Microcangiopathic hemolytic anemia. Other features are renal filure, fever, oliguria, thrombocytopenia. GI bleeing is common. Purpura and HT. Schizocytes, which represent frabmented RBC. Also Giant Platelets. Intravasculat hemolysis results in elevated LDH, indirect bilirubin and reticulocyte count. BUN andCrt are markedly elevated. . Urine contains, Hb, Hemosiderin, albumin, RBC,WBC and casts.****Its caused by E.coli released toxin, it injures kidney epithelium. Pt prsents with abdomnal pain, young pt, decreased Hb,Hct and Platelets. , bloody diarhea and swollen face. When kidney is damaged mortality is 5-10%.*****Its caused by toxin released by E.Coli, it destroys epithelial lining and causes bloody diarrhea. Subsequesnt activation of coagulation system and red cell hemolysis causes Jaundice. Its mc in children adn initial presentation is abdomnal pain and diarrhea. The classis TRIAD is Uremia(Renal Failure), Thrombocytopenia, and Hemolytic Anemia. Tx is generally suportive and involves Plasmaphoresis and Dialysis if necessary and steriods. DDX it with Campylobacter jejuni, althought it causes bloody diarrhea, there is no Thrombocytopenia.

Hemophilia
Bruising since child hood, excessive bleeding in a tooth extration procedure and uncle having the same problem. In these pts recurrent Hemarterosis ma lead to injury called 'hemophilic arthropathy'. Iron (HEMOSIDERINE) deposition and synovial thickening with fibrosis is characteristic. ***** Its X-linked. PTT is elevated and PT & BT are normal.

Hemothorax
Collectin of blood in pleural cavity. Tachypnea,tahcycardia,hypotention, deviated trachea , dullness to percusion, elevated JVP, fluic in pleural cavity(seen in Cxr) and collapsed lung. . Lung is the usual bleeding source 2ary to rib fracture. Its very important to evacuate the blood in order to stop bleeding, which stops on its own. Best initial interventon is to insert a low anterior chest tube to remove blood. Surgical Thoracotomy is indicated when more than 1500ml blood recovered when tube is inserted, or if more than 600ml blood loss after 6 hour post tube insertion.

Henoch-Schonlein Purpura
A common vasculitic of childhood, its commonly seed after URT infection and is more common in males. Classis findings are palpable Purpura in buttucks and lower extrmity. Peripheral edema nd scrotal swelling. Renal finding are Hematuria dn proteinuria. Tx includes steriod and monitoring renal function. When pt prestns with Abdominla pain two pathologies should be rules out emergently, GI bleeding and Intussuseption. Intus is characerized by sudden onet of abdominlapain with large amount of blood in stool. This is a surgical emergency and is tx with Air/barium enema.*****HSP is an IgA mediated vasculitis of small vessels, which results in rashes, arthralgias, abdominal pain and renal disease. Immuno florescence microscopy reveals IgA deposition in the kidney.

HepA
Acute disease asso w travelling. If travel <4>4 weeks give vaccine.

Heparin induced thrombocytopenia - 3
Its seen in 5-15% of of pt taking Heparin with onseet b/t 3-15 days and resolution in 4-5 days of discontinuation. While PTT is a therapeutic effect of heparin, the thrombocytopenia is an adverse effect. So PTT is increased and Platelets are decreased.****Hospitalized pt who develop DVT after a period of bed rest are standardly treated with Heparin. An adverse effect is Thrombocytopenia, along with thrombosis (causes stroke sysmptoms). The combination os Arterial/Vnous thrombosis and thrombocytopenia in pts receiving Heparin is suggestive of HIT. Antibodies against Heparin-Platelet factor 4 complex are responsible for this. The antibodies activete platelets which will cause their removal form circulation.

Hepatic Adenoma
Is a benign tumor seen in mieedle age females taking OCP . Intra tumor hemorhhage is a major comlication. Dx is by bipsy, atypical hepatocytes with glycogen deposits. Tx Superficial and large adenomas are resected. But smaller and asymptomatic onces are under surveilance with imaging.

Hepatic Encephalopathy – 3. Hepatology. 6/3
Characterized by reversal of sleep cycle, asterixis, porgreive coma, and Delta waves in EEG. It’s a CNS complication of liver failure secondary to accumulation of ammonia in blood becuase of inability of live to detoxify ammonia (that comes from intestine) into urea. Eliminating toxic enteric products is the other main therapy: (1) The bowels should be cleared with enemas. (2) Dietary protein should be eliminated, and oral or IV carbohydrate should be given. (3) Oral lactulose should be given. (4) Oral neomycin.****Pt with hepatic failure should be given vit K to correct PT and if they are bleeding in this setting, or he needs immediate surgey, then FFP is indicated.Platelet transfusion is only indicated if it falls below 20,000-30,000. Cryoprecipitate is requied for coag factor def like factor VIII. Packed RBC is indicated when RBC is <8>

Hepatitis B –8. GIT. 6/3
Hepatitis B virus Ig + Lamivudine is used to prevent recurrent HBV after liver transplant.For HepB Interferon+Lamuvidine. Vaccination criteria: if someone is exposed and has Documented response there is no need for furthur action. If vaccined but no response to vaccine must be vaccined again. If vaccined many times and still no response, give HBIG on exposure. If exposed to virus and never been vaccinated before, give HBIG w/i 24 hrs and start HBV vaccination. Best screening test to dx acute HepB is AntiHBcAg and HBsAg. **** HepB vaccine is a recombinant vaccine containing HBsAg, which stimulated Anti HBsAg, person who receives it will have immunity and thus be positive for Anti HBsAg and negative for HBsAg. Person with ACUTE HBV will have HBsAg, HBeAg and IgM Anti HBcAg. Chronic HBV will have HBsAg only, for >6months. Person with recovery phase HBV will have AntiHBsAg, HBsAg, Anti HBeAg and ABSENT HBeAg. ****If seroogy shows HBsAg, HBeAg positive and high titers of HBV DNA, he has Chronic HEpB. The 2 drugs approved are Inerferone and Lamuvidine. Either one is indicated for pt with positive HBsAg,HBeAg,HBV DNA and persistantly elevated ALT. Degree os elevation of ALT is important in deciding the Tx. Serum ALT of mor than two times the upper limit requires need for Lamuvidinr or alpha interferone. If less than that its not useful. Generally Lamuvidine has less SE and easier to administer (ORAL). Inteferone is not successful in young children and immunocompromised pts. NOW, if the pt has just been exposed to virus, POSTEXPOSURE and has never had response to vaccine, we give vaccine+Ig, and if did have response and has antibodies then we just give Ig. ****Of all acute Hep B cases 90% recover, minority go to chronic, out of those 1% goes to Fulminant hepatic failure, defined as hepatic encephalopathy that develops w/I 8 weeks of the onset of acute liver failure and evidence by marked increase in ALT. and signs of Hepatic encephalopathy. Liver transplantation is the only effective method for tx, so initial step in tx at this point is to put her name on the list****Newborn of mothers with active HebB should be passivly at birth with HepB immunoglobin followed by Vaccination. *****Transmission from mother w chronic dis & +HBs antigen and Hbe antigen to the fetus is 90%.****If given choice b/w Interferon and Lamuvidine in a depressed pt, pick Lamuvidine cause interferone is CI in Psych pts..

Hepatitis C - 4
Risk factors for rapid progression of liver fibrosis in chronic HepC are: 1-Male sex, 2-Acquire infection after age of 40, 3-Co-infection with HepB or HIV, 4-Alcohol intake. Complications of Chronic HepC: 1-Cryoglobulinemia (causes Membraneous glomerulonephritis), 2-B-cell Lymphomas, 3-Plasmocytosis, 4-Autoimmune dis like Sjogren and thyroiditis, 5-Lichen Planus, 6-Porphyrea cutanea tarda, 7-ITP. Management: All pt with mild Hep C(alevated ALT,HCVRNA, moderate bridgenecrosis) should get Interferone+Ribavirin. Liver transplantation is the last step and its done when Pt and Albumin levels are very affected. **** HCV RNA is the single most sensitive serological marker to screen HepC, HCV RNA antibody takes months to show up dont do that one. **** All pts with chronic HepC, including Pregnant women, should be tested for HepA and HepB and if not immuned should be vaccinated which is safe for pregnancy. Ribavirin & Interferone are teratogenic.****Transmission risk could be a steady sex partner but the chances are low.

Hepatitis E
Asso with pregnant women.Ocurs in India,Asia, Africa and central USA.Ther is no vaccin or Ig avaible for HepE.

Hepatocellular carcinoma- 2
Most are palpable mass. It accts for 80-90% of liver cancers. Occurs more often in men. Cause is unknown but contributing factors are Chronic liver disease, HepB & HepC, hemochromatosis. Dx is by abdominal CT. ***High serum AFP (>500) in an adult with liver disw/o an obvious GI malignany is HCC. Pt has a hx ox Chronic HepC. Dx is confirmed by biopsy, Tx is surgical resection.
Hepato-Renal Synd
Hypotension, Hyponatremia, Azothemia and oliguria with normal urine analysis with sever liver disease. No tx is available. Pathogenesis is not clear. Initial mgmt is careful volume loading and withholding of Spironolactone and Furesamide.

Hereditary Spherocytosis - 3
Osmotic fragility test is dx.***An auto RECESSIVE dis. Folic acid is encouraged in ALL pts. MCV is normal or Elevated. They rarely require transfusion unless they have Aplastic crisis. Splenectomy is usually curative.****The tx for most pt involves Folic acid oral and blood transfusion. during periods of extrmem anemia. Splenectomy is considered if htey are refractory to medication. The most feared long term complication is overwhelming sepsis with encapsulated bacteria Strep Pneumonia. The risk is present for up to 30 years and even longer after splencetomy. Current recommendations state that pt shuld receive anti-penumococcal,Haemophilus and Meningococci Vaccinces several weeks before the operation and daily oral Penicilline prophylaxis for 3-5 years following splenectomy.

Herion Withdrawl
Signs are muscle spasm, abdominla pain, rhinorrhea , lacriation, sweating. Dilated pupils. These are oppsit signs when pt is toxicated, pin point pupil, constipated, depressed respiration and bp. DDX is Cocain withdrawl:irritable, fatigue, HUNGRY(opped to anorexic when toxicated).

Hernia
Respiration and hemodynamics are altered after repair of large hernias. Because large hernia content is displaced inside the peritoneal cavity, the pressure inside the cavity increases. The diaphram is pushed upwards and this impaires respiration, causing hypoventillation. At least a week is needed for the pt to accomudate to its new state. Early physiotherapy and respiratory excercises (blowing against resistance) are mandatory to prevent , Atelectasis, mucus pluggung and possible subsequent penumonia development.

Herpes Simplex Encephalitis -2
Mainly affects the Temporal lobe, bizare behavior and gustatory hallucination. CSF show Lymphocytosis, low glucose, and elevated proteins. HSV PCR is the gold standard. Tx Acyclovir.

Herpes Simplex Keratitis or Herpetic Keratitis due to HSV-1
Simplex More common in young pt. Zoster is mc in old pt. DX is with Slit lamp. Common in health workers.

Herpes Zoster Ophthalmicus
DDX with Herpes simplex keratitis is addition of vesicular rash in Varicella-Zoster. "Bacterial keratitis" occurs w contact lenses.

Herpetic Whitlow
Is the mc viral infection of the hand. The appearance of vesicles on the volar or dorsal distal phalanx is diagnostic. Caused by type I or II herpes simplex and its self limiting. Dentist are at increased risk.

Hirshsprung dis - 2
Pts with Down, are more likely to present with Duedenal atresia, Hirshsprung, endocardial cushin defect and acute leukimia. Typically there is a "double buble sign" is seen in abdominla radioraph.****This is an emerrgency, so if the infant has it and mother refuses tx, then go ahead and treat the infant because court order will take a while.

Hirsutism
Women produce androgens. DHEA-s and Testosterone in adrenals and ovaries. DHEA-S is only in adrenals by adrenal tumors. See T2-Q16. ACTH increase in pts w hirsutism is seen with ectopic or pituitary dependant Cushings dis. ACTH increases the production of cortisol as well as angrogens from the adrenal glands, however, the adrenal glands show diffuse hyperplasia rather than a discrete adenoma.

Histoplasmosis - 3
Is a common and asymptomatic infection in endemic areas of Mississippi & Ohio. Found in bird or bat dropings. ***** It’s the mf endemic fungal infectinin USA. Disseminated histoplasmosis occurs in immuncompromised pts. Presnece of mucous membrane ulcers, hepatospleenomegally and pancytopenia are clues to Dx. **** Tx is IV amphotericine followed by lifelong Itraconazole. Histoplasmosis happens to HIV pt in OHIO.

HIV- 30
TB occurs in the course of HIV when the CD4 counts are >200. Uper lobe consolidation and cavitation is typical Xray fiding. Asso with Hairy Leukoplakia (white painless lesion that appears hairy, found on the tounge, caused by EBV). All HIV pt should have the following done: 1-Hx & PE, 2-routine chemistry and hematology, 3-two plasma hivRNA levels, 4-CD4 count. 5-VDRL for syphlis. 6-PPD test. 7-Anti Toxoplasma antibody titer. 8-MMSE. 9-Pneumococcal vaccine, unles CD4 is <200.>5mm in HIV pt then Prophylactic theapy with Isoniazide + Pyridoxine is indicated for 9months. **** in a pt with bilateral interestitial pneumoni, the agent is PCP. Tx is Trimeta-Sulfa, in pt with moderate to severe infection adding corticosteriods has reduced mortlity. Indication for steriod is 1-PaO2<70>35. *** Best screening test is Eliza, then confirmatory with Western bloting. *** Multiple ring enhancing lesionson on CT in aids pt, is tx w Sulfadiazine and Pyrimethamine which is both diagnostic and therapeutic. Remember trimeta-sulfa is for prophylaxis.**** If health worker is infected, right away get blood for serology and start him on 3 drug therapy while waiting for results. Blood serology should be repeated in 6 weeks, 3 and 6 moths. ***** HIV pts are at increased risk of TB which may cause collapse of vertebral bodies and intervertbral disk.*** DDX of Diarrhea in HIV pt with CD4=80. 1-Cryptosporidium :Modified Acid fast stain shows Oocytes in stool. It becomes persistent in CD<180,>200. ******* PML (by JC virus) presents in HIV pt with CD4<100,>90% effective in dignosinfg PCP in HIV pt, especially when CD<200.****>50% have encephalitis. Necrosis of inner retina as white flyffy lesions. 3-Herpes Simples Keratitis, pain, photophobia and decreased vision. Dendritic ulcer is the mc presentation. 4-Herpes Zoster Ophtalmicus, mostly in elderly, or HIV pt. Presents with fever, malaise, itching and burning around the eye. Vesicular rash following trigeminal nerve. If eye is involved hay conjunctivitis and dendriform corneal ulcers. 5-HIV Retinopathy, presents as benign cotton wool spots in retina which remits spontaneously. ****Tx for Condylomata Acuminata is Podophyline. *****Cryptococcal Meningitis infection in AIDS pts, tx ia iv Amphotericin+Flucytosine.

Hodgkin’s :
Tx is ABVD. Adriamycin SE is cardiomyopathy. Bleomycin SE is pulmonary. Dacabrazine is Ematogenic. Vinblastin SE is Neuropathy leading to constipation.

Homocystinuria
Marfan features+mental retardation+thromboembolic events+downward dislocation of the lens is suggestive. Tx is high dose Vit B6.

Hordeolum ( Stye)
a common staph abscess of the eyelid. Tx is warm compresses. Incision and drainage is performed if resolution does not begin in the next 48 hours.


Hormone Replacement Therapy:
According to 2005 studies HRP increases the risk for Cerebrovascular accident, CV disease, Breast Cancer and DVT. It Decreases the risk of Hip fracture, Colorectal cancer and vulvovaginal atrophy.

Human Bites
Tx of choice is Amoxicilline/Clavulanic.

Humeral Fractures
Tx of choice is closed reduction and hanging cast. In cases of Segmented fragtures, or open fracture in trauma, pathologic fracture and vascular structures, open reduction and internal fixation is done.

Huntington - 2
Atrophy of caudate nucleous is characteristic. Mood disturbance, Dementia, Chorea and family history.

Hyaline membrane dis
should be suspected in preerm infacnts. With respirtory distress and hypoxia, NOT responding to oxygen therapy. The characterisric cxy shows fine reticuar granularity of the lung parenchyma. Tx includes early ventillation and surfactant.

Hydatid Cyst of Liver
Is due to infection with Echinococcus ganulosis. Can be contacted from dogs. It can cause Cyst in lung,muscle,bone,liver. In most pts its asymptomatic. "Eggshel calcification" of a hepatic cyst on CT is highly suggestive. Aspiration is NOT indicated due to chance of anaphylactic shock. Tx is surgical resection under the cover of Albendazole.

Hydrocele
1-Non Communicating Hydrocele is refered to a fluid containing sac which is a remnant of processus vaginalis. The upper limits of the mass is easily identified. Most cases of NCHC will dispear spontanously by the age of 12 months. 2-Communicating Hydrocele, the upper limit cant be reached and it treated with surgery.

Hydrocephalus
Happens in infants. CT scan , dialation of entire ventricular system with distinct enlargment of subarachnoid space ofer the cerebral cortex), is very suggestive of non-communicating or commuicating hydrocephalus.Accumulation of blood in subarachoid space may destry Arachnoid villi and whose job is to absorb CSF and lead to hydrocephalus. SAH is the mcc od communicating hydrocephalus. Its very common in PREMATURE infants. Non-Communicating hydrocephalus examples are Dandy-Walker and Arnorld-Chiari. DW shows a cyctic exapansion of fourth ventricle, and AC will rreveal posterior protrusion of posterior fossa through foramen magnun.

Hydroxychloroquine
is the safest drug for SLE but rarely it may cause serious eye dis including Macular degeneration, so eye exam at 6mo to 1yr intervals should be performed. Remember the mc SE is Alegic skin reaction. Also CI is G6PD Def.
Hyperandrogenism
A female prsetns with virilization, balding and clitonegaly. What to do next? Rapidly developing hyperandrogenism with virilization is indicative of androgen-secreting neoplasm of OVARY or ADRENAL. So next measure serum Testosterone and DHEAS to determine the site of tumor. Elevated Testosteone level with noraml DHEAS indicate ovarian source, but Elevated DHEAS with normal Testtosterone indicate Adrenal source. Now dont try to measure 17-HO, because that is for Congenital Adrenal Hyperplasia and happen very early inlife.

Hypercalcemia - 8
Hypercalcemia due to metastatic tumor (Breast) cancer. In all womem w metastaic breast cancer and radiographic lytic bone disease who are receiving either hormone therapy or chemotherapy (Tamoxifen), IV Pamidronate (Biphosphonate) is recommended. * In acute severe hypercalcemia, its importnant to FIRST give IV 0.9% Saline before giving Furesamide. Its complicated read MERCKs. *** Constipation is the mc GI presentation in pt. The important renal manifestation is neprolithiasis. Chronic therapy with vitD is a major cause for Hypercalcemia. Tx is stopping vitD tx and low calcium diet, keeping urin acidic and give corticosteriods. ****Hypercalcemia is the mc Paraneoplastic syndrom that is asso with Squamous CC lung cancer. Hypercalcemia production is due to ectopic PTH related petide (PTHrP) production.*****Immobilization can lead to HyperCa. Prolonged bed rest can lead to accelerated bone resorption, OsteoClastic activation in increased bone turn over is established. Biphosphonate therapy is helpful. DDX:Rhabdomyolysis, HypOcalcemia (not HyperCa) is seen, normaly du eto increased binding of Ca to Phosphorous that was released by muscle. DDX2:Hypercalcemia due to Malignancy, Causes include local osteolytic metastasis, secretion of PTHrPand increased 1-25VitD. DDX3:Hypoalbuminemia, Any change in albumin levels will affect total serum Ca levels w/o affecting the ionized fraction. In pt with decreased albumin total serum ca is decreased.****Malignancy is a frequent casue, lilke Breast cancer. There are various mechanism by which cancer causes hypercalcemia. 1-Procuction of Cytokins:Tumors that are metastatic to bones cause local osteolysis by production of Cytokins like IL-1 and TNF. The mf tumors that produce hyperalcemi byt his method are lung and breast cancer. .2-PTHrH (related hormones):Themcc of Hypercalcemia in pt with non-metastatic solid tumors is production of PTHrH. in these pt PTH is low. 3-Calcitriol:Hypercalcemia in case of Hodgkins is due to Calcitrol 4-Ectopic PTH:Its very rare and has been reported in ovarian cancer, lung cancer and neuroectodermal tumurs.***Hypercalcemia 2ary to malignancy is dueto multiple reasons, including osteolytic metastasis, secretion os PTHrP, increased formation of 1,25-dihydroxyVitD, increased interleukins-6. Generaly, hypercalcemia due to malignancy(2ary HCa) is higher than primary HCa. *****Pt with Squamous cell carcinoma will have Hypercalcemia, now if he vomits, he will be at risk for Acute Hypercalcemic Crisis. Tx is first normal saline for hydration and 2nd Furesamide to maintian urine output at 200cc/hr. Biphosphonate Pamidronate would work too, but by givng saline you both take careof hypercalcemia and hydrate to prevent Azothemia (renal failure) in the pt.

Hyperemesis Gravidarum:
In a pregnant female in her first trimester, who presents with severe and persistant vomiting think of HG. It is severe enough that requires admission. Cause is unknown but related to elevated HCG, which maybe indicative of Hydatiform mole. Order HCG to confirm that levels are consistant with the stage of pregnancy.

HyperKalemia - 4
MC due to Rhabdomyolysis. Ekg shows Tall T waves. Calcium Gluconate is given first and then Insulin, glucose and Kayexalate. Insulin drives K in to the cell, its given with glucose to prevent hypoglycemia, and Kayexalate exchanges Na for K in GI and excretes K. * Caused by either Medication (K sparing diuretis, ACE inhibitors, NSAID) or PseudohyperKalemia (the lab sample is hemolysed), decreased renal K excretion, transcellular shift, increased K intake. The most serious SE is cardiac toxicity, so do an EKG in ALL pts. It shows peaked T waves, prolong PR and QRS, progressive widening of QRS leads to Ventricular Fibrillation or Asystole. The approach to Tx depends on EKG and degree of HyperKalemia. Immediate tx is needed if there is cardiac toxicity, muscular paralysis, or K>6.5. For these pts 10ml of 10% calcium gloconate stabalizes cardiac membrane. To lower K level , insulin or B2 agonist is used since they drive K into cells. Sodium Bicarbonate can also drive K into cells. Slower acting tx is loop or thiazide diuretics which excrete K. Dialysis is reseved for pts with renal failure and those with lilfe threating hyperlalemia wich wont respond to medication. So...If the pt is Asymptomatic and just non malignant hyperkalemia, just discontinue Amiloride ( to get rid of K ) for a weak and recheck. If the pt has evidence of cardiac tox or K is >6.5, then give Calcium Gloconate and IV Dextrose plus Insulin. **** best drug to excrete K from body is Kayaxelate.


Hyperparathyroidism - 3
Increaed Calcium, decreased phosphate, increased PTH. Could be Asymptomatic. While all pt with symptomatic HPTshould have parathyroidectomy, not all Asymptomatic pt need this surgery. Criteria is as follows: 1-Serum Ca level at least 1mg/dl above upper normal lilmit, 2-24hr urinary Ca above 400mg, 3-Young age<50,>60, identified during routin lab work. While Parathyroidectomy is needed for all symptomatic pts, not all Asymptomatic pts need surgery. Criteria includes: 1-Serum Ca level at least 1mg/dL above the upper limit of normal. 2- 24hr urinary Ca level >400mg. 3-Young age <50.>

Hypertension - 7
In elderly HT leads to Benign Nephrosclerosis. HT is the 2nd mcc of renal dis in US. The process of kidney damage evolves from Nephrosclerosis to Glomerulosclerosis. Nephrosclerosis is characterized by hypertrophy and intimal medial fibrosis of renal arteries, whereas, GlomeruloSclerosis is progressive loss of glomerular capilary surface area and glomerular and peritubular fibrosis. Microscopic Hematuria and proteinuria occurs due t glomeriular lesions. So pt presents with Anemia (decreased Hb). DDXiabetic Nephropathy, is the leading cause of end stage renal disease in US. Increased extracellular metrix, , basement membrane thickening, mesangial expansion and fibrosis characterize DN. ***Isolated SYSTOLIC HT, is an importnat cause of HT in Elderly.Pathophys is decreased elasticity of arterial wall, leading to increased SYSTOLIC bp, w/o diastolic bp leading to wide pulse pressure.Hydrochlorothiazide is the DOC. DDX:Aortic Insufficiency can cause the same systems, Echo will differentiate.****Tx of choice for Pt with Intermittant claudication due to atherosclerosis and HT is Ca chanle blocker. ****OCPs are common causes of 2ary HT caused by Estrogen mediated increase in in teh synthesis of Angiotensinogen in the liver. So stop taking the OCP and HT should go back to normal. If it didnt then its Essential HT and life style modification can be tried. If that didnt work either, then the next step is Thiazides.****Alcohol is a risk factor for HT, Smoking is not.****The tx of choice for Isolated Systolic HT (150/70,160/78) is Thiazides low dose. ****Lifestyle modification should ALWAYS be a part of mngmnt. All pts should be encouraged to lose WEIGHT , reduce SALT, avoid excess ALCOHOL (3bottle a week is excess). and stop SMOKING. This is more important than DRUG MODIFICATION. REMEMBER.

Hypertensive Retinopathy -2
Don’t show any symptoms assos with visual loss. Initially hay focal spasm of arteriols followed by progressive sclerosis and narrowing. Fundoscopy may reveal AV nicking, coper or silver wiring, exudates and hemorrhages.*****Grade1=slight AV nicking. Grade2=Copper wiring, AV depression with humping heads. Grade3=Silver wiring, flame shaped hemorrhages, exudates. Grade4=Flame shape hemorrhages, exudates and papil edema.

Hyperthyroidism - 5
Hyperchlosterolemia (increased LDL) is the most frequent lipid abnormality in pts. *****High estrogedn levels in pregnancy result in increase TIBG production by liver. So production of T4 nd T3 is increased (but not Free ones) . This increase however does not result in clinical symptoms because excess T3&T4 are bound to excess TIBG. And since Free T3&T4 are the same TSH will be normal.*****Atrial Fibrilation is a common complication in hyperthyroidism, Graves disease. In pts with Hyperthyroidism related tachysystolic Atrial Fibrilation a beta blocker,propranolol, is the doc. **** Antithyroid drugs, PropylThioUracil and Methimazole are asso with Agranulocytosis. Immune destruction of granulocytes starts w/i 90 days post therapy. Fever and sore throat are indicative of Agranulocytosis. Monitering in ineffetive. Stop othe drug immediately.****Palpitation should make you think of it. Thne the 1st test is TSH, almost all pt have low TSH (only exception is TSH secreting pituitary Adenoma). If TSH is low then measure Free T4, if its elevated Dx is established. Then do 24-hr thyroid radioiodine uptake to ddx Graves form the rest. Propranolol is initially used for symptoms until definitive cause is known. Radioactive iodine is tx of choice for ALL Grave's pts, however, Propranalol is STILL best initial choice. PTU can be used but hyperthyroidism can recur w/i 6 months. So PTU is only used when Iodine tx is CI, like in Pregnancy. Subtotal Thyroidectomy is also curative but its not the INITIAL tx of choice.

Hypertrophic Osteoarthropathy
Characterized by chronic proliferative periostitis of long bones , clubing of fingers and synovitis, Its asso with Squamou cell carcnimoa and Adenocarcinoma of the lung.

HypoCalcemia
Plasma calcium exists in three forms: ionized calium (45%), ALBUMIN-BOUND (40%), and calcium bound to organic and inorganic anions. Homeostasis of these forms is significantly influenced by the extracellularpH level. An increased pH level causes an increase in the affinity of serum albumin to calcium, thereby increasing the level of albumin-bound calicium, and consequently decreasing the levels of ionized calcium. Ionic calcium is the only physio logically active form, which means the decreased levels of this form can result in clinical manifestations of hypocalemia (crampy pain, paresthesias and carpopedal spasm). Increased extracellular pH levels (Respiratory Alkolosis) can cause an increase in the affinity of serum albumin to calcium, thereby increasing the levels of albumin-bound calcium, and consequently decreasing the levels of ionized calcium leading to hypocalcemia. *****Hay increased DTR. *****Hypocalcemia can occur during or right after SURGEY, especially if transfusion is involves. First manifestation is increased DTR. HypoMg manifest with Decreased DTR.

Hypochondiasis
Symptoms occur durng periods of stress (med student worried about intracranial hemorrhage), pt shoudlbe asked about current emotional stresses and then referd for a breif psychotherpay.

Hypogonadism
T9Q4. Re-read Merck highlights.

HypoKalemia - 2
A35. All Beta-2 agonist reduce serum K by driving it in to cells. In occasional pt they cause HypoKalemia. So any pt taking B2 and complaining of muscle weakness Hypokalemia must be rules out. PEFR (peek expiratory flow rate) and Cxr are not of any use. We need to do Serum Electrolyte panel. and EKG to see "U" waves. Beta-2 also produces a more common SE of Fine resting tremor of fingers and peripherla edema.

Hyponatremia
Is a bad prognostic factor for pt with heart disease.

Hypothermia
Fluphenazine causes hypothermia by causing vasodialation and inhibition of shivering. Hyperthermia is common in drug abusers.

Hypothyroidism - 5
Generalized resistance to thyroid hormones.***Its asso with wide spectrum of musle involvement ranging from astmptomatic elevation of CK to Myalgia, muscle hypertrophy, myopathy. So suspect it if hay elevated CK and Myopathy. *It’s the most common SE of radiation therapy for Graves. * Asso with Hyperlipidemia. So unexplained hyponatremia, hyperlipidemia ane elevated serum muscle enzymes are indication for thyroid function tests.****Always rule out Hypothyroidims in a pt with Major Deppresive Disorder.If this option, "ordering blood test" for TSH was offered pick it over other options. ***** Thyroid dysgenesis is the mcc of congenital hypothyroidism in US.

Iatrogenic Esophag perforax
Pt comes back w/I hours with problems. Do contrast study of esophagus, if perforation is present , priary closureof esophagus, , and drainge of mediastinum must be done w/I 6 hours to prevent development of Mediastiitis.

Ichthyosis Vulgaris(Lizard Skin)
Dry and rough skin with horny plates over the surfaces of the limb. Tx is minimizing bathing, Oral retinoids (CI in pregnant women)

Idiopathic Pulmonary Fibrosis
Presents in 4-5 decade of life, fatigue,anorexia,weightloss, rthralgia, cyanosis and clubbing. Xr shows bilateral interstitial involvement. Biopsy is done to rule out sarcoid. Tx is Steriods. Mean survival is 2-5 yr after dx. Pleurodesis (where visceral and parietal pleura are fused, is used to treat recurrent pneunomothorax and effusion) is not used here.

Iga Deficiency, Misc 6/2
Recurrent sinopulmonary and GI infections (diarrhea), and anaphylactic transfusion reaction. Dx is made if IgA serum concentration is <7mg/dl>

IgA Nephropathy
Is the mcc of glomerulonephritis in adults. Pt have recurrent episodes of gross hematurea, beginning 1-3 days after upper respiratory infection. Serum complement levels are normal.

Immune Thrombocytopenia
Occurs in children 2/6 yo. Pathogenesis involves antibodies that bind to platelets and subsequent destruction of these complexes in spleen. Its preceded with viral infection, and presents with petechia,purpura,hematuroa,or GI bleeding. No adenopathy. Lab shows no abnormality ecept thrmbocytopenia (60000). The course is felf limited. It requires no TX. If thrombocytopenia is <30000corticosteriods>

Impetigo Bullous – 2 . Dermo. 6/3
Tx: Mupirocin ointment is choice for local impetigo. If no respose then treat systmicallly. Because most cases are caused by penicillinase-producing staphylococci, Cloxacillin or a 1st-generation cephalosporin is the drug of choice in severe cases. Penicillin-allergic patients should receive cefadroxil or cephalexin rather than erythromyc.

Incontinence - 4
1-STRESS incintinence: Occurs when there is a sudden increase in abdominla muscle. Pelvic muscle exercise (Kegel exercise) Urethropexy are rcommended tx. It’s a CC of incontinence in older women, HIGH PARITY is a major risk factor. A high number of vaginal deliveries may lead to pelvic floor weakness over a period of time. Urethra relapses outside the pelvic so whenrver there is increase intraabdominal pressure (cough,sneez,laugh) urine ensues. Aggrevating factors are Obesity, pregnancy, COPD and Smoking. Postvoid cystometry is normal. Tx include Kegel excercise, esterogen in post menopasusal women. Surgical tx is Burch and Sling procedures. 2-URGE Incontinenceetruser instability, blader irritation form neoplasm, and interestitial cyctitis result in UI, which causes sudden and frequesnt loss of moderate to large amount of urine. Often accomodated with Nocturia. 3-OVERFLOW: Diabetic Nephropathy causes OI. Characterized by loss of small amount of urine from an over extended bladder and a markedly increased residual volume. There is hx of DM which is not controlled. ****CC are certain medications (Ibuprofen), Diabetic nephropathy, MS and spinal cord injury. NSAIDs have an inhibitory action on the detruser, so the first step is to stop NSAID. Then cholinergic drug (Bethanechol) should be added afterwards to improve detruser action . Intermittent self catheterization can be used.****One of effects of epidural anesthesia is urinary retension due to denervation of bladder. When bladder presure is > sphingter pt urianates until balace is achieved again. This incontinence is transient. PE may show distended blader. Postvoidal vol is high. Tx is by Intermittant cathaterization until control is regained. Oxybutyrin is used for Urge incontinence. Urethroplexy is for stress incontinence.

Infantile SAH:
CT scan shows dilation of entire ventricular system with distinct enlargment of subarachnoid space over the cerebral cortex, is suggestive of nonobstructive or communicating hydrocephalus secondary to SAH. SAH is the mcc of communicating hydrocephalus. Accumulation of blood in subarachnoid space may lead to destruction of arachnoid villi and cisterns (that absorb CSF), SAH is caused by intracranial hemorrhage common in premies. DDX Arnord Chiari, non-communicating, protrusion of structures through foramen magnum. DDX Dandy –Walker, NC, cystic expantion of 4th ventricle.

Infectious Diarrhea
Classified into 2 types, bloody or non bloodt. Bloody is caused by E.coli O157:H7 most commonly, also by Shigella, salmonella, Campylobacter, E. Histolytica and Yersinia and C. difficile.

Infectious Mononucleosis - 7
Heterophil antibody test is sensitive and specif. If its negative and you're still suspitious, do EBV specific antibody test. Splenic rupture is a serious complication. So pts with splenomegally are advised bed rest and avoidance of contact sports until no more spleenomegally. Glucocorticoids are indicated if IM is complicated by upper airway obstruction, autoimmune hemolytic anemia, and thrombocytopenia( and resultant petechia). *IM is caused by Ebstein Bar virus. Sometimes it is detected only after pt develops a characteristic polymorphic rash after taking Ampicilline for an apparant upper respiratory track infection. **** Is asso with Autoimmune Hemolytic Anemia.*****Blood smear with Atypical Lymphocytes ( Large basophilic Lymphocytes) should make oyu tink of it. It might also be in Toxoplasmosis by CMV is the mc organism.*****A negative Heterophile antibodies dont exclude IM, because sometimes they appear later in the course.


Infective Endocarditis – 14, Infx 6/2
Generally if the procedure involves bleeding, prophylaxis is recommended. For procedures that is low risk like, GI endoscopy, there is no need for prophylaxis. Tx inclludes: 1-For IV drug user is IV Vancomycin+ IV gentamycin, since the incidence of MRSA is increased in IV users Vancomycin is better than Nafcilline). 2-For non IV users IV Nafcilline+ IV gentamycin. If IE is due to Strep Bovis, he is at risk for Colon cancer, colocoscopy is recommended. Chordinae tendinea rupture occurs as a complication if IE. **** Pathophysiological consequesnces and clinical manisfestation od IE can be explained by:1-Cytokine production, responsible for fever. 2-Embolization of veg fragments that leads to Pulm and Spleen infarction. 3-Hematogenous infection of sites. 4-Tissue injury due to Immune complex and immune responses to the deposited bacterial antigens. ROTH spots, are due to immune vasculitis. They are oval retinal hemorrhages with pale centers, they have been noticed in pts with collagen vasculat dis and hemorrhagic disorders. OLSER NODES, violacious nodules founf at the pulp of the fingers and toes, due to immune complex deposition. Immune complex is also responsible for GN and Rehumatolic manifestation of IE. JANEWAY LESIONS, macular,blanching, non painful erythomatous lesions on the palms and soles, they are due to SEPTIC EMBOLI, revealing subcutanous abcesses.****Tricuspid Endocarditis is asso with IV drug abusers. S.aureus is the mc organism. Tricuspid murmurs are accentuated by inspiration and neck vein distention. Echo is the dx choice. Cxr shows peripheral Welll circumsribed lesion with cavitation, Surgery is required in majority of pts. Valve repair or replacement is therapeutic.*****Pt with IE who goes inder GU instrumentation for evaluation of microscopic hematuria could have an exacerbation post procedure with murmur and other symptoms of IE.****Subacute bacterial infective endocarditis (SABIE) is seen n pts with damaged valves. Strep Viridens is the mcc. Acute BIE is caused by S.Aureus in IV drug users. S.Epidermitis is seen in pt with Prosthetic valves. ***Strep Viridans (S. Mutans) are the mc responsible for endocarditis after dental work.****Decision to give prophylactic antibiotis depends on risk due to condition of the pt and also depends on the procedure being done. Risk classication are:1-HIGH risk pt are Prosthertic valves, previous hx of IE, Cyanotic pts. 2-MODERATE risk pt are congenial cadiac abonormmalies Acquired valve dis, MVP and regurgitation, and HCM. Now conditions that DONT REQUIRE prophylaxis are MVP w/o regorgitation, innocent murmurs, Pacemakers and defibrilators. ****IE in IV drug users is in right heart and caused by staph aureus with involvemtn of Tricuspid valve. The holosystolic mumur that intensifies with inspiration is Tricuspid Regurgitation. Vegetations can emboli to remote organs, so if pt have fever and hemoptysis this would be SEPTIC EMBOLI. DDX with Bronchiectasis is that there s a hx of CHRONIC productive cough. DDX of Abscess is foul smell and cavity in Cxr.****Prophylaxis medication guides: 1-Amoxicilline is the DOC in Dental, and Respiratory procedures. In pt with penicilline allergy, Cefazolin, Clindamycin or Clarithromycin is used. 2-In Genitourinary and GI procedures, other than esophageal, the doc is Ampicillin plus Gentamycin. If pt is allergic to penicillin Vancomycin Plus Gentamycin is used****Once you suspect it the next step is to give IV biotics after you draw blood. TEE comes afterwards. Positive blood cultures and vegetatin on the valve seen in TEE confirms dx.****Always suspect IE when a pt is febrile , hx of Rheumatic fever and hematuria. Hematuria in Bacterial Endocarditis is due to glomerlar injury caused by deposition of immune complex. *****If pt has FUNGAL endocarditis then the next step is surgery because they are very aggressive in the valves.

Infertility - 4, OBGYN, 6/2
The first step in WOMEN is to check Basal Body Temperature and mid luteal PROGESTERONE. The ovulatory factor involves defects in the hypo-thalamic pituitary ovarian axis, and related infertility maybe due to impairment of follicular maturation ovulation or endometrial development. BBT assess the DURATION luteal finction and MLP asseses LEVEL of lutal function. Endometrial biposy is done to confirm luteal phase defect. rather than initial evaluation. ****MALES: Male coital factor is responsible for 40% of all cases infertility, common conditions include varicocele, genital tract trauma or surgerydisruption of hypothalamic-pic axis, or Iatrogenic causes like smoking and occupational exposure. The first step in MALE evaluation is sperm count. if its normal then an endocrine hormonal evaluation is carried out. It includes: 1-TFT (since increaed TSH inhibits GnRH and then decrease FSH. 2-Testosterone levels to indicate the presene or not of Gonadism. 3-Gonadotropin to determine whether hypogonadism is central or testicular and 4-Prolactin lelevs. ****Causes of infertility in femlaes falls in 4 factors: 1-Peritoneal factor. 2-Ovulatory. 3-Cervical. 4-tubo-uterine. Peritoneal is the mc type and includes Endometriosis and peritoneal adhesions. Laparoscopy is the procedure of choice. for dx and tx. Mild forms of endometriosis usually respond to meds like GnRH agonists, Danazol and Medroxyprogesterone. 2-Ovulatory factor involves hypothalamus-pit-ovary axis. and infertility might be due to impairment of follicular maturation, ovulation,or endometrial development. ovulatory abnormality may initially be screened by Basal body temp and midluteal phase level of progesterone, the former asseses DURAtion and later LEVEL of luteal function. If luteal phase shows low progesterone, hence infertility, then tx is suppository progesterone deposition. 3-Tubo0uterine is seldom a cause. It onvolves Fibroids, endometrial polyps, tubal occlusion(2ary to IUD or endometriosis). Investigation is ainlt hysterosalpingography or laparoscopy. 4-Cervial involves cervial structure abnormalities and abnormal mucus production. In 5-10% infertility remains unidentified. Intrauterine insemination is the tx. *****Clomiphene Citrate is an antiesterogen that acts by competitively inhibiting esterogen receptors at hypo-thalamus, thus inhibiting the negative feed back esterogen has on GnRH production and consequesntly increasing LH & FSH secretion and improving ovulation. Along with HCG and HMC its indicated for chronic anovulaation. Side effects include large ovaries, hot flashes, abdominal bloating, breast discomfort and abnormal uterine bleeding. Major complications include Ovarian Hyperstimulation Synd and multiple gestations. Danazol is an androgen derivative having a gonadotropin inhibitory effect, indicated in endometriosis, fibroids and fibrocystic breast disease.

Inflamatory Bowel Disease - 2
Erythema nodosum, arthralgias, diarrhea, and positive PANCA ( 60-80% in UC and 10-25% in Crohn) in a young pt are highly suggestive of IBD. ***Any young pt with with bloody diarrhea should make you think of IBD. DDX would be infectious diarrhea, mostly Campylobacter. If pt presents with rectal tenderness and mucus and distended abdomen he might have UC with a fulminant course and Toxic megacolon. Fulminant colitis is a serious comlication, xray shows it. Proctosigmoidoscopy with biopsy establishes Dx.

Influenza
Presnts with, cough,coryza,fever,chills,malaise,sorethroat, muscle pain. Dx is made clinicaly, however a rapid lab test for Influenza antigens srom nasal swap is available. The infection is self limiting b/w 1-7 days. Treat with bed rest and acetaminophen. Two calsses of drugs for prvention and tx are 1-Amantidine (influ-A), 2-Oseltamivir for both A & B.

Influenza Vaccine:
Is recommended in annual basis for all adults over 65 and adults of any age at risk of developing influenza (1-Chronic dis like CV or COPD. 2-Immunocompromised. 3-Nersing home residents. 4-Pregos in 2nd trimeseter in influenza season). This is NOT Influenza B vaccine.

Insulinoma - 2
Pancreatic B-cell tumor. Whipple's triad of attack occurs in fasting, there is hypoglycemia and ingestion of CHO releives the symptom. Tx is surgery. 80-90% are single benign tumor. !0% is malignant.DDX with Sulfunyluria(The sulfonylureas lower plasma glucose primarily by stimulating insulin secretion. SE is hypoglycemia and increased Cpeptide and increased plasma sulfunyluria) and DDX2 is Exogenous insulin admin (normal Cpeptide).

Intellectualization
Helps the pt to be emotinaly detached from the wrong doing (murder) or unacceptable fact (cancer). DDX:Rationalization is a logical reasoning for an upsetting event rather than the true reason (students says they failed me).

Interestitial Lung Diseases
1-Extrinsic Allergic Alveolitis or Hypersensitivity Pneumonitis, due to exposure to organic dust like fungal sporres or actinomyces, Farmer's Lung and Bird Breeders are two examples. Features are fever, sypnea and non productive cough. Cxr shows interstitial infiltrates. PFT shows restrictive pattern (Reduces total lung volume). The best tx is aviodance. 2-Alveolar Proteinosis, accumulation of phospholipid rich material in alveoli. It presents with dyspnea and cough. Cxr shows Bilateral alveolar infiltrate and PFT shows restrictive pattern. Dx is lung bipsy and PAS positive material. Tx is total lung Bronchoalveolar lavage. 3-Acute Interestitial Pneumonia, an acute fatal disorder that rapidly progress to pulmonary fibrosis. It presents in >40 people, fever, breathlessness and cough. Pt has hypoxia and requires ventilation. Cxr shows diffused bilateral alveolar infiltrate. 4-Asbestosis, Its initial presentation may be Obstructive. Presents with Pleural Fibrosis. Its exposed to IN organic dusts.

Intermittant explosive disorde
Is an impulse control disorder. Characterized by multiple episodes of assault resulting from aggressive impulses, out of proportion to any stressor. Its asso with abnormality in serotonergic pathway of limbic system.

Internal carotid a. occlusion
most commonly manifest in ocular disturbances and ischemia in middle cerebral artery territory.

Interossseous access
whenever pediatric iv line cant be found, this is the best place for it.

Intestinal Obstruction:
If hay simple mechanical obstruction then both Barium enema and Naslgastric tube+IV fluids+NPO would be appropriate. But if hay obstruction with metabolic acidosis and shock, then laparotomy is the only way to go and Laparoscopy is CI due to shock & acidosis.

Intestinal perforation:
Best test is standing abdominal xray. Used for PUD rupture. If negative then US, Ctand DPL is indicated.

Intra abdominal bleeding
Once you know the pt is bleeding into the abdomen the next thing to do is either US or Diagnostic Peritoneal Lavage to find out the location of bleeding and then exploratory laparotomy.

Intracranial Pressure:
Increased ICP is indicated by 1-Bilat dilated pupils. 2-Anisocoria, pupils are non reactive to light. 3-Flacidity or decerbrate motor posturing. 4-Papiledema. Glascow is not anindication for increased ICP.

Intraductal Papiloma:
A benign tumor of lactiferous ducts. Clinically manifest as serous bloody discharge. Mamo wont show it, too small. Resection has to be done to relief pain guided by galactogram.

Intra parebchymal hemorhage -2
Due to hypertension, think of Straie arteries that cause bleed in parenchymal in Basal Ganglia.***** HT is the most important risk factor. 1-Cerebellar hemorhage: acts for 16% of cases. Pt presents with Ataxia,vomit,occipital HA, gaze palsy. NO HEMIPARESIS. Emegent decompresion may be life saving. 2-The mc site of hypertensive hemorhage is Putamen, 35%. Internal capsule is always affected thereby leading to HEMIPARESIS. Other signs are hemi-sensory loss, homonomous hemianopsia. stupor and coma. The eyes are deviated form paralytic side. 3-Pontine is accounted for 5-12%. Pt presents with DEEP COMA, and paraplegia that developed w/i minutes. Pupils are pinpoint and reactive to light. No horizontal eye movement.

Intrahepatic cholestasis of Preg
Jaundice in the third trimester of pregnancy should be evaluated for hepatic disorders specific for pregnancy. Marked pruiritis and elevation of bile acids should make you think of ICP. Tx includes Cholestyramine with or w/o Phenobarbital or Ursodeoxycholate. Fetal monitoing is mandatory. DDX is Primary Billiary Cirhosis, intense pruiritis, makredly elevated Alk phosphatase and cholestrol levels. DDX primary Sclerosing Cholangitis is asso with UC, presents with RUQ pain,Jaundice and pruritis.

Intratrochanteric Fracture
Of the femur is mostly seen in elderly fall. The extremity is shortened and rotated. Xray is Dx. Tx is internal fixation with sliding screw and plate and EARLY mobilization.

Intrauterine fetal demise
It’s the death of the fetus in utero that occurs after 20 weeks gestation and before the onset of labor. Its suspected when mother reports the disappearance of fetal movements, decrease or stagnation in the uterus size and no heart tones. The most appropriate to confirm is Real Time US, which will demonstarate the lack of movement and absence of fetal heart activity. After dx is established coagulation profile has to be determined to to detect an eventual DIC, which is a serious complication of IUFD early in the course.

Intra Ventricular Hemorrhage of the new born:
Occurs in low birth weight infants. Its most commonly seen in premature infants. Pt presnts with palor, cyonosis, hypotension, seizures, focal neurologic signs, bulging or tense fontanels. So transfuntanel US is mandatory for all infants with risk factors.

Intussusception
2-yr old child with abrupt onser of abdominal pain,nasea,vomit, red current jelly stool containing blood and mucus. 75% of pts are yonger than 1-yr old, the mc is in Ileiocolic. Sausage like abdominal mass and draws knees towards chest.

Iron Def Anemia - 8. Hemo. 6/3
Fe normal is b/w 50-170. Decreased Fe, increased TIBC (460), MCV=68 and hypochromic microcytic anemia. Infants are at increased risk of this disease due to lack of Fe in milk. [DDX1 is Sideroblastic Anemia, is apart of utilization anemia which is caused by inadequate or abnormal utilization of intracellular Iron for Hb sysnthesis despite increased amount of Iron, sometimes it help to give Pyridoxone (B6)]. IDA is the mcc of anemia, suspect it when a person is not eating well and lab shows microcytic hypochromic anemia. **** In elderly, blood loss from GI will lead to Fe def anemia. Since fecal blood test is NOT sufficient we need to do Colonoscopy because in elderly pt w/o GI complain, the cause is probabely cancer. ****Its the mcc of anemia in children. The early intruduction of Cow milk to infants diet clearly asso with iron def anemia, the larger the amount of milk consumed the higher the risk. Typical lab shows, low MCV, low Hb, Low hematocrit, low reticulocyte count, microcytosis, hypochromia, low serum Iron, elevated TIBC. Low serum feritin is diagnostic for IDA. Infants should receive breast milk for the first year or iron fortified formulas. iron fortified cereal should be added the first 4-6 months. DDX1:Thalasemia, High Ferittin and signs of extramedulary hemopoises (hepato/spleno megaly and widened bones.). *****Microcytic hypochromic anemia due to chronic blood loss (Iron def anemia) results in decresed serum Fe, Ferittin, Transferin Saturation, and increased TIBC. DDX:Sideroblastic naemia has, Increased Fe, noraml Ferritin, Increased TIBC, normal to incresaed Transferin saturation. DDX2 Anemia of Chronic DISEASE will show decreased Fe, increased Feritin, low TIBC and low Transferin saturation. ***The mc type of anema in elderly pts. This pt presents with Fatigue and palor. Another cause of IDA is cheronic diseases like infections, inflamatory dis, neoplasm. ****BMIron stain is the most definite way to dx IDA. Low feritin, increased TIBC, Low Iron are not as specific.


Iron Poisoning
Presents with N&V and Diarrhea and abdominal pain, GI bleeding and Metabolic acidosis. Fe accumulates in mitchochondria and resulting in cellular damage. Hypotensio nthen occurs due to increased vascular permeability and venodialation. Since Iron tablets are radiopaque they will be seen on Xray. Dx is confirmed by serum Fe levels. IV Defroxamine is tx. DDX is Aspirin Tox, presents with lethargy,fever,hyperpnea,vomit, tinnitus and metabolic acidosis. Abdomen Xray is unremarkable.

Ischemic Colitis
MC site is Splenic flexure because its supplied by end arteries, It’s a 'watershed' area b/w superior and inferior mesenteric A.*****Ischemic colitis is due to onstruction of IMA. After AAA repair, diarrhea and blood in stool should raise suspicion. If CT is inconclusive a sigmoidoscopy is recommended. Angiogram is not recommended. Barium enema could cause perforation in case of IC. It’s a good technique for detection of colonic masses.

Isolated Proteinuria
IP (w/o pyuria or Hematuira) can happen due to stress or any febrile illness. Evaluation of pt should begin by testing the urine on at least three occasions (dipstick testing).

Isoniazide Tox
It might cause elevation of AST ALT in the beginning, do nothing it will decline.

ITP - 2
Idiopathic thrombocytopenic Purpura is an autoimmnue disorder characterized by isolated thrombocytopenia absecence of splenomegally, absence of fever or other systemic signs normal BM with normal or incresed Megakaryocytes.. ITP is acute and self limiting in children, but becomes chronic in adults. It presents with skin or mucosal bleeding. Coagulation studes are normal. Autoimmnue destruction of platelets maybe primary or it may be 2ary to SLE or infection with CMV,Toxoplasma, HIV. Whnever pt is having Chronic ITP BM must be exam must be performed to rule out primary hematological disorder. Pt must also be screned for SLE by anti niclear antibody testing, as isolated Thrombocytopenia may be a presenting feature of SLE, especially in a young female. Now, if a pt shows hepatomegaly, lymphadenopathy,or atypical lymphocytes , 2ary causes of autoimmune thrombocytopenia like CMV, HIV, Hepatitis and toxoplasma should be cinsidred. ***The mc acquired bleeding disorder in childen. It follows after viral infection, with easy bruisability,and petechia. Thrombocytopenia and normal PT&PTT. DDX:HUS, combination of Thrombocytopenia,Microangiopatic hemolytic anemia and Acute renal failure.

IUGR - 2
IUGR is defined as birth weight below the 10% for a given gestational age, and refers to fetuses and neonates whose growth potential has been restricted by pathologic processes in utero. These fetusus are particulary prone to problems such as meconium aspirarion, asphyxia, polycythemia, hypoglycemia and mental retardation. IUGR maybe of maternal, , fetal or placental origin. Common maternal causes include poor nutrition, cigarette smoking, drug abuse, alcoholism, cyanotic heart disease, pulmonary insufficiency, and antiphospholipid syndrome. Placentall causes include conditions that result in a lack of decidualization of myoetrial arteries resulting in deficient perfusion of the fetus. Such conditions include Hyper Tension, DM, CRF, and Preeclampsia. Feal causes are TORCH and congenital abnormalies. IUGR maybe symmetrical, the insult to the fetus before 28 week leads to damage to both Head and Body. Its caused by fetal factors. Asymmetrical IUGR is a result of insult after 28 week, only abdominal circumference is small. Its usually caused by maternal factors like DM, Hypertention, Preeclampsia & CRF and it has a better prognosis****The moft effective parameter for estimation of fetal weight in cases of suspected IUGR is Abdominal circumference.****Is defined as birth weight below 10% for a given age. These fetuses are prone to Meconium aspiration, hypoglycemia, and mwntal retardation. Once IUGR is dx fetal well being has to be monitored, with NST and BPP twice weekly. Mother can contribute to this monitoring by assesment of the kick count. Delivery is usually indicated at 34 weeks or when lung is matured. If there is Oligohydramnios, AFI of 4, DELIVERY SHOULD BE STRONGLY CONSIDERED. At birth , neonates with UGR are prone to hypothermia and hypoglycemia. RDS also frequently occurs. So bottomn line is that eventhogh gestational week is 28, go ahead and deliver vaginally.

Jaundice Dx Procedure
Once the pt has jaundice (bilirubin>1, then the next step is to determine if its mostly conjugated or unconjugated. Conjugated hyperbilirubinemia is present when direct bilirubin cinstitues >50% of total bilirubin. Unconjugated Hyperbilirubinemia is present when in-direct bilirubin constitutes >90% of the total bilirubin. Normaly direct bilirubin constitutes <10%>

Jervel-Lang-Neilson
Or Congenital 'Q' synd. Syncopal episode w/o following disorientation, hearing impairent, normal PE, and familt hx os sudden cardicac death is characterictic of this syndrome. Tx is Beta blockers.

Pain Management:
Pt with excruciating pain, even if in rehab for drug abuse, should be getting IV Morphine. Once the bolus morphine is given, the pt can be maintained on PCA (Pt Controlled Analgesics). The advantage is high quality analgesia, Pt and nurse satisfaction and freedom from painful injections. PCA can be used both at home and in the hospital by IV or SC. PCA is used mostly in post operative conscious pt for initiation of parenteral opiod therapy, treatment of incidence pain (trauma) and short term home care intractable pain like cancer pain.

Paget's Dis of Bone - 3
Hx of hearing loss, increased hat size and occasional headahces and elevation of Alkaline phosphatase. Pthophysiology is impaired bone remodeling. Initially increased osteoclastic activity is observed, with following osteoblasts and immature bone remodeling. Eventually, sclerotic bone is formed. Osteoporosis: Bone demineralization. Vit D deficiency: increased osteoid deposition. Primary hyperparathyroidism: Fibrous replacement of the bone. Hypervitaminosis A: Abundant mineralization of the periosteum. *The MCC of Asymptomatic increase in Alkaline Phospatase in an elderly is Pagets, with normal Ca,Phosphorous,normal ALT and AST. *Indication for tx of pts are:1-Bone pain, 2-Hypercalcemia, 3-Neurological defect, 4-High output cardiac failure, 5-preparation for orthopedic surgery, 6-involvement of weight bearing bones. Tx of Asymptomatic pts is not required. Biphosphonates are DOC for Paget's, they inhibit bone resorption(loss of bone tissue which is soaked up by blood), and last for years. Cacitonin also inhibits osteoclastic bone resorption, however its efect on osteoclasts is much weaker than biphosphonates.Tx with Calcium and Vit D has no effect.***PD is characterized by excesive Bone destruction and repair. Its generaly asymptomatic and may be suspected by elevated Alk Phosphatase. Serum Calcium is normal. Overtime 1/3 of pts may develop OSTEOSARCOMAor SACROMATOUS CHAGNGES. This will manifest as Marked bone pain, Lytic lesions and a sudden increase in ALk Phosphate. Bone remodeling is constant in Pagets, both osteoclasts and psteoblast are involved in process but BY ITSELF IS NOT A CAUSE OF PAIN.

Paget's Dis of Breast
Peristent dermatitis of nipple. Red, oozing crusted lesion non responsive to antibiotics or steriods. Biopsy reveals large cells surounded by halo like area invading the epidermis. This disorder is a breast cancer and almost always there is an underlying breast carcinoma present. Most commonly this is an infiltrating Ductal cell carcinome but occasionally ductal carcinoma in situ.

Pancoast Tumor
In a smoker with arm pain, cough and weight loss, mass in the lung apex is PT unless proven otherwise.

Pancreatic Carcinoma -5
Risk factors are: Family hx, Chronic pancreatitis, smoking, DM, Obesity and a diet high in fat. Alcohol is not a risk factor. *****Abdominal US is initial choice in pt with Jaundice. However they are usually not clear so the next step is CT has higher sensitivity for suspected pancreatic cancer. ****Anorexia,weight loss, painles jaundice in a smoker with increased Alkaline phosphatase and abscence of gallstones is suggestive. CT of abdomen is the next best thing, its very sesitive for PTtumor.Then we can do ERCP to get biopsy or refer to a high volume surgery unit to do Whipple’s procedure and remove tumor. Percutaneous Transhepatic Cholangiogram (PTC) is to asses biliary duct not PT. CA-19 is good for PT not for Dx of PT. Amylase is good for Pancreatitis no PT.

Pancreatitis, Acute - 8
Hypertriglyceridemia > 1000 mg/dl can cause AP. If its due Acute Cholesystitis, once the pt is stabalized perform cholesystectomy. Management is conservative, pain med, iv fluid, npo, nasogastric sucction.****Development of palpable mass in epigastrium 4 weeks after theacute pancreatitis is Pseudopancreatic Cyst. They are not true cyst, they lack epithelial lining. Collection of fluids,tissue,and necrotic debris. Amylase rich fluid leaks out so Amylase is increased. USG is dx of choice. THey should be drained ONLY if they persist after 6 weeks and are >5cm.*****Serum Lipase and Amylase is the first step in confirming dx and the most sensitive and specific test for AP, not ERCP(its used for CBD and Chronic pancreatitis). CT is used only if the pt is not responsing to management and we need to see if there are any complications. *****The 2 mcc are Alcohol and gallstones. If pt isnt alcohoic then cause is Gallstones. So the next step is to do US to llok for stones. CT cant help with Gallstones, but its good for necrosis and Cysts. CT is indicated when pt has WBC>20000 and Necrotizing pancreatitis is suspected. HIDA is for calculus or acalculus Cholecystitis. ****Cyst+Fever = Abscess. In AP the abscess needs to be drained EXTERNALY. Abscess needs to be drained, Pseudocyst can be managed conservatively .****In severe pancreatitis and 2ary paralytic ileus , naturalization of acidic pH with antacids should be considred, for severly illpts, to prevent gastric stress erosions.

Pancreatitis, Chronic - 5
Admit to hospital is pt has weight loss, pain, steatorrhea (malabsorption, best test to confirm it is 72hr fat in stool collection, if >7g/d its positive), malnutrition. CT to rule out pseudocysts and dialatation of the duct, when duct is dialated a cm the best tx is lateral pancreaticojejunostomy. This relieves the pain and improves nutrition.****PseudopancreaticCysts: are untouched for 6 weeks if they are <5cm. If it doesnt ersolve then either do percutaneous drainage or gastrostomy.****Surgical intervention is needed in case of chronic pseudocyst and debiliating pain (due to compaction of stones in pancreatic duct). Tx is ERCP with removal of stone and stent insertion. Partial panreatectomy is the last result, if ERCP didint work.**** Pt with recurrent pancreatitis after US is done should go under ERCP and BYPASS CT because CT is not good for pancreatic and billiary bile ducts and would miss microlithiasis.*****Lateral Pncreaticojejunectomy can provide relief for chronic pain.

Panic Disorder - 3
Women 20-40. Sense of IMPENDING DOOM. Acute tx is Benzodiazepine, takes effect immediately. For long term tx use SSRI or TCA with conginitve therapy or behavior. ALSO according to Valia, Panic attack is asso with mitral valve ProlaPs.***Of the two drugs for PA, Alprozolam has a Seisure attack due to withdrawl that happens ina few days of not taking the medication. But Diazepam, does not give you seisure that quckly. So if pt comes with Seizure and didnt take panic drug for a few days Alprozolam is the drug.****Pts suffering from Panic dis are at increased risk for DEPRESSION, agorophobia, GAD and substance abuse.

Paranoid Personality:
Cluster A (odd, eccentric), distrust and suspicious to others. Negatively interpret actions and words. Hold grudges for long time.

Paralytic Ileus
Absent bowel sounds with gaseous distention of both small and large bowels are indicative.

Paraneoplastic Syndromes
Hypercalcemia is the mcc of PNS (confusion, lethargy, fatigue, anorexia, constipation, polyuria) it is asso with Squamous cell carcinoma of the lung . The classical pathophys responsible for hypercalcemia of the malignancy is the production of PTH related peptide (PTHrP), which is homologous with PTH. Serum Ca is Increased and Phosphate is decreased *****Myopathy in asso with PS: PS can manifest as muscular weakness. Proximal muscles are typically affected and strength is diminished symmetrically. Reflexes are normal and no sensation abnormality is detected. Elevated Ck and myopathic EMG help to confirm Dx. Pathology in PS is in the muscle membrane, in MG is in Postsynaptic membrane, and in Lambert-Eaton is in the Presynaptic membrane.

Parenchymal Brain Hemorrhge
Always susprct it whn pt has hx of uncontrolled HT and then prewtns to ER unconscious with focal neuro signs.

Parinaud's Synd
Consists of paralysis of vetical gaze that maybe asso with pupilary disturbances and eyelid retraction (Collier sign). Most commonly caused by Pinealoma. Pt ( 10 yo ) prestns with facial hair and 2ary sex characteristics, plus vertical gaze and Collier sign. DDX is Craniopharyngioma that does not cause the Collier sign.

Parkinson's - 2
The most effective symptomatic therapy is L-Dopa.*** First drug to use for tremor and rigidity is Benztropine.*****asso with Seborrheic dermatitis.

Parox Supra vent tachycardia
Test-1, Q-43. See picture. DOC is IV Adenosine

Paroximal Nocturnal Hem.- 2
PNH. A red cell membrane defect causes increasing binding of complement to the red cell leading to increased intravascular hemolysis resulting marked anemia. Since RBC is more suseptable to hemolysisi in acidic environment and serum is more acidic at night, lysis occurs at night and morning urine shows red urine. Thrombosis of Hepatic vein is a cc of death in these pts. Lab shows increased LDH, bilirubin and reticulocyte. Test specific to PNH is sugar-water test and the Acidified Hemolysis test (HAM test), which determines increased suseptibility to cell lysis by complement . DAF (decay Activating factor) is diminished in PNH. DDX1:RBC enzyme def (G6PD def) results in suden hemolysis after drugs or infection. DDX2:Peripheral destruction of RBC is seen in Hereditary Spherocytosis.****Is an acquired disorder that should be considered in ALL CONFUSING cases of hemolytic anemia. emolysis is intravascular with low serum haptoglobin, and elevated LDH. Loss of Ironin urin may lead to iron def anemmia. Congestive splenomegaly is a complication. BM maybe hypocellular. Flow cytometry is a better choice for dx vs Ham test, it shows absence of CD59. DDX1:Aplastic Anemia,BM is hypocelllular, but peripheral blood smear does NOT show abnormal cells. RBC are normo or macrocytic.There is NO hemolysis or Slenomehaly. SUGER test is for dx of PNH.

Parvo Virus infection - 2
Parvo virus infenction involves joint. , its symetrical, hands, wrist, knee and feet are mostly affected. Rash may or may not preent. Pt has Arthralgias or arthritis, mc in femlaes. Pt presents with pain in small joints, arthralgia and arthritis. **** School teacher gets it. Anti-B19 IgMis the dx of choice.

Passive aggressive behavior
A mature defense which an individual expresses aggression toward other person with repeated , passive failure to meet theother person's demand. A secretory wont let you talk to the doctor because she had an argument with you.DDX is acting out. A child throws a temper tantrum.

Patient consent
If pt needs transfusion and wife says its agianst his believes, go ahead with transusion. Now if Pt himself is awake and tells you that he dosnt want transfusion then you use other alternatives, Plasma.

PCP toxicity –3. Poison 6/3
Characterized by violence, vertical nystagmus, confusion, ataxia. DDXCOCAINe causes tachycardia, ht, sweeating, mydriais.****LSD is like PCP but agitation and agression is more in PCP. Visual Hallucination (Flash backs) are hallmarks of LSD.

Pediatric CNS tumors:
Infratentorial tumors are more common. And benign astrocytomas are the mcc type. CNS tumors are the mc solid tumors & 2nd mc malignancy after Leukemias. Meduloblastoma is the 2nd mc tumor in posterior fossa, 90% arise from Vermis.

Pediatric Immunology:
1-Hyper IgM syndrome: Characterized by high levels of IgM, w deficiency of IgG and IgA and poor specific response to immunization. HIM presents with recurrent sinopulmonary infections and PCP. The unique suseptability to opportunictic infections and neutropenia DDX it from XLA and other conditions.
2-X-link Agamaglobinemia or Bruton’s: characterized by 4 findings, a-onset of recurrenct bacterial infections in 1st 5 years of life, b-IgG, IgM and IgA are very low, c-poor response to vaccination, d- <2% CD19+B cells in peripheral circulations.
3-Common Variable ID: presents with low IgG, IgM and IgA levels, but normal B cells. Clinically they present with recurrent sinopulmonary infections like XLA. Most pts don’t become symptomatic until 15-35 years.
4-Selective IgA deficiency: is the mc well defined ID disorder. Infections occu predominantly in respiratory, GI and urogenital tract. Serum concentrations of other IG are normal. Dx is IgA <50 with no other immune defects.
5-Selective subclass IgG def: Its seen in pts with recurrent infections despite normal IgG levels.
6-Severe Combined ID: is a life threatening synd. Presents w recurrent sinopulmonary infection, oral candiasis, persistant diarrhea, opportunistic and viral infections. Dx is confirmed by absent lymph nodes and tonsils, lyphopenia, absent thymic shadowon Cxr. And abnormal B & T cells.
7-Wiskot Aldrich Synd: is an X-linked recessive dis, caused by a defective gene encoding for WAS protein. Classic presentation is young boy with eczema, thrombocytopenia, and recurrent infections w encapsulated germs. Initial manif is at birth and consist of petechia, bruises, bleeding form circumcision, or bloodt stools. Low IgM levels, high IgA and IgE levels reduced T cells and platelets.
8-Chronic Granulomatous Disease: a defect of phagocytic cells, due to dsfunction of NADPH oxidase enzyme complex, leading to recurrent and uncontrolled infections w catalse positive organisms (Staph, Seratia, Klebsiela, Aspergillus) and not susceptible to catalase negative (Strep and H Influenza). The mc infections are lymphadenitis and abscesses of skin and visera. Lymphocytes are normal. Dx is by Nitro blue tetrazolium test. Tx is prevention with DAILY Trimeta-Sulfa and Gamma Interferon three times a week BMT is Curative.
9-Transient Hypogamaglobinemia: is characterized by decreased IgG levels, normal IgA levels, and variable IgM levels. These Igs normalize by 6-11 month of age.
10-Di-George Syndrome: Typical of an infant with hypocalcemic seizures. Pathology is defect of 3&4th pharyngeal pouch leading to hypoplastic or aplastic thymus and parathyroid glands.
11-Chediak Higashi Syndrome: characterized by decreased granulation, chemotaxis. It’s a multisystem disorder , peripheral and central neuropathy, hepatospleenomegaly, pancytopenia, partial albinism, infections w Aureus. The fiding of neutropenia and giant lysosomes in neutrophils will confirm Dx. Tx is prevention with Tri-Sulfa daily and daily ascorbic acid.
12-Leukocyte Adhesion Defect: result from failure of innate host defenses against bacteria, fungi due to adhesion targeting. Pt has hx of delayed seperation of umbilical cord, recurrent bacterial infection, necrotic skin lesions, severe gingivitis.
13-Job’s Syndrome: or Hyper IgE syndrome. Characterized by chronic pruritic dermatitis, recurrent Staph infections (resp and skin), markedly elevated IgE, eosinophilia and coarse facial features.

Pediatric Jaundice:
1-1st 24 hrs: requires immediate attention, maybe due to erythroblastosis fetalis, concealed hemorrhage, sepsis or infection.
2-2nd & 3rd day: is usually physiologic.
3-3rd day to one week: suggets bacterial sepsis or UTI and requires prompt and aggressive attention. Sepsis in neonate rarely causes neck stiffness or boging frontal or shock. The clues are poor feeding, lethargy, womiting, or alteration of activity. The first step is to do a blood culture and a lumbar puncture.

Pediatric Meningitis:
1-Group B Strep: mcc of meningitis in infants. Usually acquired form mother duting childbirth. Its not asso w rash and very unlikely for a 18mo child. 2-Meningococemia, suspect it in a neonate with petechial rash. 75% present with rash in trunk, wrist, ankle. 3-H. Influenza, 6-48mo, no rash involved. Antigen is in CSF. 4-Listeria can cause meningitis in newborn vaginally, it doesn’t cause rash. Anotehr mode of transmission is from unpasterized milk. Mc in DM mothers. 5-Aseptic; Always due to viral cause. CSF shows, lymphocytosis, WBC<250,>18yo S. Pneumonia is the mcc 60-70%.

Pediatric Rheumatic fever:
Suspect it in children with low grade fever, pericarditis, sore throat, arthritis, chorea, and subcutaneous nodules. Erythema marginatum, elevated ESR and prolonged PR. Its caused by GroupA Strep. Dx is made w two major or two minor and one major Jones Criteria are satisfied. Five major criteria PECCS (Polyarthritis, Erythema marginatum, Carditis, Chorea, Subcutaneous nodules) and three minor FAR (Fever, Arthralgia and Previous RF). Tx is Benzathin PenG. . If chorea give antiepileptic, Pericarditis give salicylates and Arthritis give codeine.

PEEP - 2
Positive End Expiratory Pressure, refers to develpoment of positiv epressure inside the thorax resulting in decreasing venous return. Decreased venous return leads to decreased cardiac output, which ultimately leads to hypotention. The increased ventilate rate causes development of auto PEEP by not allowing enough time for expiration. Thus, the ventilator delivers a tidal volume, in addition to that is still in the lung at the end of the expiration, which is already inside the lung. This produces positive prssure. The most appropriate step when Hypotension happens is to decrease the respiratory rate, to allow the lung to deflate in b/w respirations.****When PEEP is increased its major drawback is decreased CO. Pts who are maintained on PEEP should be monitored with Swan Ganz cathater. PEEP has no direct effect on Sytolic BP or Systemic Vascular Resistance. ****If Tidal Volume is too high, 800, reduce Pressure support and increase ventilation rate to not to increase pCO2.

Peitonsilar abscess - 2
Fever, sore throat, dysphagia, trismus, pooling of saliva and muffled voice. Management includes needle drainage(trendelenberg position) close monitoring and IV antibiotics.

Pellagra
4Ds.*****Most commonly in people with corn diet. Also in Alcoholics and people with Carcinoid tumor. Characterized by 4Ds: 1-Diarrhea, Dementia, Dermatology (skin rash in sunexposed areas), Death.

Pelvic Abscess
Percutanous drainage is the procedure of choice. If it fails then Laparotomy with debridment and drainage is indicated.

Pelvic Trauma:
Xray should be doen in ALL pt with trauma to pelvic to screen for pelvic injury. Not CT.

Pemphigus Vulgaris. Dermo. 6/3
Mucocutaneous blistering with flaccid bullae, with IgG deposit in intracellular in dermis. Autoantibodies against desmogelin. First appears in Oral mucosa. Easy seperation of edidermis on superficial pressure (Positive Nikolsky sign) is characteristic. For tx use steroids. DDX:Bulous Pemphigoid, tense blisters, IgG and C3 in dermal-epidermal junction.

Penile Fracture:
From women on top. Surgical exploration is the tx of choice but it should always preceded by retrograde urethtogram to rule out urethral injury

Peptic Ulcer Dis - 3
If perforation occurs the first step is Upright abdomnal Xray. Now although the symptoms might suggest that pregnany is a DDX, do xray first to rull out the most dangerous possibility. ***** PUD can lead to gastric ouotlet obstruction. , unceasing vomiting and dehydration resulting in Hypocholeremic Met Alkolosis. Corrected with 0.9%NaCl. This kind of Alkolosis is usually accompanied with Hypokalemia. POTASIUM supplementation is indicated whenever NG suction is done.SO tx is Potasium Chloride.*****MC comlication is Hemorrhage.***PUD causes sudden inset of severe epigastric pain, spreads to the whole abdomen, and upright films show air under diaphram. DDX is Cholecystitis and Diverticulitis, which wont have such sudden onset and short clinical course. DDX Biliary Cholic will give more localized pain w/o rebound.

Pericardial cyst:
Diagnosed with CT. Located at the middle mediastenum and are benign. The fluid can be aspirated and the cyst will shrink. Surgery is NOT indicated. Other medial mediastinum masses are Bronchogenic cyst, lymphoma, and aortic aneurysm.

Pericardial Tempnade
Preents with Beck's triad, Hypotension, elevated JVD and muffled heart sounds. DDX: In case of hypotension due to bleeding, jvd will be decressed compare to PT or Tension Pneumothoras. In TP mediastinum is deviated, but in PT its not. ****Pulsus paradoxus is a dx clue.

Pericarditis, Acute
When combined with renal failure is Nephritic Synd that leads to Uremic pericarditis, which is an indication for Hemo dialysis. If vascular access is not possible then we do Peritoneal dialysis. *****Severe constant pain that localized to anterior chest, radiates to arm, back, shoulder, epigastrium, is intensified with respiration, releived by sitting up or leaning forward. MCC by Coxackie virus. A hx of current viral illness is common. EKG shows, 1-Diffuse ST elevation with cncavity at "J" point, 2-PR elevation. Uremia can cause Pericarditis and this may represent an indication for dialysis. *****Uremic Pericarditis: Its seen in 10% of chronic renal failure pts. UP is an absolute indication for Dialysis. Pericarial fluid in UP is most often Hemorrhagic. Clinical features are the same as Pericarditis, but BUN is >60and there is Anemia. Most pts responde rapidly to dialysis with resolution of chest pain. as well as decrease in size of pericardial effusion. Indications of Dialysis are :1-ABSOLUTE indications are fluid overload not responsive to tx, HyperKalema not responsive to tx, UP, Refractory metabolic acidosis. 2-RELATIVE indications are GFR<10, Serum Creatinin <8, severe uremic symptoms (Seizure and coma are indication of immediate dialysis). 3-CI to dialysis are Debiliating chronic dis, and severe irreversible dementia. Remember NSAID (and sometims corticosteriods) are 1st line tx for Pericarditis. Also for Pericardial Temponade Pericardiosenthesis is indicted. *****Once you dx PC the next sep is to start Hemodialysis not NSAIDS. Indications for dialysis are: 1-refractory hyperkalemia, 2-Volume overload and pulmonary edema not responding to diuretics. 3-Refractory metabolic acidosis (pH<7.2). 4-Uremic pericarditis, 5-Uremic encephalopathy or neuropathy, 6-Coagulopathy due to renal failure.

Peripheral Artery Aneurysm:
Manifests as a pulsatile mass that can compress adjacent structure (nerve, vein) and can result in thrombosis or ischemia. Popliteal and femoral aneurysms are the mc PAA. Frequently asso with AAA. Pt has all the atherosclerotic risk factors.

Peripheral Artery Disease
Goal of therapy for occlusive arterial dis of lower extremity is to relieve pain, prevent limb loss and maintina gait. Most pts
with Intermittent claudication remain stable with conservative management. Majority should be given exercise and aspirin. Over 75% remain stable. Angiogram is not required if pt has no evidence of ischemia, infection, gangrene or loss of pulse. If surgery is not planned a duplex arterial study may suffice.

Peritonitis, primary
Abdominal pain with guarding and fever in a pt with ascites is suspicous. Most likely pt has peritonitis secondary to paralytic ileus. If pt has signs of peritoneal fluid (shifting dillness) then diagnostic Parecenthesis is the simplest rest to perform. It may reveal signs of Exudate: Turbid fluid, >250 Leukocytes, Protein >2.5, Serum to Ascitic albumin gradient <1.1,>1.016.

Peritonsilar Abscess ( Quinsy ):
Unilateral sore throat, neck pain, refered ear pain, dysphagia with swollen tonsils, Trismus is characteristic. *****It is a supporative complication of Acute Tonsilitis. Senario involves a pt with sore throat, dysphagia, trismus, pooling of saliva and muffled voice. Uilateral and vervical lymphadenitis and deviation of uvula. Management includes needle drainage (in Trendelberg position), close monitoring and IV antibiotics.

Pernicious Anemia
PA is the mc megaloblastic anemia. B12 def is caused by intrinsic factir def 2ary to gastric atrophy. There is also basophilic stippling. Dx is with achlorhydria, dec IF, dec B12, extremely elevated LDH. ***DDX is with Lead poisening since they both have basophilic stippling. Lead poisening presents like Iron def anemia, MCV<80.

Pesticide poisoning
First step in mgmt is to remove the source of poison ( if pt vomited the poison remove cloths to remove source os poison).

Pharyngitis, Strep
A 7 yo with fever,erhythema of pharynx has Pharyngitis, if Rapid dx test for Strep is positive the next step is to give him One IM dose of Bezatin PenG (which is IM), not IV aqueous crystalite PenG. Or 10d oral Pen V (which is Oral). If allergy then Erythromycin is used.

Phencyclidine intoxication
Causes psychotic symptoms.

Phenelzin CI
It’s a MAO inhibitor and its use is CI with SSRI (Paroxetine). Its called serotonin synd and presents ith hyperthermia,rigidity and altered mental status. Its given to Depresed pts with SIGECAPS. Another SE is Hypertensive Crisis that occurs with wine and chocolate.

Phenytoin SE
causes Folate def. (megaloblastic anemia)

Pheochromocytoma - 2
Always give alpha blocker first, followed by beta blocker. If you do it the wrong way you can precipitate a very dangerous increase in BP. ***Sudden attack of palpitation, HA, prophuse sweating.

Physiologic jaundice
Is seen in ALL new borns. Begins on the 2-3 day and resolves w/I the first week. Pathophys is increased billirubin production, decreased billirubin, clearance and increased enterohepatic circulation. DDX:Naeonatal Sepsis, jaundice after the 5th day and w/i one week. DDX:Breast milk , jaundice that starts afte the first week. Cessation of breast milk and replacement with formula will reverse it. *****Jaundice after 5th day and w.i 1st week is Neonatal Sepsis. Jaundice notices after the 1st week is Breast milk jaundice. Jaundice w increased at birth or w/I 24hrs is due to Erythroblastosis fetalis.

Pick's Dis
Fronto-Temporal dementai. Personality change (disinhibition, apathy, euphoria), compulsive behavior(peculiar eating habits, hyperoraliry) and impaired memory. May be positve famly Hx. DDX1:LEWY BODIES:fluctuating cognitive impairment. Bizzr visual hallucinations may occur. Parkinsonism may occur. DDX2:MULTI-INFARCT:congnitive dysfunction with motor and sensory dysfunction. DDX3:NEUROSYPHLIS:Pt maybe psychotic and personality change but NO family Hx..

PID - 2
Is suggested by triad of abdominal pain, uterine and adnexal tenderness, and cervical motion tenderness. Other criteria are fever, cerviac/vaginal mucopurulent discharge. Criteria for hospitalization include: pregnancy, outpatient treatment not responding, noncompliance, severe illness and abscess. A common regimen for severe PID is Cefotetan (2gr IV every 12hrs), or Ceftriaxone (2gr IV every 6 hrs)+ Doxycycline (100mg PO or IV every 12hrs). ***** severe PID presents with nausea nd vomiting and high fever (103), so admit pt. Mild PID just single IM Ceftriaxone and 7 day PO doxycycline or Azithromycin.

PKU
Auto rec. In fair skin,blue eye, rash, musky oddor pts with vomiting in a month old neonate. Dx is Guthri test , a qualitative test for colr, shows metabolites of phenulalanin in the urine. Tx is low phenulalanin, low protein diet.

Placenta Previa - 2
PP presents with painless vaginal bleeding in the 3rd trimester. Dx is with US, accuracy of 90% transabdominally and 100% transvaginally. Pelvic exam is CI. Risk factors are: multi parity, advanced age, multiple gestation. If the bleeding continues, C-section has to be done ASAP. Even if pregnancy is not term yet. In cases of extended bleeding surgeon might discover A Placenta Acerta. Now if the mother is stable and fetus is at term, scheduled C is choice. However, until then pt has to be monitored closely. If both mother and fetus are stable IM steriod are used to mature the lungs. Forcepts are not used in complete PP.*** In any case Vaginal delivery is not an option for any kind of PP. If both mothe and fetus are OK and there is no more bleeding and mother has access to close hospital then she can be sent home and monitored at home until 36week, then schedule c-section is done.***** Pt with complete PP, after bleedng is controlled and both baby and mohter are fine, Schedule Elective C-section.

Pleural Effusion
When the lung is dull, Cxr shows half of lung is white its due to effusion, too much fluid in pleural space causes collapse of partial lung. , the first step is to find the cause , so managmnt starts with whether its exudate or transudate. Diagnostic Thoracocenthesis is the preliminary procedure of choice, EXCEPT in CHF pts where a diuretic is used. Thoracocenthesis is a bed side, minimal invasive. If its Exudate furhter procedure is followed. Learn what causes transuddate vs exudate. (ie Cancer causes exudate). Now Bronchoscopy is an alternative, but since its very invasice TC is donr first, if its inconclusive then next step is Bronchoscopy. *****MCC of PE is CHF. Transudate effusion. The important diagnostic criteria for separating pleural fluid into transudate and exudates are : measurement of serum and pleural fluid protein & LDH levels. At least one must be present for exudates. If non is present its Trans. Pleural Fluid Protein/ Serum is >0.5, PF LDH/S LDH >0.6. Determination of pH is important in parapneumatic effusions in which a value of <7.2 requires a chest tube aspiration to prevent empyema. Normal PF is 7.46. Pleural pH <7.3 is inflammation. PH of 7.35 is transudate PE.

PMS
A menstural diary for at least 3 months is a useful aid for confirming PMS. PMS is confirmed when one or severl particular symptoms occur repeadedly at the same cylce interval. Like anxiety, mood swing, bloating,decreased libido, difficulty concentrating. Tx depends on the pt and problems. Decrese caffein may help with anxierty, SSRI is the DOC for mood change

Pneumomediastinum:
May accompany spontaneous pneumothorax: the air from ruptured alveoli or bulla dissects along the vessel into the hilum and mediastinum. Pneumomediastinum asso w SP usually respondsd to chest tube drainage and doesn’t need surgical decompression, unless its very massive and causes cardio-vascular compromise.

Pneumococcus Vaccine:
Recommended for 1-adults >65. 2-Chronic dis like CV and COPD. 3-Immunocompromised and DM. Chronic alcoholics. A one time booster is given 5 years after primary vaccine in elderly.*****It’s a T cell independent B cell response mechanism.

Pneumonia - 10
1- Community acquired(mcc Strep P. in nursing homes). 2-Legionella: Any elderly pt w pneumonia, abdominal pain, confusion and hyponatremia should be suspected. Also if beta lactam antibiotics fail. The definite dx is ELISA and tx is hig doze Erythromycin or Azithromycin. 3-Nosocomal : Pseudomona is the mc considered G- aerobic bacili in ddx of G- infections and is a cc of G- Nosocomal pneumonia.Its seen in mechanically ventilated pt and intubated pts. Tx is IV 4th generation cephalos like Cefepime or Ceftazidime. The new generation Quinolones (Levofloxacine and gatifloxacine) is being used for inpatient Community Acquired Pneumonia, for outpatient Azithromycin or doxycycline are used.**** Klebciella is an encapsulated gram negatie bacili can cause pneumonia in Alcoholics (3-4 ounces daily). Friedlander's penumonia affects UPPER lobe and presents with fever, productive cough and copious CURRANT jelly sputum. Klebsiella grows in MUCOID colonies.****Pneumonia pt present with fever, sweats and productive sputum, cxr shows consolidation. Antibiotics are best given IV. Two weeks after agressive tx of a consolidation you have to ensure the pt that there is no bronchhial obstruction by a tumor and that lung abscess has not developed. If pt has a tumor it might block the bronchus resulting in failure of antibiotic therapy. Thus if a pt fails to resolve after two of antibiotics, a Bronchoscopy is indicated. It will help obtain cultures, visualize the brunchus, and do a drainge. CT is indicated if a mass or abscess in llung is indicated. CT will also help to see if an Empyema has setin. Empyema at early stages can be tx with antibiotics and drainage. Abscess requires antibiotics for 4-6 weeks. Do CT first and then Bronchoscopy.****1-Mycoplasma:is common in teens. mostly it causes Pharyngitis or bronchtis only. in few cases hay fever,cough,chills, sorethroat, rhinorrhea. cxr appears worst. Erythema Multiform is one of the extrapulmonary signs, characterized by TARGET shaped lesions. It does not stain because has no cell wall. 2-Pneumococcus is rusty sputum. Gram stain reveals lancet shaped diplococci. 3-H. Influenza occurs in children, vacination prevents it. URT invovment is unlikely. It shows Gram negative staining. 4-Legionella occurs in smokers and alcoholics, water sourse. may be diarrhea. Gram stain fails to reveal it.****All pt suspected of bacterial pneumonia should have a cxr doen as first step. ****Paraneumonic effusions occur in penumonia. The fluid maybe steril in initial stages and resovlve with antibiotic therapy, however in a few cases the pt will continue tohave elevatred twmperatures, pleuritic chest pain and dypnea. CT will identify the fluid collection. The fluid may be aspirated to see is Empyema has set in. The fluid should be analysed for protein content, gam stain, cell count, cytology(rule out malignancy) and glucose level (low in RA, TB, Empyema, malignancy, esophageal rupture). pH should be assessed, Low pH (<7.2) ALWAYS indicative of Empyema and indictes removal of fluid by thoracostomy. Glucose of <60 is ALWAYS indicative of Tube thoracostomy. So the lab test most helpful in placing tube in parapenumonic effusion is Pleural fluid pH.

Pneumothorax - 5
Pneumo Mediastinum may accompany Spontaneous Pneumothorax: the air from ruptured alveoli in the hila and mediastinum. Pt may present with facial subcutaneous emphysema, breath sounds are absent on the affected side and crushing sounds are heard over pericardium. It usually responds to chest tube drainge and doesn not need surgical decompression. Now PM along with Mediastinitis would need surgical drainage of mediastinum. **** TENSION P: This is an EMERGENCY, once you suspect it dont even do CXR the FIRST thing to do is needle thoracotomy. Its due to air in pleural space, it creates a one way valve that allows only entery of the air and no exit. Pt prestns with Severe chest pain, shortness of breath, tachycardia,tachypnea, deviation of trachea, JVD, absence of breath sounds. TP is a clinical dx so dont wait for cxr needle it.****Spontaneous Pneumothorax occurs when there is no provoking factor. When you see a tall thin male think of this. Death from it very rare. Dx is best made with Cxr. ****Tension Penumothorax: after head on accicent and hemothorax. FIRST thing to do is not oxygen mask (it doent hep ), it is CHEST TUBE. ****Spontanous Penumothorax which occurs in young males is almost always due to rupture of SUBPLEURAL BLEBS. Cxr shows deviation. ****Tension P is a possible complication of mechanical ventilation when high PEEP is applied. The barotrauma causes rupture of the lung parnchyma and leaks air into pleual space. Rapid collaps of lung leads to RV filling failure. This results in Hypotension. Remember in Spontaneous P where there is shiftng of trachea, the main prblem is with Oxygenation and Cardiovascular problem becomes secondary not like Tension P which CV is primary.*****In Tension P. even if the Pt presents with severe hypotension (50, 0) the cause is still TP not Cardiac Temponade. Remember Hypotension+shortness of breath+JVD+Decreased breath sounds. And if you do thoracocenthesis and hypotension is resolved then it wasn’t Temponade. ****DDX b/w TP & Hemothorax?

Polycystic Ovarian Dis - 2
Insulin-resistant hyperinsulinemia is a common finding in patients with PCOD. PCOD is an important risk factor for development of DM-2 in women. About 10% of diabetes in premenstural women is POS related. Thus a glucose tolerance test is needed in all POS pts. A two hour test with > 140 is Dx and needs furthur action. Lifge style modification and Metformin is then indicated. Metformin helps in following ways: 1-prevents DM, 2-Causes anorexia and prevents obesity, 3-corrects hirsutism mildly, 4-menstural irregularity and infertility. Clomiphene Citrate induces ovulation and is used for infertility.*****This condition should be suspected in any pt who has mentrural irregularities, and evidence of Hyperandrogenism(facial hair). Presence f these establishes the dx.50% of pts are obeseand are at risk of DM-II. The next step is to do Oral glulcose tolerance test as mentioned above. *****Characterized by unbalanced Estrogen secretion resulting in Endometrial Hyperplasia.

Polycythemia Vera - 3
A12-**** Persistently elevated Hct and spleeno and hepato megally is highly suggestive of PCV. Pts are at risk of hepativ vein occusion (Budd Chiari synd) which is characterized by tender congestive hepato megally with ascites. Hepatic vein thrombosis in these pts is due to hypercoagulability. In cases of tender congestive hepatomegally the first step is to rule out right sided heart failure. If RS heart filaure and Constrictive pweicarditis are ruled out then the next best step is to do a Hepatic Venogram or biopsy.****Its distnguished from pther kind of Myeloproliferative dis by the marked increase inRBC mass and total blood volume. A typical pt is an old Plethoric male who complains of prurites after bathing. Both thrombosis and bleeding can occur due to elevated platelet count and impaired platelet funciton . They have granulocytosis, thrombocytosis and splenomegaly. BM is always hypercellular. There is an elevated Leukocyte alkaline phosphatase, normal oxygen saturation andlow erythropoetin level. A urinary erythropoetin assay reveals of ABSENCE OF MEASURABLE ERYTHROPOETIN in urine. ESR will be low in PCV.

Polymyalgia Rheumitica
Tx is Low dose prednisone. Pt has STIFFNESS rahter than pain. Morning stiffnes last >30minutes, for more than a month and include shoulder,hip, neck and torso. Pt says "Doc my should hip and neck are stiff". ESRis >40.

Polymyositis
An inflmmatory muscle dis of unknown etiology. Presens as proximal weakness, characterized by difficulty Ascending and Descending stairs, combing hair and kneeling down. There are no Skin rach or pathces like Dermatomyositis. These pt has dysphagia due to weaknes of straied muscle . MUscle biposy is the best Dx study. It shows ENDOMYSIAL infiltration.

Polyps
Classified as 1-Hyperplastic: the mc non-neoplastic. No follow up work is needed. 2-Hamartomatous: Include Juvenile polyp and Peutz Jeghers(non-malignant lesion). 3-Adenoma: the mc polyp found in colon. Presents in 30-50% of elderly. Only >1% become malignant. Most are asymptomatic, <5% have positive occult stool test. Probability of adenoma becoming malignant is judged clinically according to size histology and appearnce. A-Adenoma can be sessile or stalked (pedunculated). Cancer is usually in sessile.
B-Histologically, adenoma are villus, tubular and tubulovillus. Villus, which are sessile, are more likely to be malignant. 2nd is tubulovillus and 3rd is tubule.

Porcelaine Gallblader
A13-

Porphyria Cutanea Tarda
Pinless blisters, Hypertrichosis and hyperpigmentation, and increased skin fragility of hands. Asso with HepC. It can be trigered by ingestion of OCP(Etrogen) or Etanol. Elevated urinary porphyria confirms Dx. Phlebotomy or Hydroxychloroquine provides releif.

Post-Exposure Prophylaxis:
For Chicke Pox can be provided with VZIG ro acyclovir. VZIG is preferred and indicated in high risk people exposed w/I 72 hours of exposure. If its longer than that its tool late and the child will get it, warn the mother for the rash

Post traumatic Dis Synd
PTDS and Acute Stress Disorder are identical except that ASD cant last more than 4 weeks and PTDS lasts for months.***AVIOD Benzodiazpine in these pts since PTDS is asso with substance abuse and Benzos are addictive, tx is best with SSRI and exposureor cognitive therapy.

Posterm pregnancy - 2
Is pregnancy age > 42 weeks resulting in macrosomia. Assos syndromes are anencephaly and trisomy 18. Mgmt depends on well being of the baby. Follow biweekly with non-stress test and BPP, if baby is in danger(oligohydamniotics or spontaneous deceleration) deliver. If not labor should not be induces if cervix is not favorable. If the term is .34 weeks then delivery is mandated. If baby is >42 weeks and cervix is flavorable then deliver.

Post operative Endophthalmitis
is the mc endophthamitis. It occurs w/I 6 weeks of surgery. Pt has pain and decreased visual acuity. Conjuntiva & eye lids are swollen. Send Viterous for culture, and based on severity intravitreal antibiotic injection or vitrectomy is performed.

Post strepGN - 2
Its seen 10-20 days after throat or skin infection. Features include periorbital swelling, hematuria and oliguria. Pt may show hematuria with RBC Cast and proteinuria. Serum C3 complement are low. DDXrug induced intertitial Nephritis, occurs with penicillines, cephalos, sulfas. Pt prestns with fever,rash,arthralgia, esosiniphilia, hematuria, pyuria, and eosiniphiluria. WBC cast may be present but NO RBC Cast. ****There is increased ASO titers, decreased C3 and CH50. Renal function goes back to normal in 1-2 weeks. C3 goes back to noraml 8-12 weeks. ASO returns to normal 3-4 months.

Post Term Pregnancy:
Is pregnancy more than 42 weeks gestation. Fetus has long nails, abundant hair and dry skin. Mngmt is based on well being of the baby. The non stress test and Biophysical profile should be performed twice weekly and if there is oligohydamnion or decelerations If all favorable delivery should not be induced until cervix is favorable, fetus is macrosomic. If pregnancy goes over 43 weeks then delivery is mandatory. If its more than 42 weeks, cervix is favorable and head is in pelvis labor should be induced.

Prador-Willi Synd
Deletion in arm of Ch15. It presents Genomic Imprinting where genetic disposition depends whether its inherited from mother or father. Narrow bifrontal diamond shaped eyes, down turned mouth, and he has to be restrained from eating so they are obese. Also Hypogonadism.

Precocious Pubarche:
Its important to ddx b/w Precocious Puberty that is caused by premature activation of the hypothalamic-Pit-Gonad axis and Precocious pseudopuberty that is caused by gonadotropin-independent process, typically an excess of sex steroids. If pt presents with severe acne and severe growth acceleration, its pseudopuberty. It can be cused by LATE CAH.The axis PP is not so dramatic. Its sequence is testicular enlargement, penis enlargement, pubic hair growth, and then growth spur is absent.

Precocious Puberty, Idiopathic
it represents 40% of cases of PP in females and resuts from premature activation of Pit-Hypothal-ovarian axis. It chaaracterizes by simiar cases in family and a normal sequence of secondary characteristics. GnRH stimulation test may be performed to confirm the activation of axis, leads to release of LH. Pt should be tx with GnRH agonist in order to ingibit the secretion of estrogen, preventing, thus, premature epiphyseal plate fusion. Althought parents need to know that this is benign, but they need to also know tat action needs to be taken.****DDX with precocous PUBArche, which is sign of severe androgen excess , caused by gonadotropin independant process like exces sex steriod due to 21-hydroxylase def.*****Etiology of PP can be divided into 1-true isosexual PP 2ary to activation of axis, 2-pseudosexual PP oncrease in steriod hormones and resultant 2ary sexual characteristics, mostly due to ovarian tumors, also adrenal tumos and exogenous esterogen exposure, hypothyroidism and Mc-cune Albright syn. In GnRH stimulation test, a significant release of LH indicates activation of pit gland, therefore true isosexual PP.

Prednisone SE
Insomnia, Nervousness, indigestion,hirsutism, DM, Arthralgias, Epistaxis and Osteoporosis.***** Chronic use can cause Myopathy and pt presents with acidosis due to decreased pulmonary function, next step is PFT.

Pre-Eclampsia - 7
Sever preeclampsia is defined as > or = 190/110 or one or more of the following: 1-oligurea, blures visio, Epigastric pain. Pts are at risk of Eclapmsia, It results from cerebral vasospasm and resultant cerebral hypoxemia. It’s a generalized Tonic-Clonic seizure that manifest like grand mal. It occurs 25% before labor, 50% DURING LABOR 25% after delivery. ****Depresed DTR is the first sign for Magnesium Sulfate toxicity which requires stoping it and administer Calcium Gluconate. The second sign is respiratory depression, then coma, then cardiac arrest and death. In pt with MILD PE the prophylaxis is performed during labor, delivery and w/i 24 hrs after delivery. In pt with SEVERE dis the prophylaxis is started on admission, and carried out though out the period of evaluation and observation. Now if pt iis also taking Hydralazine, it needs tobe stopped and c-section started if BP drops abruptly. *****If the HT presesnts before 20week gestation its either Mole, or Chronic HT. If US does not show "snow storm" then its Chronic HT***** Transient HT occurs in 2nd half of pregnancy during labor or delivery. Proteniuria may be present but doesn’t exceed >300mg/24hr, if it does then it is Preeclampsia. ****The mcc of death in ECLAMSIC pt is Hemorhagic stroke due to HT and thrombocytopenia.****Its a miltisystem dis asso with HT and Proteinuria. Rarely presents before 20week. Proteinuria is >2gr/24hr and asso with creatinin increase. Risk factors are:first conception, DM, renal failure and extreme age.****Its important to ddx b/w PE and SLE in pregnancy because they respond to two different therapies. HT in a pregnant woman in the setting of massive proteinuria , malar rash, and positive ANA titer is most likely due to SLE which causes Glomerulonephritis. Now SLE very rarely STARTS in prgnancy, and it usualy exists prioe to pregnancy, so there is Chronic GN. ****The most effective tx for Preecclampsia and Eclampsia is delivery and evacuation of placenta. Most times it cant be done because its too far from delivery date. Guidlines are:1-MILD Preecalmpsia, If at term deliver, if not bed rest,salt restriction adn close observation. Dexamethasone is given b/w 24-34 weeks and once the maturity is achieved baby is delivered. 2-SEVERE Preeclamsia, pt needs to be stabalized and evaluated before making decisions. Bed rest and salt reduction is mandatory, pt with > 160/110 requires antihypertensive therapy.It pt is stabalize, the decisionis then made according to the term, is mature deliver, if not wait till 34 week and deliver. If pt doesnt responde to therapy or if mother or baby are at risk, delivery is promptly required, wich is accomplished vaginally after induction of labor, if vaginal is not possible then a C-section is done. Now HT therpay guide lines are: If pt is near term or already in labor, Hydrazaline or Labetolol, short acting, are used. In pts far from term Oal agents lilke Methydopa(doc) is used. Calcium Chanel blockers are 2ndline agents, and ACE is CI. Nitropruside is used when BP is extremly high, avoid it before birth bacuase it causes cyanosis in fetus. For SEIZURE prophylaxis and Tx: the most effective agent is Mg Sulfate. For prophylaxis its given IM to all severe preecmpsic pt from admission, and milder pts get it during labor and w/i 24hr of delivery. For Seizure tx, IV injection is used.

Pregnancy early detections
1-Chorionic Villus Sampling: is done in 10-12 weeks gestation. Its indicated in women >35 directly after abnormal US appearance. Serum screening in this age group is less accurate. It detects chromosomal abnormalities. 2-Amniocenthesis is done 16-18 week of gestation. Cordocenthesis or Percutaneous Umbilical Blood Sampling (PUBS) is used for rapid karyotype analysis or Rheusis isoimmunization is suspected. MSAFB is routinly done in 2nd trimester. for Neural tubes and abdominal wall defects.


Pregnancy problems - 12
Screening for gourpB strep should be done 36-37 week gestation and positive cases should be tx with Penicilline G during labor, even in the absence of risks.*****Low back pain is very common in thrid trimester. Its caused by lumbar lordosis and relaxation of ligament to the joints.*** Both Graves dis and Migrain will improve in Pregnancy. ***Excessive use of oxytocin may cause water retention (acts like ADH), hyponatremia and seizures (water intoxication).****ACE inhibitors and Oral hypoglycemic agents (Glibenclamide) are CI in pregnancy. Stop them and give Insulin for DM, and Methyldopa(most comonly used), Hydralazine and Labetalol for HT control in pregnancy. ***Pregnancy hasa protective efect on both MS and PUD. ****Pt 6week gestation has positive FTA-ABS for syphlis, alergic to peniciline, the best tx is Desensitize her to penicililn since Erytromycin cant cure syphlis.****In pregnancy both BUN and Creatinin are decreased to half of prepregnancy levels. Amoxicilin has no effect on them.****Asymptomatic Bacteriuria of Pregnancy, increases risk of developing cystitis and pyelonephritis. E coli is the cause 70% of the times. tx is 7-10 days of Nitrofurantoin, Ampicillin or first gen Cephalos.****Neonates of pts with graves dis treated with with surgery are at risk for Thyrotoxoicosis.because of the passage of throid stimulating immunoglobin across the placenta.*****Hypotension is a comon se of epidural anesthesia. The cause of hypotension is blood redistribution to the lower extremities and venous pooling.****In pregnancy its recommended to CONYINUE excercise ust as you were doing before, like an aerobic instructor.*****Edema of lower extremities (Bilateral) in pregnancy is most commonly a benign problem. Pre-eclampsia should be suspected if the edema is associated with hypertension or proteinuria, no need to do ECG or DVT (presents unilateral and fever).***** Oxytocin is like ADH so it causes water retention and Water toxicity due to decrease in Na concentration ( 123 ). So it could cause seizure post partum.***** Screening cultures should be performed at week 36-37 and positive cases should be treared with IV penicillin therapy during labor to prevent the new born from getting infected.***** If hypertension sets in before 20 weeks, its either Mole or CHRONIC hypertension. If it sets in after 20 weeks, its either Preeclampsia (Proteinuria, >300mg) or TRANSIENT hypertension (not accompanied by proteinuria, <300mg)*****Antibodies to ABO antigens belong to IgM antibody class so they don’t cross placenta and hence mother and baby can have different blood type. But Anti-D antibodies that are responsible for Rh alloimmunization belongto IgG class and do cross placenta.

Premature Adrenarche
Isolated apprearance of axillary hair before agee of 6. Results from androgen secretion from Adrenal gland. Its benign and has no clinical significance. But Premature Pubarche (pubic hair growth before 8) is in 50% of cases asso with CNS disorder.

Premature labor
Premature labor must be managed agressively, and tocolysis has to be instituted at once. Magnesium Sulfate is the DOC for tocolysis therapy, but its not FDA approved (but Ritordine is). We also need to administer steriod for speeding up lung maturity.

Premature Ovarian Failure:
It refers to a failure of estrogen production by the ovaries of a a woman <35. Its most commonly idiopathic. But it could also be due to adhesion, Hashimotos’s, irradiation or chemo. Pt presents with few months of amenorrhea, Atrophy of vagina, increased FSH, previously had a child. The only way to treat infertility is Egg Donation.

Premature Rupture Of Membrane:
It’s a rupture that happens before labor at anytime of gestation. Premature PROM is a rupture that happens before term whether there is contraction or not. In PE there is amniotic fluid in vagina, confirmed with Nitrazine paper. In case of PPROM, amniotic sampling to measure lung indices is mandatory. Also do US to measure aniotic volume and determine fetal anomalies. If fetal has congenital anomay let vaginal delivery proceed. If fetus is viable then prolong pregnancy to mature lungs. REMEMBER if there is no contraction, no need to do tocolysis.

Premature Vent contractions
Require no tx, just observation.

Presby-cusis
In older people, it’s a high frequency hearing loss in both earswith difficulty in speech differentiation. "Acoustic neuroma is a unilateral hearing loss asso with rotational vertigo

Presby-opia
Normal aging loss of accomodative capacity.

Preterm labor
Defined as labor prior ro 37 week and is dx by 1-occurance of contractions, 2-cervical change or effacement of 80% or dialtation of >2cm. Preterm birth is delivery b/w 20-37 weeks. Low socioeconomic have higher incident. RDS is a common problem in premature births. If both mother and baby are stble, FIRST thing is to try bed rest and hydration. Eventhough there is effacement and dialation. Its succesful in 20% of times. Hydration is a negative feedback on the secretion of ADH and Oxytocin (post pituitary) , which is responsible for induction of labor. If this doesnt work, NEXT STEP is tocolytic therapy is initiated with Magnesium Sulfate. Pt should also be cultures for GroupB strep and antibody considered. Steriods are necessary for thi sstage of labor, when administered b/w 24-34 week they accelerate lung maturation. Agent used is Dexamethasone IM.


Preventative Medicine
1-DRE and hemoccult (PSA option) >50yrs, annualy. 2-Colonoscopy >50yrs every 10 years. 3-Pap is not needed for women who have had regular check up until age 65. 4-Mamogram should be done yearly from 50-75. 5-Routine Chlamydia infection screening for all sexually active women over 25 and younger. 6-If LDL Cholesterol is = or >100 for a DM risk factor then start life style modification. 7-Women who have had CIN ii or iii should have annualy pap smear even if the result is negative 3 years in a row. 8-Hormone replacement therapy is still the best therapy for osteoporosis in postmenopausal women. 9-Influenza vaccine on annual basis for all adults above 65 and adults of any age at risk of Influenza. 10-MMR is highly recommended for all HIV infected pts who are asymptomatic and are not severly compromised. 11-HIV infected pt should receive Tetanus and Diphteria immunization according to normal dosing schedule (meaning a booster every 15 years). DTP is only required for children since individuals >5years are highly unlikely to get Pertussis. 12-HepA vaccine is recommended for travelling to developing countries. 13-Yellow fever vaccination if necessary for Africa and south America travelers. 14-Menigococcial Meningitis is recommended for Hajj. ****Pneumococcus vacine is given at age 65.***Pap smear must be performed 3 yearsin a row before it could be left alone. *****If child develops zeizure after DTP vaccince, it is CI, so next time, Dont give Pertusus and just give DT.*****people with pancolitis and Ulcerative Colitis have in creased risk of colon cancer and should do colonoscopy 8 years after surgery and then every1-3 years after that to identify recurrent cancer in early stages.

Priapism
MC drug that cuases it is Prazosin, but in boards Terazodone is mostly associated with it

Primary billiary cirrhosis - 3
Although no specific tx is indicated, Ursodeoxycholic Acid is recommended to reduce itching and slow prrogression of dis. And untimately Liiver transplant. ***Presents in midle age and 90% are female. Its asso with Sjogren, RA and CREST. Liver biopsy is pathognomic with granlulatatose destruction of bile ducts in portal triad. Def tx is liver transplant.

Primary Dysmenorrhea
Pt present with hx of lower abdominal pain, that radiated to upper thighs and back. The pain is colicky and starts a few hours prior to menses, lasting 3-4 days. It usually appears 6-12 months after menarche. Tx is NSAID, so are OCPs. The pathology here is that menstural fluid has higher levels of prostaglandins.

Primary Hyperaldosteronism - 6
Parathyroidectomy is the only effective tx. Know how to ddx from Renovascular hypertension based on low renin activity in PHA and high renin activity in RVHT. Characterized by Hypertension, hypokalemia, and suppressed renin , The mcc is aldosterone producing adenoma (Adrenal Adenoma). ****The mcc of combination of hypokalemia+HT is PHA. *****The mineralocorticoid activity of Aldosterone results in increased salt & water retension, followed by nautriuresis (Aldosterone escape phenomenon). So there is MILD hypernatremia and NO EDEMA. ****Suspect it in a young pt with muscle weakness, and numbness and HT. The most specific lab value is high aldosterone / renin ratio .

Primary Hypoparathyroidism - 2
A7. *Increased serum calcium+decreased serum phosphate+increased PTH is the hallmark. DDX:Familian Hypocalciuric Hypercalcemia. In PHPT, urinary calcium excretion is normal or elevated, in FHHC calcium excretion is decreased, below 200 and creatinin is normal.

Primary Ovarian Failure:
It could be caused by chemotherapy and will cause premature menopause. Thr diagnostic blood test is increased FSH levels. Chemo, especially of Alkaline agents, causes failure of follicular cells of the ovary reaulting in decreased production of estrogen and inhibin. This results in loss of feedback inhibition of estrogen in LH & FSH, causing their high levels. Inhibin cuases the feedback inhibition of FSH only, so in the absence of inhibin the absence of inhibin FSH levels are higher than LH levels, which is pathognomonic for ovarian failure. Clinically, pts have Amenorrhea, hot flashes. They might develop anxiety, depression and irritability. Loss of estrogen results in atrophy of breast, vagina, myometrium.


Primary Polydypsia
Or Psychogenic Polydypsia. Characterized by primary increase of water intake. . In contrast with DI and DM , it’s the polyuria which is the driving force of increased water intake. This is seen mostly in anxious women young, . Pts taking Phenothiazine have this problem because dry mouth caused by anticholinergic effect of the drug. Promary Polydypsia can also be caused by hypothalamic lesions affecting the thirst center.

Primary Sclerosing Cholangitis
Its an inflamatory destruction of both intra and extrahepatic bile ducts. Its asso with IBD especially UC in 70% of cases. Hay jaundice, pruritis, RUQ pain, or acute cholangitis. It may progress to complete obstruction, 2ary billiary scirrhosis, hepatic failure or portal HT. ERCP is investigation of choice. Cholangiography shows characteristic 'Beading' due to stricture and dilations of intra and extra hepatic ducts. LFT shows very high Alk Phosphatase, mild ALT & ALT, <300. Other findings are Hypergamaglobinemia, increased serum IgM, P-ANCA. Tx is directed towards refleif of obstruction & trx against infectious complications. Liver transplantation is indicated in cure.*****CholangioCarcinoma is a complication of PSC especially in smokers with recurent UC. One of the early manifestations of CC is a severe STRICTURE in the billiary tree leading to Cholangitis characterized by Fever,jaundice,Leukocytosis. CC can happen in 20% of PSC pts. So ERCP shows the stricture, the next step is to do BIOPSY of the stricture to rule out CC. Ursodeoxycholic acid can be used in pts with PSC to lower liver enzymes.

Prolactinoma - 2
Another nmae is Lactotroph adenoma, the mcc pituitary tumor. A pituitary tumor <10mm is called a Microadenoma. Microprolactinoma is a prolactin producing microadenoma. In females it presents with galactorrhea and in males with hypogonadism. Dopamin agonist such as Bromocriptine or Carbegoline are the mainstay of Tx. If that didnt work for both Micro and Macro adenomas, then do MRI and then surgery is indicated.

Propylthiouracil
MC SE is allergic reaction. But the most serious SE is Agranulocytosis. Routine monitoring of WBC is not helpful, but majority of pt revocer.

Prostate cancer - 4
Radiation is usful in managing bone pain with prostate cancer who have undergone orchiectomy. *Acute Cord Compression could happen secondary to metastasis of cancer. Its also called Cauda Equina. First symptom is back pain asso with tenderness at the site of the metastasis. Tx step are: 1- IV Dexamethasone, 2-MRI of spine, if MRI is CI (Pace maker pt) or CT myelogram. 3-If confirmed, then radiotherapy.****Screening is : 1-Men 40-50 , do yearly digital exam. 2-age>50 do yearly digiral plus PSA. If either recta of PSA (>4) is suggetive the next step is transrectal USG, followed by niddle biopsy and then staging. For stagin, do bone scan to see metastasis.***When the cancer is in late stages (metastasized) surgery is not performed unless needed for palliation. Palliative radiation along with antiandrogen thrapy is the tx of choice. Leuprolid (LHRH analog) is doc, Flutamide is not as good.

Prostatitis, acute bacterial
In Younger pt its caused by STD, ini older pt by E Coli. First get a mid stream urine sample to cultrure and then give antibiotics empirically. Pt presents with chills and fever, urge to urinate and pain in urination, prostae is tender and boggy.

Prostatitis, non bacterial
Pts are afebrile and have irritable voiding symptoms, like frrequency, irgency and suprapubic discomfort. Urin analysis in normal. Expressed prostatic secretion show a leukocyte count greater than 10WBC/HPF and the culture is negative. We need to Rule out bladder cancer on all elderly pt with urinary cystology and cystoscopy.

Prostatodynia
Pt is 30-40 yo, afebrile, have irritative voiding syptoms. Expressed prostatic secretions show normal leukocytes (DDX with non.bact prostatitis), and culture is negative for bacteria. No hx of UTI.

Pseudocyesis - 2
a condition that a woman presents with all symptoms of pregnancy, owever, US reveals a normal endometrial stripe.*** All pts need Psych evaluation.

Pseudodementia - 2
Asso with Deppresion.***** SSRI ix tx.

Pseudogout - 3
Positive bifringent crystals. *****Asso with aging, trauma, , Hyperparathyroidism, Hemochromatosis. Pt complains of constipation, polyuria, fatigue and joint pain. His serum calcium level is high and phosphate is low, this suggests hyperparathyroidism. The joint aspirate under light microscopy shows Rhomboid shape, calcium pyrophosphate positive bifringent crystals. Gout has Needle shape negative bifringent. Struvite srytals in Nephrolithiasis have Cofin lid shaped crystals.

Pseudomembraneous colitis – 2, GI, 6/2
tx, Stop offending drug and start Metro. ****Clostridium difficile is the cause. Suspect a pt with diarrhea who has been taking antibiotics for a while. The most sensitivd test is Cytotoxin assay in the stool. Tx of choice is Oral/IV metronidazole. Or ORAL not iv Vancomycin.

Pseudotumor Cerebri - 4
Risk factors include medication(steriods and OCP), trauma and increased weight. Initial sign is a headache that is pulsatile and awakens thept from sleep. Then increased ICP occurs. Also nausea, vomiting, back pain, double vision, Papil edema, palsy . First things to do is CT/MRI to rulle out mass occupying lesion, most pt have empty sella in CT. Only then LP is indicated . CSF reveals increased P and normal cells. Sufficient CSF should be removed to lowerr P to 150mm/H2O. Following are criteria for DX:1-Presence of features of ICP in an alert pt. 2-Absence of focal neuro signs except 6th nerve palsy. 3-Normal CSF exept icreased opening presure. 4-Absence of any ventricular abnormality other than enlagement caused by ICP. ***Vitamin A and its derivatives taken in large doses could cause PC, like an acne pt taking Isotretinoin.***If LP didnt reduce the ICP, then the first line of therapy is Acetazolamide that inhibits Choroid plexes CA Inhibitor. If that filaed then Surgery, optic nerve sheath decompression or lumboperitoneal Shunt, is indicated.

Psoas Abcess
Pt presents with RUQ pain, fever, and positive psaos sign, he is also having furuncles which means he is having infectious sysmptoms. The infection has lead to Psaos abcess and the nest best thing to do is CT of abdomen to look for it. If suspcion is high and CT is negative the next step is Lapraroscopy. Tx is drainage and systemic antibiortics.

Psoriasis
Lithium can precipitate it.******Silvery scale skin.

Psoriatic Arthritis
Is Asymetric and oligo-articular. Pt has silvery scales on eryhtomatous plaques on flxural surfaces. Think of PA when pitting nails is in the hx. Othe rfeatures are spondyloarthropathy, DIP involvement. DDX with OA: OA is not inflammatory. DDX with RA: RA is not in DIP.

Puerperal Fever:
Is defined as T of >38 more than 2 days w/I 10 days after delivery. Occurs in 6-7% vaginally deliveries. The mcc is Endometritis. In assition, gnital tract infection may secondary evolve into pelvic Thrombophlebitis, abscess or schock. . It suspected if hay persistant spiking fever that wont responde to antibiotic therapy. Add HEPATIN to antibiotics for 2-3 weeks. If a rapid response is not noted then suspect abscess.

Pulmonary Contussion
Cx presents within first 24 hrs of tauma , tachypnea, tachycrdia, and hypoxia are charcersitc. PE reveals chest wall bruising, and decresed breath sounds. Cxr shows patchy irregular alveolar infiltrates. ABG sows hypoxia and ,iteslef, is an indication to suspect PC in trauma pts.

Pulmonary Embolism (Pulmonary Vascular Diseas) - 14
There is no pulmonary edema in PE. Think of PE in a post operative pt with JVD and new onset RBBB. * PE occurs when there is a sudden dislodgment of blood clot from DV in to the pulmonary artery. Risk factors are prolonged bes rest, obesity , trauma, malignancy, inherited def of prothrombin-III, protein C and S and use of Estrogen. Symptoms include dyspnea, chest pain, cough and hemoptysis, tachycardia, hypotension, and pain in the leg. The most typical feature of PE in seen in the blood gas. Aterial Hypoxemia is ALWAYS present. . Dx involves V/Q mismathc or Venous Duplex Ultrasound of the legs. Pulmonary angiogram is the gold sandard. Tx is Oxygenation and anticoagulant. Low molecular weight Heparin, thrombolytic therapy, inferior vena cava filter may be required. *Pt in whom anticoagulation is CI, require placmeent of IVC filter when they are diagnosed with DVT, for prevention of PE. * A pt with PE and hx of recurent unexplained DVT , factor V Leiden is a common cuase of it. factor V Leiden is the result of a point mutation in a gene coding for the coag factor V, because of mutation factor V becomes resistant to inactivation by protein C, an important counter balance in hemostatic cascade. *The initial symptom maybe sudden onset of hypoxia, tachycardia, and tachypnea, hypotensive. The Cxr maybe normal, if that is the case Pneumonia, Atelectasis and Pulmonary edema are rulled out and the next step is Ventilation/Perfusion scan.*****A truck driver sitting all the time with tachpnea,tachycardia and dyspnea and right axis deviation has PE. The next sep is perfusion/ventilation. You should expect larg area of perfusion defect w/o ventlation defect.****Tx steps: If the pt is hemodynamically compromised and the clot is situated in main pulmonary artery, the best tx option is pulmonary Embolectomy. Now Heparin is a good choice but it doesnt dissolve the clot, so give him heparin and take him to surgery for embolectomy. Fibrolytic therapy is an excellent choice but not in a post operatve pt, it may cause diffused bleeding. So to summ it up, When an acute PE occurs, with hemodynamic compromise, the best tx is Fibrolytic therapy. if the pt has CI to it (has surgery a few days ago), an embolectomy id the tx of choice.****In PE pt the cxr is NORMAL.****Sudden onset shortness of breath with clear lung is the clue. Put the pt on O2, and bed rest. Heparin should be started immediately, if the index of suspition id high, for 10 days. After you do cxr, ABG and EKG Then do V/Q ro rule out PE. Angiogram is gold but is rarely done. Doppler US is choice for DVT. Spiral CT is done if emboli is large and Cxr is abnormal, in that case its better than V/Q. Thrombolytics are used in pt with hemodynamic instability or if after iv filter pt is worst. If pt has CI to prolonged anticoag therapy or has another embolization while in the hospital, placemnt of IVC filter should be considered. Embolectomy is done in pt who is hemodynamicaly Unstable and has CI for thrombolytic use. So bottomn line is if you suspect PE clinicaly, and Cxr, ABG and EKG ruled out other ddx then you should being Heparin w/o waiting for V/Q to confirm your Dx. ****PE is the same as "Pulmonary Vascular Dis". dont mistake it for "Pulmonary Septic Emboli: which is caused by release of emboli from right heart due to IE of IV drug users.****When the suspicion is established V/Q is the next step,no need to rule out MI by doing CKMB. But if one of the options is Start coag tx , then that is the first step and then do v/q.

Pulmonary HT
T13Q39. Its defined as mean pulmonary arteial presure >25mmhg at rest and >30mmhg with exercise. Pt presents with dyspnea on excertion, Cxr shows enlargement of pulmonary arteries, enlargement of the right ventricle. EKG shows right axis deviation. Untreated PHT leads to Core PUlmonale.

Pulmonary Regurtitation:
Early diastolic murmur, decresendo, high pitched, blowing, best heard around the left sternal borderand usually develops secondary to pulmonary hypertension. Becomes more prominent with inspiration.

Pyelonephritis - 7
Urine and blood cultures must be obtained prior to obtaining antibiotics. After 48-72 hours of parenteral therapy (IV Ceftriaxone) the pt can be switched to an oral agent (TMP-SMX). Dipstick is positive for both Nitrites(indicates Enterobacteriaciae) and Esteras (indicates Pyuria). Its a risk factor in pregnant woman, soits important to treat UTI before it progresses to full disease. When there is no responde to 72 hr therapy with antiobiotics, US or CT is recommended to rind out other pathology like obstruction or Abscess.* Tx is empiric therapy w IV antibiotics. ****CHRONIC Pyelonephritis is characterized by focal parenchymal scaring and blunting of Calysis in IVP.

Pyloric stenosis - 2
Surgery is tx but first hydrate and give K to infant.****4 Week old infants has sudenly started to vomit after feding for the past 2 days. Its PS. Don’t be fooled by no mass being palpated. You need to do US to dx, it'll show hypertrophied pylorus.****Presents in 4-8 weeks with non bilous projectile vomiting. *****1st order of tx is to correct hydration and electrolytes and after that do surgery.

Quinidine Toxicity
Tinnitus, diarrhea, torsade de pointes, prolonged QT, hemolytic anemia.

R. A.
Pts with erosive joints need Disease Modifying Anti-Rheumatic Drugs (DMARD). Indomethacin or other NSAID may be used as joined therapy. These drugs slow down progression of erosions and cartillage lost. If conservative mngt didn’t work then steriods are used. DMARDs include Methotrexate, Hydroxtchloroquine, sulfasalazine, azathioprine. Methotrexate is the doc.****Methotrexate works by inhibiting DHF reducatase, so Microcytis anemia is a side effect. Hydroxychloroquin SE is GI stress, visual disturbances. and hemolysis in G6PD def. Prednisone may cause iatrogenic Cushings. Azathioprine may cause Pancreatitis, liver tox and BM supression. Cyclosporine is used in ttansplantation and may cause Nephrotoxicity.

Rabies
Try to capture the dog to see if he has rabies, if the dog was not captures assume he had rabies and tx is both actve and passive immunization.If dog is captures, observe for 10 days,if there is suspicion then do brain autopsy if its confirmed then the post oxposure prophylaxis (both pasive and active immun) is given.

Radial fracture:
Fractures of shaft of humerus present with tender, swollen, crepitus and shortend arm. Xray confirms Dx. The recommended tx in isolated diaphyseal humeral fracture is closed reduction followed by hanging cast

Raloxifene
Is a Selective Estrogen Receptor Modulator (SERM) used for prevention of osteoporosis. Unlike Estrogen, it doesn’t increase risk of Endometrial cancer. It decreases risk of Breast cancer. It has noeffect on Ovarian cancer. Ts most important SE is it increases the risk of Pulmonary Thromboembolism and is CI in pt with a hx of DVT. It may also worsen hot flashes and vaginal dryness.

Ramsay Hunt synd
A facial nerve palsy, caused by Herzes Zoster. Pt presents with lesions of external ear w or wo invlvement of tympanic membrane, vertigo, tinitus, and deafness due to geniculate ganglion involvement.

Rationalization
When a pt ignores a breast cancer and does not go to doctor, when asked why? Her rationalization is that because there is no family hx of breast cancer in my family.DDX is Denial , when a pt dx with terminating diseae but he denies it. DDX Intelectualization allows someone to talk about his dis in a detached manner.

Reaction formation
When a women who hates foreigners volunteers to help them in a center. DDX is Alturism, which is when a rape victim establishes a rape center to help other raped people, in order to help her own anxiety.

Reflex Sympathetic Dystrophy
Is a syndrome of Pain and swelling aso with vasomotor instability. Usually asso with move resriction of shoulder. Xray shows Osteopenia. Physical therapy is done to restore function. Prednisone is used in resistant cases.

Reiter's Synd or Reactive Arthritis
Arthritis, Conjunctivitis and Urithritis. Typical skin lesion is Keratoderma Blenorrhagicum on palms ans soles. Clear vesicels on red bases that later develop into macules and papules. Other lesion is Circinate Balanitis on penis, shallow painful ulcer. ****Its grouped under "Seronegative Spondyloarthropathies" and these pts have NEGATIVE RF. HLA/B27 is found in ONLY 80% of white pts and 60% of blacks.*****Its Arthritis reactive to genitourinary infection due to Chlamydia, might develop heel pain and sausage digits. Tx of choice is NSAID, if disease is due to infection with Chlamydia add Tetracycline.

Relapsing polychondritis
an idipathic disorder characterized by recurrent inflammation of catilagenous structures and other internal organs

Renal Arteery stenosis
In a pt with multiple risk factors for atherosclerosis (hypercholestrolemia, DM, smoking, HT), RESISTANT HT should make yo thnk of RAS. Continuous murmur (Systolic and Diastolic) in the periymbilical area or in flanks, is characteristic of RAS.

Renal Calculi
Excessive use of Vit C in pt with renal insufficienct can cause Oxalate stones (Radioopaque) (Gout stones are Radiolucent). The mc renal stones are calcium, CT of the abdomen w/o contrast is the dx procedure of choice, it can detect radioopaque(calcium) and radiolucent(uric acid) stones, KUB or abdominal xray is not the best test, the dietry recommendation for pts with renal calculi is 1-Decreased dietry protein and oxalate, 2-decrease sodium intake, 3-increase fluid intake, 4-increase dietry calcium. *****Renal stones in pregos require special care. Since there is no radiation with renal and pelvic US, this is the procedure of choice.

Renal cell carcinoma
Classic triad (flank pan, hematuria,palpable mass) in uncommon, but whne present it means metastasis has occurred. No matter what the vignette says if it mentions VARICOCELE fails to empty in recumbent position, it is RCC. So the next step is Abdominal CT.

Renal Laceration:
Presents with hematuria and retroperitoneal extravasation. CT, with contrast, will visualize both kidney and spleen. CT is the best study to evaluate solid organ damage in a stable pt.

Renal Tubular Acidosis
RTA results in impaired ammonia excretion which is the principle mechanism of met acidosis in CRF. Suspect RTA-type4 if hay DM pt +Hyperkalimia. *RA presents with decreased pH, then looking at PaCO2 we see increase and then HCO3 is a little elevated to compensate. Common causes are: 1-Pulmonary diseases. 2-Neuromuscular disease, any condition that weakens or paralysis respiratory muscles (MG, Polimyolitits, MS and Kyphoscoliosis.). 3-Primary CNS dysfunction like a brain lesion. ******RTA is a term used to describe conditions in which normal AG metabolic acidosis occurs from decreased net renal acid secretion. 3 types:
1-RTA-1: Distal tubular acidosis- characterized by decreased distal tubular capacity for hydrogen ion secretion and therefore inability to generate new bicarbonate. Suspect it in a pt with AG MA and a urine pH>5.5. Serum K is classically low in pts with distal RTA. Causes are Sjogren, SLE, Amphotericine toxicity, Wilson’s, Sickle cell, Fabry;s. Its asso w Nephrolithiasis.
2-RTA-2: Proximal tubular acidosis- Hallmark is impairment in proximal HCO3 reabsorption leading to excretion of excessive urinary HCO3. Pt has urinary pH<5.5. Normo or hypokalemia The mcc are heavy metal poisoning, Wilson’s, Multiple Myeloma, Amyloidosis. Suspect it when pt has normal AG MA and HCO3 around 15 and urine pH <5.0. Asso with osteomalacia.
3-RTA-4: Hyperkalemic RTA- This is the mc form in adults and results form Aldosterone resistance or deficiency. Almost everyone shows Hyperkalemia which is asymptomatic. Some causes are DM and Cyclosporine. Suspect it in pt w non AG MA and hyperkalemia.

Renal Vein Thrombosis
Is a complication of NephrOtic Synd. Antithrombin III is lost in urine. Pt presnts with sudden onset of abdominal pain, fever and hematuria. It can occur from any Nephrotic synd but MCC is Membraneous GlomeruloNephritis

Respiratory Alkolosis - 4
Increased pH and decreased PCO2. Due to hyperventilation. Homeostasis of 3 forms of palsma Calcium is dependant on extracellular pH. An increased extracellualar pH levels causes an increase in the affinity of serum albumin to calcium, thereby increasing the levels of albumin-bound calcium and consequesntly decreasing the levels of ionized calcium. Ionized calcium is the only physiologically active calcium, so decreased level lead to hypocalcemia(crampy pain, paresthesiaa and carpodedal spasm)

Restrictive Lung disease
DoMerck. A-a in increased in interestitial lung disease due to poor oxygenation. In restrictive lung disease TLC, FRC, and RV are all reduced. But FEV1/FVC is either normal or increased.

Retinal artery occlusion,cental
Painless loss of monoocular vision secondary to embolism. Embolism of retinal artery is the mcc of ocular stroke. Its commonly asso with Amorousis Fugax before the occulsion. Fundoscopy reveals ischemic retinal whittening and red cherry spots. Its an ophthalmology emergency, delay in tx may result in blindness. Immediate intervention includes ocular massage (which moves the emboli) and high flow oxygen therapy.

Retinal Detachment
Sudden onset & mostly unilateral of photophobia and floaters, the most classic descritpion is "a curtain coming down over my eyes".Tx is laser and cryotherapy.

Retinal necrosis, Acute
Most commonly by HSV and VZV in HIV pts. Asso with pain keratitis, uveitis and peripheral pale lesions. In CMV retinitis is painless, not asso with keratitis and conjunctivitis, but fundoscopy shows hemorrhages and fluffy or granular lesions around retinal vessels.

Retinitis, CMV
T9Q13. Occurs in HIV pts when CD4 falls below 50. It presents as yellowish-white patches of retinal opacification and retinal hemorrhages. Tx is ganciclovir or Foscarnet.

Retinoblastoma
Highly malignant tumor and failure to dx and tx early may lead to death from liver and brain metastasis. It’s the mc intraocular tumor of the childhood. Every case of LeukoCoria (white reflex) is considered retinoblastoma until proven otherwise, although the mcc is congenital cataract. So these pts shoud be refered to Ophtamologyst. Other signs are strabismus, decreased vision, ocular inflamation, eye pain, glaucoma.


Retro Pharyngeal Abscess
Fever and cervical adenopathy, POSTERIOR pharyngeal edema and nuchal rigidity.Tx is IV broad spectrum antibiotics or drainage.

Rhabdomyolysis - 2
Characterized by break down of muscles. MCC is alcoholism. Risk factors are crush injuries, strnous exercise and seizures. Dipstick testing reveals Hematuria, but microscopic analysis of urine does not show RBC. Serum creatinine is disproportiany elevated compare to BUN. ATN can occur due to ischemia or toxins. Hb or Myoglibin are endo or exogenous nephrotoxic substances which can accumulate in the kidney due to break down of muscles, and leading to ATN. Tx include aggressive IV hydration, and alkalinization of urine. In some cases, forced diuresis with Manitol may be required. ***In crush injuries tx is IV fluids, osmotic diuretics and Sodium Bicarbonate (to alkalinize urine). After crush injury in accidents, it might show with mypglobinemia, myoglobinuria and eventual renal failure, urine dipstick test is positive for hematuria. Hyperkalemia is common showing tall T waves on EKG. The best Dx test is Creatin Kinase. The best initial management is osmotic diuretics and bicarbonate (to alkaline urine) to prevent renal failure.

Rhesus Isoimmunzation
It happens when there is contact of Rhesus-positive fetal blood and Rhesus negative of maternal blood. This results in mother's antibodies to cross over to baby's circulation and cause hemolysis of baby blood. In sever cases it causes Hydrops Fetalis. In mild cases it causes, jaundice, w/i the first 24hr after birth. Any incident that causes bledding can cause this (amniocentesis,CV Sampling, Abortion, ectopic pregnancy, labor and delivery. The best tx is to prevent mother's immune system to be in contact with fetus blood cells, the best thig is AntiD-gamaglobin (Rho-GAM)wich prevents contact by decreasgin availablity of fetal RBC in maternal circulation. In case mother was not sensitized (antibody titers < antibody ="">1:6) administration of RhoGAM is useless and close fetal monitoring for hemolysis is required. Methods used for this is Amniotic Fluid Spectrophotometry, it measured bilirubin in amniotic fluid. US is used in conjunction with it to detect Hydrops fetalis.

Rheumatic fever - 3
The mcc of Mitral stenosis is RF. Principle symptoms are dyspnea, pulmonary edema, and precipitated by exercise fever, anemia, sexul intercourse, tachycardia and pregnancy. In MS the left atrium gets very large and pushes up the left main stem bronchus. Occassionaly pressing the phrenic nerve causes persistant cough. Other findings are loud S1, opening snap and a diastolic murmur. ECG reveals atrial fibrilation. Echo will confirm dx. All pt should receive Endocarditis prophylaxis. Tx is mitral valve surgery. *Prevention os recurrent attacks of RF might slow down progression of MS, so Penicilline prophylaxis with monthly IM injection of Benzathine Penicilline is recommended in adulescent. Pts with or W?O Carditis need prophylaxis, with carditis need 10 yr and w/o 5 yrs. If there is a hx of embolic events anticoag is indicated.

Rib Fracture
MCC is auto accidents. Comlications are atelectasis, pneumonia and respiratory arrest. It could be deadly in elderly, this is due to painwhich causes hypoventilationand atelectasis/penumoina. Pain management is the PRIME priority in these pts. Local nerve block is used if oral or systemic analgesics are not helpful.

Ritordine SE, OBGYN, 6/2
Beta 2 agonist(Ritordine, for Tocolysis) may increase edema by decreased water clearance, tachycardia nd increased myocardial work load. They also increase Gluconeogenesis in liver and muscle resulting in increase demand for insulin in DM pts.

Rocky mountain spot fever
Characterized by fever,myalgias, HA, and petechial rash. It’s the mc and fatal Tick born dis in US. A delay in Dx can lead to Shock and systyemic signs. If shock is there (BP reduced a lot) then IV hydration is to be done prior to anything else. Then Tetracycline, oral or IV.


Rosacea
In pt of 30-60 yrs, light skin and eye color, telangiectasia over the cheeks, nose and chin. Flushing of these areas is precipitated by hot drinks, heat , motion and rapid body T changes. Tx is aimes at papules, pustules and erythema by topical Metronidazole.

Rotator Cuff Tear
Pt presents with shoulder pain aggrevated by movements like pushing, pulling, and positioning the arm above shoulder. If limitation of abduction or external rotationd does not reverse with Lidaocaine injection rules out Tendenitis and focuses on RCT. Def Dx of RCT is with MRI or Arthrography. If that is the case then surgery will fix the problem.

Rotor's Synd
Presents with Conjugated bilirubin in urin (urine dipstick show is). If billirubin is in urine it must be congugated since uncongugated can be excreted.

Rubella - 5
Macupapular rash, posterior cervical and posterior auricular lymphadenopathy and polyarthralgia. Pt can be tx with acetaminophene for symptoms. Pt can be infectious 1 week prior to the onset of the rash and 15 days after. Live attenuated vaccine os the best protection. Rash starts on the face and then progress to trunk. Arthritis that develops in adult females is a clue for rubella.***Most cases of postnataly acquired rubella are asymptomatic. Classic Rubella presents with Rash, low grade fever, lymphadenopathy. Lymphadenopathy and fever appear about 14 days after initial infection and prior to rash. Rash is in distinguishable from Measles. DDX1:Scarlet fever, pharyngitis, fever, sandpaper like rash. DDX2:Chicken pox has vesicular rash. DDX3:Erythema Infectiousum, has Slapped cheek appearing rash. Roseola Infantum Herpes virus 6, abrupt high grade fever, and then maculopapular rash on trunk and then periphery. Pt is no longer febrile whne rash appears.***After rubela vacicantio its recommended that no pregnancy for 3 months. But so far there are no reported fetal transmission in this regard, so if it does happen and the is then gets pregnant, just reassure the mother nothing else needs to be done.****Congenital Rubella Syndrestns with triad of sensorineural deafness, cardia malformation(PDA,ASD) and Cataracts (White reflex)ery muffin spots and thrombocytopenia. Pt also has 'blub. transmission in 4th week of pregnancy. Maternal infection manifest as Rash, arthralgias and lympgadenopathy. If it is at the begning of pregnancy its worst, might lead to spontaneous abortion or CRS (risk is 50% if transmited in 4week, and 1% if in last trimester). Matrenal infection is confirmed by antibody to rubells virus In infant, there is IgM antibody or persistant IgG beyond 6 mo. Prevention of Rubella is by viccination of ALL femlaes of child bearing age. If immunstatus is unknown, titers should be obtained first trimester. Vaccination should not be given in pregnancy. DDX1:Sturg-Weber:neurocutanous dis, port wine stain in distributin of trigeminal nerve, mental retardation,seisure and galucoma.DDX2:RetinoblastomaMay present with unilaterl or bilateral White reflex, HOWEVER hay No deafness or PDA as is is CRS. DDX3:Congenital CMV and Toxiplasma, both manifest with Hydrocephalus, cerebral calcification,chorioretinitis, and microcephaly. but no deafness or cardiac anomaly.

Salicylate tox
mixed respiratory alkolosis and metabolic acidosis.

Salmon patch:
A flat salmon colored lesion over eyelids and neck. It’s a vascular lesion that disappears early in childhood.

Sarcoidosis - 4
Systmic steriods are the DOC for Sarcoidosis. Its indicated in pt with disabling systems (visual disturbances,cough,shortness of breath) and Organ dysfunction. Pt has Hilar lymphadenopathy(organ derangment). Sterids suppress activated T-induced cell process at disease site. Asymptomatc pts need no tx. ****Cxr shows granular Hillar opacity in lungs bilaterally( midiastinum adenopathy). One midiastinum adenopathy is seen on Cxr, the next step is to perform mediastinal bronchoscopy and obtain tissue for biopsy . Ca is increased and Steriods are tx of choice for SYMPTOMATIC pts.****Pt with no symptoms but dyspnea that has gotten worst and joint pain with cxr showing hilar adenopahty and non-caseating granuloma.****Skin manifestation os rash is found in 30% of pts, to Dx Scintigraphy, to Confirm biopsy and NC Granulomas.

Scabies
Tx for adults is 5% permethrin cream

Scaphoid Fracture
Common in adults, after a fall on a outstretched hand. Pt presents with pain at wrist movement. Tenderness in Anatomical sniffbox. It might not show on xray for 2 weeks. So if signs are there but xray is negtive tx as if it is fracture. 10% go to have avascular necrosis due to tx error. Cast immobilization is recommended in the treatment of all Non-displaced scaphoid fractures (fractures <2mm displacement and no angulation). Open reduction and internal fixation if initial Xray shows displacement fracture. *****If fracture is suspected even if the xray doesn’t show it (cause it takes a few weeks) then the best mngmnt is long arm cast and treat it as if there is a fracture.

Scarlet fever
Presents with fever, toxicity, pharyngitis(Grey-white exudate), sanpaperlike rash and strawberry tongue and 'circumoral pallor'. By GroupA strep, because of erythrogenic toxins. tx is PenicilinV. If allergic, Erythromycin and Clindamycin are alternatives.DDX1:Kawazaki, due to pharyngitis, strawbery tongue, Lyphadenopathy and rash. However, it must hace either change is peripheral extremity or conjunctival injection, to make the dx. Its difficult to DDX but response to Penicilline makes the dx.

Schizoaffective disorder
Combination of Schizophrenia and mood disorder. So pt hears voices, but is not violent and just wana be alone.

Schizophrenia - 10
Fluphenazine & Haloperidol (both long acting and injectable) are the tx of choice in pts who suffer relapses due to tx non-compliance. Injection can be dome outpatient and intervals (twice per month). ******The symptoms are 2 types, + & - . The + symptoms include, hallucination, delusion, disorganized behavior and speech. The - symptoms include the Five "A"s, Affective Flatening (diminished emotional responsiveness), Alogia (poverty of speech), Apathy (impaired grooming, unwilingness to perform activities), Asociality (social detachment) and Attention (impaired attention when interviewed). Positive symptoms respond well to typical antipsychotics. The Negative symptoms responde to Atypical antipsychs, Risperidone.***** 1-Schitsoid personality:social detachment and restricted range of expressed emotions. 2-Schizotypal: Eccentric behavior and a reduced capacity for close relationship. They might believe in magical thinking and might have bizzar fantasy or believe in telepathy,or sixth sense. 3-Avoidant personality:hypersensitive to critisim. They fear ridicule. 4-Borderline personality has Splitting. 5- Schizofreniform:hallucinations, delusion, disorganized speech, catatonic or flattened affect. Symptoms must be there for at least one month but less than 6 months. 6-Schizophrenia is like number 5 but sysmptoms last more than 6 months.***Subtypes of Schizophrenia are: 1-Paranoid:Preoccupation with delusion,auditory halluciantion w/o disorganized speech or inappripriate affect, they rsponde to pharmacotherapy. 2- Disorganizedisorganized behavior & speech, and flat or inappropriate affect. 3-Catatonichysical symptoms including immibility or excessive motor activity and assumption of bizar postures. 4-Undifferenciated:mixed symptoms that dont meet the critetia of other subtypes.***Schizotypal is presents with odd behavior (like saying I have some powers) and no close friends. DDX is Schizoid are those hace social detachment but no odd behavior (powers and things). DDX is Avoidant are those who want to be socially involved but are afraid of rejection.****Schizophrenic pts have enlarged ventricular size, Decreased cerebral mass, hippocampal mass, temporal mass and no change in cerebellar mass.****The following s&S are indication for hospitalization of schzophrenic pts: Homocidal ideation, suicidal ideation, grave disablity, gross disorganization, agitaed or threatend behavior. ****Catatonic pts are tx with Benzodazepine. Pts dont move,talk, and there is rigidity. so Lorazepam ix doc.****remember DDX b/w Schizophrenia dn scizpphreniform is duration of more then 6months.

Scleroderma
CREST syndrom. Asso with anti scl-70 antibodies.Wegener asso with C-ANCA. Polymyositis and dermatomyositis are asso with ANA and Anti Jo 1 antibody.

Scoliosis
Types are:1- Idiopathic ( the mc), has three types infantile(<3yr),>10yr. 2-Neuromuscular , 3-Congenital. Dx is by bending over and Cobb's angle of scoliosis. Tx is aimed at control of curve progression. Careful obervation and follow up with mild (<20)>30 give bracelets. If initial notic is >40 degree then Surgery is required.

Seborrheic Dermatitis
Dry scales and underlying erythema of scalp central face, interscapular areasumbilicus and body folds. Asso w Parkinson's Disease. Suspect HIV with a young pt with SD. Suspect Hepatitis C in pt with Lichen Planus.

Seborrheic Keratosis
Refered to as "barncles of old age". Benign. Welvety or warty, greesy, with stuck on appearance. Color from pink to black. Anywhere except palms and soles. Dx is shave biopsy. Remove for cosmetic reasons.

Secondary to the spine
Metastasis from carcinoma is the mc malignant tumor of the skeletal system. In pts w hx of malignancy, back pain raises suspicion for bone metastasis. Progresive low back pain not relieved by rest and worst at night suggets vertebral body metastasis. Secondaries to th ebone mainly arise from priamries f the prostate, breast, lung, lymphoma multiple myeloma. Technetium 99 scinti scanning is the method of choice.

Seizures - 2
1-Generalized (involving all brain) Tonic Clonic: Characterized by aura (lights), followed by stiffness and lost of consiousness. The next step is tonic-clonic jerky moves, biting the tongue, incintinence and postictal confusion. 2-Status epilepticus: multiple epileptic seizures one after the other. 3-Absence seizure, last a few seconds and no postictal confusion. and no loss of consiousness. 4-Simple partial seizure: focal motor sensory or autonomic symptoms and no loss of consiousness. 5-Complex partial seizure: focal seizure followed by transient or incomplete impaired consiousness.****CT of the head WITHOUT contrast is the initial Dx of choice when a pt is presented with impaired consciousness or seizures. CT WITH contrast is usually indicated if brain tumrs or other masses (Toxoplasma,lymphoma) is suspected. If the NON contrast CT excludes the hemorhage then MRI or CT WITH contrast is the next step.***Tods palsy is a transient hemiplasia that occurs after a seizure.

Selective IgA/IgG def
Pt presents with recurrent sino-pulmonary infections and diarrhea chronica, due to bacerial infection. Quantitavive measurment of serum Ig will help determine Dx.

Septic Abortion
Is to be suspected in ALL abortions outside hospital who come bcack with pain and vaginal discharge. Its managed with cervical and blood sampling, IV antibiotics and gentle suction cuertage.

Septic arthritis - 2
Acute inflamatory monoarticular arthritis in a previously damaged joint suggest SA. Leukocyte cont of >50K or 100K in synovial fluid.****Like Osteomyelitis in a child, SA is also hematogenous in origin and occurs after URI , The mc organism in children is Aureus and strep. Charcterized by severe pain,leukocytosis,elevated ESR. Pt refuses to walk,keeps the limb externally ROTATED. US guided aspiration shoould be done immediately. Fluid >100,000 is dx. Empiric antibiotics shoul dbe given immediately. In Infants Nafcillin+3rdgen cephalos, and for >5yo Nafcilliln alone. Its a true surgical emergency and needs surgery and drainage immediately, a delay of 4-6 hors can lead to Avascular necrosis.

Septic shock
To treat the induced metabolic acidosis, IV normal saline+Vasopressin+antibiotics is the tx. Use Sodium Bicarbonate is only recommended for pH<7.2, and then its only given to raise pH to 7.20 and then we use other means to normalize it.

Serous otitis media
is the mc middle ear pathology in pts with AIDS. Its charachterized by hearing difficulty, dull tympanic membrane, air bubbles. DDX with Chronic OM is that COM has purulent aural discharge. Tympanic membrane is thickended with calcific pathches and perforation. Otosclerosis occurs in 3rd decade of life and there is absence of stapedial reflex. Its limted to temporal bone.


Severe Combind I D
A lifethreatning syndrome. Presents with recurrent sinupulmonary infection, oralcandidiasis,persistant diarrhea,opprtunistic and viral infectins. Dx is confirmed by: Asence lymph node and tonsils, lymphopenia, absent thymic shadow on cxr, and abnormal T & B cells & natural killer cells. DDX1:Common Variable ID, or Acquiired Hypogamaglubinemia is similar to Burtons, however less sever symtoms and at a later age like 15-35, Ig A/G/M/E may be decreased but no absence of B cells. DDX2:Bruton's Agamaglubulinemia, X-linked, Pt is a male infant asyptomatic until 6mo of age, then recurent pyogenic infection ( S. penuonia, H.inf), Dx is Decreaed Ig A/G/M/E along WITH decreased B cells. DDX3:Wiskott Aldrich, X-linked, Pt is a young boy with Eczema, thrombocytopenia and recurrent infection with capsulated germs. Initail manifestation is at birth with petechia,bruises, bleeding from circumcision or bloody stools. Low Ig M/A/E levels, redueces T cells and platelets. DDX4:Chronic Granulomatous Dis, Defet of phagocytic cells due to dysfunctin of NADPH oxidase enzyme complex, leading to recurrent and uncontrolled infection with catalase positive organism, the mc infectin are lymphadenitis, abscesses of skin adn liver, Lymphocytes are normal, Dx is by Nitro Blue Tetrazolium test.

Sheehan syndrome
Develops due to ischemic necrosis of pituitary gland. Impaired ADH secretion casues full blown Dibetes Insipidus.

Shingles
After initial infection with VZV ( chicken pox ) latent infection is establised in the sensory dorsal ganglia. Characterized by vesicular eruption that occurs in a dormatomal distribution.

Shock - 4
From First Aid, How to Dx shock? 1-Look at CO, if <5,>18 its Cardiogenic shock(Tension pneumothorax,CHF,Temponade,arrhythmia). 2-If CO is >5, look at fever, if there is fever its Septic, if there is no fever, its Neurogenic Shock(or Anaphylactic shock. Bee sting,medication, food alergy). *****Hypovolemic: Has 4 stages depending on degree of blood lost. 1-CLASS ONE: 10% loss of blood (about a unit). Pt is alert but a little lightheaded. BP is normal. Skin and organs are not affected. Pt respondes by tachycardia. This is the first response to compensate in hemorrhagic shock. Physiologically this response is affected by secretion of catecholamines and an insrease in sympathetic tone. 2-CLASS TWO:20% blood lostt. He is like to be confused anc combative. Althogh mean arterial BP is normal, Pulse pressure (systolic-diastolic) has narrowed. Urine output has decreased. Skin is cool and moist. His mental status reflects fight or flight. The change in PP is consistent with increased vasoconstriction. 3-CLASS THREE:35-40% blood lost and can no longer maintain BP. . He will maintain CO by further increasing HRand increasing after load. His mental status is stupor. 4-CLASS FOUR:if blood loss is >40%. pt goes to coma and BP is incompatible with life.**** 1-Septic: results from decreased systemic vascular resistance due to significant vasodialation. Hyperdynamic circulation leads to elevated CO. Normal MVo2 results from hyperdynamic circulation. So there is a)elevated co, b)low systemic vascular resistant, right atrail pressure and Pulmonary Capilary Wedge Pressure. c)Normal Mixed Venous oxygen concentration. 2-Cardiogenic : has increased PCWP (too much blood to pump due to shock). 3-Neurogenic: has low MVo2 due to increased oxygen extraction by hypoperfused tissues. Low left vent preload is also characteristic . 4-Hemorrhagic shock presents wth bleeding.***Hemorrhagic Shock: happens in trauma setting, most propabely from ruptured abdoinla organs bleeding. Either ER US or Diagnostic Peritoneal Lavage should be considered in UNstable pt, onve IV fluids are connected. CT is indicated in stable pts. with suspected solid organ injury. REMEMBER muffled heart sounds are common in severe hypovolemiaand DONT necessarily indicate Pericardial fluid, moreover Cardiac Temponade is unlikely in the absence of JVD. So problem is most likely ruptured abdominal organ.***Septic shcok:both RA presure and PWCP are low. Hypovolmic Shock:Both RAP and PWCP are low. ****hypovolemic shock, 1st parameter to change is pulse rate. *****Hemorrhagic shock: Pt presents with 70/0 BP and CVP of 0. If pt is Unstable with suspected intraabdominal bleeding the best dx procedure is Peritoneal Lavage. USG is actually the best option. The pt may ultimately need exploratory laparotomy but 1st step is DPL. If pt is stable then CT is the best option for dx.

Shoulder Dislocation
Tonic-Clonic seizures may cause posterior dislocation os the sholder. It presents with internaly rotated arm and pt can not do external rotation os the arm, and there is intact sensation and reflexes. Anterior dislocation occurs with sensory loss and Pt can rotate the arm externally. Axillary nerve is the mc nerve injured in this situation.

Shy-Dragger synd
Characterized by 1-Parkinsonism, 2-Autonomic dysfunction(Postural hypoension, abnormal sweating, bowel or bladder control problems, abnormal salivation or lacrimation, Impotence or gastroparesis). 3-Widespread neurological signs. Always suspect it when a pt w Parkinson experiecnes postural hypotension, impotence or incontinence. Anti-parkinson druga are ineffective, and tx in aimed at intravascular volume expansion. DDXM Neuropathy, can cause the symptoms but it happens in a pt who is not controlling blood sugar and it takes a while to develop.

SIADH - 3
Asso w small cell carcinoma of the lung. Severe hyponatremia(specialy when pt has CNS symptoms like disoriented and irritable) requires aggressive mgmnt with IV 3% saline w/wo Furesamide. Rapid correction should be avoided cause it can lead to pontine myelinosis. Summary of the mngmt of hyponatremia due to SIADH : 1-Mild(Asymptomatic w Na 120-130) = Fluid restriction, 2-Moderate(Asymptomatic w Na 110-120) =Loop diuretic+Normal saline, 3-Severe(CNS symptoms) = Hypertonic saline(3%).*****HYpotonic hyponatremia with euvolumia, Low plasma osm and high urine osm. before the dx of SIADH is entertinaed, hypothyroidism and adrenal insuficiecny should be ruled out. One of the causes of SIADH is NSAID therapy. DDXI, polyuria,polyduspsia, hypernatremialow urine osm and high serum osm. *****Tx steps:1-Water restriction is the 1st step in managing hyponatremia. 2-If the pt sodium is below 120 or if he is seizing, emergency tx is administration of 3% Sodium Chloride solution (solution of hypertonic saline) to raise serum sodium to 125 . 3-If the pt has evidence of fluid overload, Hx of CHF or is resistant to tx then Furesamide maybe added. 4-Chronic tx may involve Lithium or demeclocycline, which inhibit ADH action.

Sialolithiasis
Sialolithiasis or calculus in the ductal system of salivary gland is mc in Submandibular gland followed by sunlingual and parotid. Pt will have recurrent Sialoadenitis (infection of salivary gland). Sialolithiasis presents as postprandial pain and sweling. Stones in Wharton duct are radioopaque, so xray is dx. Tx is dialation and incision of involved duct to remove the calculus.

Sick Euthyroid syndrome
Pts with any acute severe illness may have abnormalities of throid hormones and TSH in absence of any underlying thyroid abnormality. This is called SES. The mc pattern is a fall in total and free T3 and normal T4 and TSH levels.

Sick sinus syndrome
Test#6. Q=22. Tx is permanat pacemaker.

Sickle cell disease - 6
Sicke cell trait is most commonly Asso with Painless hematurea in black male. *****Suplementation of Folic acid is recommended in ALL pts to prevent occurance of aplastc crisis. Aplastic crisis happens for two reasons, 1-Folate def, 2-Infection with Parvo virus. Note:Hydroxyuria is used when painful crisis happens frequesntly, it produces its effect by increasing HbF levels.*** May cause acute painful crisis. adequate hydration and oxygen therapy is necessary to terminate the episode. Hydroxyuria is used when frequent painful episodes interupts life. Acute Vasoocculsive crisis is an important complication that may cause Stroke, PRIAPISM. Exchange transfusion is indicated whenever Oclussive crisis occurs.****Pts with SC encounter constant clumping of sickled cells, leading to repeated microinfarctions in the spleen. By 2-3 of age, they have functional Asplenia. Since one of important functions of spleen is to remove CAPSULATED organism, like Pneumococcus, and H.Influenza, pts will be at risk of infection with these organisms. For this reason, antibiotic prophylaxis and vaccination against Pneumonoccal and H.Inf are given standardly to pts. with functional Asplenia. So if a 5-yo boy persnts with high grade fever, hypotension, altered mental status, elevated WBCand Bandemia, its sepsis due to Pneumococcus.****Acute painful Crisis:is due to vasoocclusive events. First step is hydration and opiod analgesics. If pt has frequent acute crisis, then Hydroxyurea is indicated. This is a cytotoxic agent that increases Hb F by simulating erythropoisis. Since the function of HbF is to retard sikcing leading to decresed sicking and less occlusion of vessels by sickled cells. Now Folic acid is helpful in all SC pts, becuase it helps with eryhtropoasis, its helpful in preventing aplastic crisis. If aplastic crisis happen, blood transfusion is indicated. *****when pt present with acute severe anemia, they may be Aplastic Crisis, Splenic Sequestration Crisis or Hemolytic Crisis. Aplastic crisis result from transient arrest of erythropoesisand mf asso w infections, the mc is B19. Hay sudden drop of Hb concentration and absent reticulocytes. Tx is blood transfusion. Splenic Seq occurs in pts who have not yet had splenectomy. Characterized by vaso constriction and pooling of blood in spleen. Hay marked decrease in Hb concentration but persistant reticulocytosis. Pt develops rapid splenomegally, severe hypotensive shock. Tx is spleenectomy to prevent recurrences. ****Acute Chest Synd are chest pain, fever, and infiltrate in cxr. It is one of the mc complications of SC.

Sideroblastic Anemia
Results from defective heme synthesis, most commonly due to Pyridoxine-dependant impairment. Specially if pt is taking Isoniazide. It manifest as hypochromic microcytic anemia simulating iron def anemia. But iron studies reveal elevated iron and increased TIBS (DDX). Tx is pyridoxine (B6). MCC is Chronic Alcoholism. Smear shows Iron granules in blue around the nucleus.

SIDS
Sudden Infant Death Synd. Infants should be put on their back(SUPINE) while sleeping to reduce mortality. They are the leading cause of infant mortality from 1mo-1yr.

Sinusitis, Acute Bacterial
MC Pediatric bacterial agent is Strep pneumonia, then H. Influenza, then Moraxella catarrhalis. Dx in children <6 is based on clinical rather than radiological criteria. For uncomplicated sinusitis, Amoxicillin. Presents with persistnat thick nasal discharge, nasal congestion, HA, cough, low fever, maxillary sinuses are tender. Complicated sinusitis could present with orbital complications and abscess. Do a CT to Dx or rule out complicax.

Sjogren- 6/8
An autoimmune chronic dysfunction of exocrine glands. Pt devlops dental carries due to salivary insufficiency. Xerostomia leads to difficulty swalowing and talking. Eye show keratoconjunctivitis. Lab shows, anemia,leuukopenia,eosinophelia and elevated ESR. Lip biopsy is the ONLY specific dx technique.

SLE - 8
MCC of death is chronic renal fialure. ****DDx b/w SLE and RA: Arthritis in SLE is non erosive and in RA its erosive. Kidney is damaged due to immune-complex deposition (type II). "Goodpasteur" damage is caused by type III. "Methicilin interestitial nephritis" caused by IgE (Type I). "DM & Hypertension" are caused by non immune reasons. When there is evidence of kidney failure in SLE, the next step is to do renal biopsy and therapy is directed to the pattern of the glomerular involvement. There are five patterns. Type I & II no need for tx. Type IIi&IV need immunosuppresants. Mainstay of tx is IV methyprednisone. If it didnt responde then Immunosuppresants (Cyclophosphamide) is given. In SLE pt with active Nephritis OCP should be avoided. ****SLE presents with 5 types of renal involvements:1-Mesangial, is the earliest and least severe. 2-Focal, is more severe than Mesangial, there are proliferative changes and areas of necrosis. 3-Diffuse proliferative, is the mc type and the severest type. It has the worst prognosis. There is Hematuria, proteinuria, renal insufficiency, hypertention, hypocomplementemia, marked elevation of anti-DSDNA. 4-Membraneous GN, renal function is preserved, basement membrane is thickened, and subepithelial deposits are present, it has a better prognosis than Diffuse. 5-Sclerosing type, presents healing of previous inflamatory damage, urine is normal and immunosupresive drugs are not effective since hay no inflamation.****Tricky Q, if the vgnette says no anti-sm antibodies that does not rule out SLE because 30-40 of pt dont have it. SLE pt could have: muskuloskeletal hematological, skin,lungs,kidney and serous membran problems. If a pt has 4 or more its dx. DDX with Mixed CTD is Anti-RNP antibodies.****Renal involvment in SLE is due to immune complex mediated glomerular injury. These complexes are primarily composed of Anti-DsDNA antibodies. Remember Anti-Sm antibodies remain elevated even when dis is no longer actve. Anti-RO antibodies are asso with Neonatal Lupus.*****DDX b/e RA and SLE is that SLE has non erosive arthritis.****Scarring Alopecia is the skin related problem in SLE pts.

Sleep Apnea
First line of tx is Weight loss,avoidance of sedatives (benzos) and alcohol, and avoidance of supine posture during sleep. Other options are Positive Continues Airway Pressure. If all fails then Tracheostomy is the optin. Dx is with polysomnography.

Sleep terror disorder
episodes of sudden fearful waking . Later they dotn remmeber it. DDX:Nightmare dsorder, pt remembers in detail the dream.

Slipped Capital Femoral Epiphysis - 2
SCFE Is a result of failure of the growth plate b/w femoral neck and femoral head, resulting in displacement. Its seen in adulescence with overweight problem. Pain & limp are presenting complains. Dx is by Xray. Must contain Hip, and both front 'AP' and Lateral 'frogs' views. SCFS can progress to Avascular Necrosis if untrearted, xray will show bone cyst or sclerosis. If AN is in advanced stages, xray shows collapse of the bone.Pt needs urgent surgery in SCFE.****Its a medical emergency the prompt surgical intervention is needed to prevent the two cmoplications of Avascular necrosis of the femoral head and Chondrolysis. Surgical fixation of the hip with screws is the tx of choice.

Small Cell Carcinoma Lung
Asso with SAIDH. SCC has usually metastacised by the time its discovered.

Smoking cessation
Use Bupropion.

Social Phobia
pt is isolated because he is afraid of being in public, He can speak infront of people and gets anxous evenin small parties. Tx of choice is Assertiveness training (Cognitive&Behavior Therapyor CBT) plus SSRI.

Solidary pulmonary nodule
T9Q8

Somatization disorder - 2
Pt presents with pain and symptoms in : 2 GI, sexual or reproductiive symptoms, and one pseudoneurological symptom (HA,blindness, deafness, weakness.). It presents before age of 30 and in mostly females.*** If pt asks for CT, tel her you woud like to see her on regular basis before CT scan.

Specific Phobia - 2
Pt is afraid of swimming pool. The besr approach is Systemic Desensitization. Beta blockers are for Performance anxiety. Benzodiazepin is for acute panic attack. Cognitive-Behavior therapy is for chronic Panic attack and GAD.****Pt 9 yo doesn’t go tro shcool and he says he s afrid of Dark clouds. Tx is CBT (congitive-behavioral therapy) that repeatedly exposes th ept to the object.

Spherocytosis - 2
Due to RBC membrane defect mostly from Spectrin deficiency. In PNH defect is in Decay Accelerating factor. In Thalasemia absence Beta chain synthesis is the problem. Malabsorption results in B12 and Folate def which is the cause for Macrocytic anemia. In Sickle Cell there is substitution of Glutamin for valine amino acid in polyeptide chain. ****Autoimmune Hemolytic Anemia: Pallor,Jaundice,elevation of indirect bilirubin & reticulocye count & LDH are all indicative of AHA. Splenomegally is asso with ExTRA-vascular hemolysis, while descreased level of Haptoglobin is indicative of INTRA-vascular hemolysis. AHA pt have decreased Haptoglobin and Intravasculat hemolysis. Hereditary Spherocytosis pt have Extravasculat hemolysis. Its an aduto dominant so there MUST be family Hx in vigniette. MCHC is elevated, Osmotic Fragility test is positive, but its ALSO positiv ein AHA. . Now in AHA Coomb's test is also positive. But since its not always positive in all cases of AHA we do Micro-Coombs test to confirm the dx of AHA. So bottomn line, spherocytosis may also be seen with AHA and Negative family hx and positive Coombs test , thes DDX AHA from Spherocytosis.

Spider bites - 2
1-Black Widow spider: causes acute abdomen that is tx with combination of Muscle relaxant+Calcium Gluconate. 2-Brown recluse spider: Produces extensive localized skin necrosis resembling Pyoderma Gangrenosum. Dapsone is used to reduce the extent of local necrosis in pts who have been screened for G6PD deficiency. ****Brown spider bite, pt has an ulcer on the thigh, the mc complication is deep ulcer with necrotic center and erythomatous halo. Local excision is the tx of chouce. In BLACK WIDOW spider ther mightbe abdominla rigidityand muscle cramps. Pts develop N&V and initial site of bite is not seen. Tx is Calcium glucanate and muscel relaxants.

Spinal Cord Compression
Or Cauda Equina Synd. Pt presens with absent rectal tone, urinary incontinence, motor and sensory loss in extremities. This is a surgical emergency. Do MRI to determine the site of compresion. It may occur in males repeatedly due to prostate cancer that has metastasized to spinal column.

Spinal Cord Injuries
1-Anterior Cord Synd: Asso with BURST fracture of the vertebra characterized by total loss of motor function beloe the level of lesionwith loss of pain and temperature. On both sides below the lesion. MRI is the best dx method. 2-Central Cord Synd: Burning pan and paralysis in upper extremities with relative sparing of lower extremities. Seen in elderly due to hyperextension of the neck injury. 3-Brown Sequard synd: Ipsilateral motor and proprioception and loss of contralateral pain below the level of lesion.

Spinal Stenosis, Lumbar - 2
It can present with Neurogenic claudication. The pt presents with calf pain when he stands up and walks. . Neurogenic claudication from spinal stenosis. Its POSITIONAL Intermittant Claudication , pain does not occur at rest while standing or in certain position, it happens only pt walks. Dx is MRI. DDX1:Cada Equina synd (tumor), causes urinary retention or overflow incontinence. DDX2: Leriche Snd, presents with atherosclerotic vascular dis, impotence and intermittent claudication. DDX3: Arterial Insufficiency is not positional. Dx is Arteriography and Doppler.

Spleenic Contusion:
Asso with fracture of the left lower ribs. Breath sounds are normal and no respiratory distress.

Spleenic Trauma
The immediate mngmnt caused by blunt abdominla injury depends on hemodynamic status and response to IV fluids. If initilay he is Unstable, but improves with fluid, Next best step is to get a CT. If initialy Unstable and iv fluid doesn’t help, Emergent Exploratory laparotomy is indicated. If CT shows no operation is required, then admit him to surgical ICU for monitoring.

Spondylolesthesis:
Is a developmental disorder characterized by a forward slip of a verebra, that usually manifest in pre adulescent children, Back pain, neurological dysfunction (urinary incontinence) and a palpable “step-offâ€
 at the lumbosacral area are present if the disease is severe.

Spontaneous Bacterial Peritonts
Always suspect SBP in cirrhotic pt with fever and ascites. SBP is peritonitis in the absence of an apparent source of infection. SBP is almost always seen in adults 2ary to severe cirrhosis especially alcoholic cirrhosis. It almost always occurs in pts with ascites. Its characterized by growing a single bacteria in ascites fluid, usually E.Coli. Clinical features include unexplained fever in a pt with cirrhotic ascites and abdominal pain. Ascites fluid containing PMN>250/L, SAAG(Serum to Ascites Albumin Gradient) >1.1 and G- bacili in a cirrhotic pt is highly suggestive of SBP. Presence of >10000WBC/L ,multiple oraganism, or failure to improve after 48 hr therapy is suggestive of 2ary Peritonitis. Suspected SBP should be reated empiricaly with Cefotaxim or an Ampiciline with an aminoglycoside. Recurrence is very common.

Sporotricosis
Garderners dis, postules and painless ulcers at finver tip.Tx is ????

Squamous CC Lung
Hypercalcemia nd hillar mass in a smoker of more than 45 yo is most likely SCC of lung. Even though hypercalcemia usually develops due to metastatic bone involvement , its also because PTHrP (r=related, p=protein). Its like PTH and bind to same receptor and increased absorption of Ca in distal tubules.

Squamous CC Skin - 5
The single most important factor is sunlight exposure. Its associated with Leukoplakia (a whitish patch in tongue that is hard to remove and its granular, an incisional biopsy or cytology should be done). Appears as an ulcer that does not heal, on the LOWER lip. Characterized by invasive cords of squamous cells with Kerai pearls. Differential diagnosis includes many types of benign and malignant lesions, including basal cell carcinoma, keratoacanthoma, actinic keratosis, verruca vulgaris, and seborrheic keratosis.***Whenver an open ulcer fails to heal after a period, you need to biopsy to ensure that it han not degenerated into SCC. These ulcers are known as MARJOLIN's ulcer. Also remember Aktinic Keratosis is a recursor to SCC.**** When ulcer dosnt heal, suspect SCC, nest step is PUNCH biopsy. If SCC, WIDE excision is the Tx of choice.****Actinic Keratosis turns into SCC.

Statin intolerance/Toxicity
CPK elevation of more than 10 times in presence of myalgias/myopathy. Rhabdomyalysis in an Acute renal failure could be a consequence. Tx is stop the drug and supportive therapy for Rhabdo.

Status Epilepticus
An emergency. Mgmnt step are: 1-Place pt lateral with mandible pushed forward. 2-medication started. 3-if medication failed after 30 min , then general anesthesia and intubation is indicated.

Stevens Johnson's
Target shaped mucocutanous lesions and systemic signs of toxicity. Pathology involves immune complex mediated hypersensitivity.

Still's dis, Adults
Is a variant of RA. Pt presents with 20-30yo, high spiking fevers with CHARACTERISTIC salmon colored rash, arthralgias, arthritis, Leukocytosis. DDX1:Parvovirus (Slapped cheek dis), malar,eryhtomatous rash, arthralgia or arthritis. DDX2:Henoch-Schonlein purpura, in children, rash,abdominla pain,arthralgias and renal dis. Rash is pruritic and involves lowerlegs or Buttucks.DDX2:RF, PECCS. Erythema Marginatum is Evanescent(tending to vanish)erythomatous non pruritic mostly on the trunk.

Stomach cancer
The only malignany that has decreased universaly. The reason is not known.

Strabismus:
The mcc of amblyopia (decreased visual acuity) is strabismus. The mc type of strabismus is esodeviation (medial deviation of th eye). Tx is to cover the normal eye.

Stranger Anxiety
When a child is left in unfamiliar places (day care). From 6-8 months and peaks in 12-15 months. DDX:Seperaion aniety is with older childs when seperated from a love one.

Stress Fracture
Bone pain at rest, worsens with exercise, swelling and point tenderness, Xray is normal at initial stage and MRI is dx, Tx is restrictive weight bearing along with short leg cast, healing takes 3-4 weeks. It occurs in young dancers's legs.

Stroke - 6
Stroke can be 2 types, Hemorrhagic (intercerebral or subarachnoid) or Ischemic (secondary to thrombosis, embolism, or systemic hypoperfusion). Blood supply of the brain is 1-Anterior vasculature, internal carotid A and its branches Anterior and Middle cerebral A. 2-Posterior vasculator, paired Vertebral A and they join to make Basilar artery, which divides to make Posterior cerbral Artery. Now Deficiencies produce: 1-ACA:Contralaterla motor and sensory deficit wich is more pronounced in the lower rather than upper limbs.Urinary incontinence, gait ataxia. 2-MCA: Contralateral motor and sensory deficitwhich are more pronoucedin the Upper rather than lower limbs, and homonomous hemianopia. If the dominant lobe (left) is involves hay aphasia, and if non-dominant (right) is involved hat neglect syndrome and anosognosia(pt cant tell is there has been an injury to the body). 3-PCA: Homonomous Hemianopia, ataxia w/o agraphria, visual hallucination(Calcarine cortec), sensory symptoms (thalamus), third nerve palsy with paresis of vertical eye movement and motor deficit (mid brain). 4-LACUNAR infarcts: are small non cortical infarcts caused by occulsion of a branch of a cerebral artery. Risk factors are hypertension, DM or Polycythemia. They usually dont produce both motor and sensory symptoms but rather have well recognized syndromes, like pure motor hemiparesis, pure sensory stroke, dysarthria-clumsy handor ataxic hemiparesis. *If a pt presents w/i 3 hours of ischemic attack, thrombolytic therapy with tPA (after CT scan) should be started. Dont try to regulate BP first, it might impair autoregulation and make it worst. Streptokinase does not help either. tPA is tx of choice. * THALAMIC stroke:Involves VPL part of thalamus that transmits sensory info from contralateral part of body. Presents with hemianesthesia accompanied by hemiparesis, ahtetosis. Dysesthesia (numbness and tingling burning feeling) of the area affected by the sensory loss is chracteristic, and is called thalamic pain phenomenon.****HT has the highest risk factor for Stroke, more than smoking,alcohol,hyppercholesterol.....****If pt presents with hemiparesis and speech difiulty like broken words then its Expressive aphasia related to Dominant frontal. But if he cant understand what is being said to him then it is Parietal dominant(Dyscalculia,Dysgraphia). ****If a pt comes in with hemiplasia, you need to first ddx b/w Hemorrhagic and Ischemic stroke and then give medication. To DDX you need to do CT WITH OUT contrast, if its ischemic (thrombi) then we give aspirin and then we do carotid Doppler and TEE to evaluate source of embolism.

Stuge-Webber Synd:
A neurocutanous condition. Its auto dominant therefore doesn’t affect multiple generations. Pt will have a cerebral lesion on the same side as facial nevus. Also exopthalmus due to ICP. Dx is CT. The cerebral lesions are nevi involving the leptomeninges and are thus similar to the facial lesions.*****Caverness unilateral hemangioma is another presentation. Skull Xray after the age of 2 years shows gyriform intracranial calcifications that resembles the tramline. Tx ia aimed at controlling seizure and reducing ICP. Laser therapy w Argon to remove skin lesions.

Subacromial bursitis
It refers to inflamation of the subacromial bursa. It occurs in athlets as pat of impingment syndrome. Its characterized by shoulder pain, which is absent at rest but present at overhead activity. Range of movement is limited by pain. Neer sign (pain on passive internal rotation and forward flextion at shoulder) is present. US or MRI confirms Dx. Tx is conservative with NSAID, physical therapy. DDx is Tear of long head of bicept tendon, which will lead to bulging muscle mass in the middle of the arm.

Subacute Combined Degen
T9Q42. Tx is B12

Subarachnoid hemorrhage –4. 6/3
Rupture of aneurysm is the most frequent cause. Hypertension is the mcc for Intracerebral Hemorrhage. They are most prone to rupture when they’re >7mm. Pt should be evaluated with cerebral angiography and treated surgically. Pt should be evaluated by cerebral angiography and treated surgically. AVM is the mcc of SAH in children, the hx of seizure and migrain like headache is characteristic.****Vasospasm is the major cause of morbidity and mortality in pts with SAH. Calcium channel blockers (Nimodipine) are used to prevent spasm in these pts.****SAH or "cerebral-salt wasting syndrome". Patholgy involves 1-SIADH (inappropriate vasopressin secretion) which causes water retention. 2-an increased secretion of an atrial/brain natriuretic peptide. SIADH also results in hyponatremia for which water restriction is the tx of choice. So Hyponatremia is one of the important complications of SAH.

Subcunjuctival hemorrhage. Ophthalmo. 6/3
Redness in the eye, due to hard rubbing or trauma. Its benign and heels spontaneously.

Subdural hematoma
Tear of bridging veins. Picture shows semi lenticula rhematome. MC in Elderly & Alcoholics. Tx is conservative if no midline shift is present in CT. Tx is centered on prevention of ICP by head elevation, hyperventilation (causes vasoconstriction and thus decreases cerebral blood flow), and if needed acetazolamide and mannitol. If there is a midline shift then Craniotomy is indicated but its asso with grave prognosis. DDX1:Epidural hematoma where Middle Meningeal A. is injured. Biconvex hematoma in CT, non contrast. Lucid interval. DDX2:Hypertensive hemorrhage, putamen & thalamus. DDX3:Subarachnoid hemorhage, rupture of aneurysm.

Sublimation:
A mature defence mechanism that allows for unacceptable impulses to be channeled into more acceptable activities. Like aman w fiery temper who channels his anger into athletic pursuits.

Sumatriptan Tox
CI include Printzmetal Angina, CAD, Pregnancy. So is a femlae of child baring age wants it make sure she is not pregnant.

Superficial thrombophlebitis:
Dull pain in the region of the affected vein, eryhtema, induration and tenderness along the vein. High fever and chills and rigor is suggestive of septc phlebitis. Presence of edema and deep calf tenderness is characteristis of DVT. ST is not risk factor for Pulmonary Embolism. Localized ST is treated wth bed rest, heat and NSAID

Superior Vena Cava Synd
Pt presents with Venous congestion of face and arms. The mcc of superior vena cava onstruction is Bronchogenic carcinoma(Small cell tumor) (smokers). Today Angioplasty with stenting is choice.

SVT:
Narrow QRS, HR>140, regular, loss of P wave. If pt is unstable, cardioversion. If stable, vegal maneuvers initially, if failed then IV Adenosine. Verapamil is 2nd DOC.

Supraclavicular fracture:
It compromises brachial artery resulting in bradial artery pulse loss.

Symptomatic Bradycardia
Pt has HR of 40, dizzy,lightheaded, clamy extremities, but no dyspnea or chest pain and no hypotention. He is having severe symptomatic bradycardia dn the tx is iv Atropine.If that didn’t work net step is Transcutaneous pacing. If pt has bradycardia AND Hypotension, then tx is Epinephrine. Reember Adenisine is used for SV tachycardia.

Syncope - 3
Know basic pathophys mechanism and different types of Syncope. The most common pathophysiologic basis for syncope is an acute decrease in cerebral blood flow (with resultant cerebral hypoxemia) secondary to decreased cardiac output; arrhythmias, including conduction abnormalities, are the most frequent cause. 1-Exertional (effort) syncope suggests cardiac outflow obstruction, mainly due to aortic stenosis. 2-Syncope of cardiac etiology typically begins and ends suddenly and spontaneously. It is most commonly due to an arrhythmia. 3-Vasovagal (vasodepressor) syncope is typically precipitated by unpleasant physical or emotional stimuli (eg, pain, fright, sight of blood), usually occurs in the upright posture, and is often preceded by vagally mediated warning symptoms. 4-Syncope due to seizures is abrupt in onset and is associated with muscular jerking or convulsions, incontinence, and tongue biting. 5-Syncope due to pulmonary embolism usually indicates massive pulmonary vascular obstruction and is often associated with dyspnea, tachypnea, chest discomfort, cyanosis, and hypotension. 6-Syncope of gradual onset (with warning symptoms) and slow clearing suggests metabolic changes, eg, hypoglycemia or hypocapnia of hyperventilation. ***7-Situational syncope, Typical senario would include a man middle age who looses consiousness immediately after urination or during coughing fits. The pathophysiology involves autonomic dysregulation, which can be explained by staining or rapid bladder emptying. ****Vasovagal syncope dx is with upright tilt table testing which includes Carotid sinus massage.

Synovitis of the hip joint
Bed rest with the hip joint in a comfortable position is the tx of choice.

Syphilis - 4
Once Dark field microscopy shows the spirochet and its positive, no need to do VDRL or FTA-ABS. But since the pt is at risk of HIV we need to do Elisa for HIV screening.****PRIMARY: Presnts with Painless, shallow CHANCRE ulcer WITH PUNCHED OUT BASE AND ROLLED EDGES and painless bilateral lymphadenopathy. The best dx test is Dark field microscopy. **Secondary: presents with Condylomata lata, highly contagious.****** In pt who is allergic to Penicillin, give Oral Doxycycline.

Syringomyelia
Suspect in a pt with upper extremity areflexia weakess and associated anesthesia in a "cape" distribution. When syringomyelia is associated with Arnold-Chiari malformation, there is caudal displacement of cereberal tonsilsthrough the foramen magnum

Systemic Sclerosis
The cause of pulmonary complication is interestitial fibrosis not tighning of the skin. Prolonged oral administration (> 1.5 yr) of penicillamine (0.5 to 1.0 g/day) can reduce skin thickening. For renal disease, ACE inhibitors are the drugs of choice.

Tachycardia, Vent
Regular (Wide Complex) Ventricular tachycardia has 2 types of tx. 1-If pt is hemodynamically stable (BP is normal, K normal, no bleeding) then DOC is IV Amiodarone, or alternative is Lidocaine. If pt is not stabalized then cardioversion is choice. Digoxin is CI in VT, Its used for Atrial Tachy. For SVT Carotid massage is chioce.

Tamoxifen - 3
An antiestrogen drug used for breast cancer. When used as adjuvent therapy for early stage disease it reduces the risk of recurrence of original cancer and new cancer in other breast. However it increases the risk of 2 types of cancer, 1-endometrial (lining of Uterus) and Uterine Sarcoma. ***It reduces the risk of breast cancer in those who are at increased risk for developing breast cancer, ITS PROVEN. SEE RALOXIFEN.****It increased the risk for endometrial cancer by 1% and ONLY in postmenopasusal women. It decreased risk of breast cancer, so overall it reduces mortality rate. It also decreases risk facto in the opposite breast. It protects against osteoporosis. However it does cause hot flashes and vaginal dryness due to its antiesterogenic effect. Remember its mixed agonist-antagonist on estrogen receptors.

Tay-Sachs dis:
Def of Hexosaminase and accumulation of GM2 gangliosidase, particularly in CNS. Pt presents with hyperacusis, MR, seizure, chery red macula but NOT hepatomegally or lymphadenopathy.

TB -2
1-Induraation of 5 or > is + in: Close contacts of TB pt, HIV pt, Organ trnasplant, chronic steriod therapy. 2-Induration of 12 or > is + in : Immigrants recent, IV drug users, homeless person, prisoners and healthcare workers. Once PPD is positive, it does not mean pt has TB, they have to undego Cxr. If TB is diagnosed then full therapy, if TB is excluded then 9 months of Isoniazide prophylaxis. TB is the mcc of Constrivtive pericarditis in immigrants. It should be considered in pt with unexplained elavation of JVP and hx of predisposure.*****Erythema induratum are nodules in the sheen and calves. They are small tender erythomatous nodules.

TCA intoxication - 4
T9Q15. tx is sodium bicarbonate prevents arrythmia by alleviatng cardio-depressant action on sodium channel. Asso with QRS widening on EKG. It also helps correct acidosis. If pt presents with seizures that need tx we give Diazepam.

Tennnis Elbow
Or Laterla Epicondylitis, is condylitis about the origin of extensors of forearm. Characterized by point tenderness over the lateral epicondyle of humerus and exacerbation of pain by extention of the wrist agaisnt resistance (hitting the ball). DDX with Radial tunnel syndrome which could coexist, pain is produces by simultaneously extending the wrist and ringers while the long finger is passively flexed by the examiner.

Testicular cancer - 2
1-Seminoma: elevated Placenta Alkaline Phosphatase. 2-Embryonal: elevated Alpha Feto Protein (AFP). 3-Choriocarcinoma: elevated beta-HCG. Once the dx of carcinoma is made, US shows solid nodule, then initial mngmt is Orchiectomy. Trans scrotal biopsy and FNA is CI becauseit might spread lymphatically or hemotgenously. ****Leydig cell tumors are the mc type of testicular sex cord stromal tumor. Testosteone and Estrogen are markedly increased with 2ary inhibition of FSH & LH. Pt prestns with Bilateral Gynecomastia. DDX1:Choriocarcinoma, bHCG is increased. DDX2:Seminomas contain Syncytiotrophoblastic giant cells.DDX3:Yolk sac tumors show increased AFP.

Testiclar feminization:
Defect of absence of androgen receptors resulting in feminine phenotype with 46XY genotype. The MIF is produced by the gonads, so th euterus, vagina and tubes are absent. Breasr develop due to peripheral production of estrogen, whereas axillary and pubic hair does not. Tx is to resect testicles and make Vagina.

Tetanus guideline
Hx of Tetanus Immun Clean Wounds Dirty looking wounds
<3 doses of tentanus toxoid in past TT:Yes, Tig:No TT:Yes, TIg:Yes
>=3 dose of tet toxoid in past TT:Yes if last dose >10y ago, TIg:No TT:yes if last dose>5y ago, TIg:No

Tetracycline Toxicity
A5- Photosensitivity.

Tetralogy of Falot - 2
Most prominent feature is cyanosis that rarely improves with oxygen. A Classid presentation is Squanting that improves cyanosis. "Tet" spells are hypoxic episodes characterized by rapid breathing. Immediate tx is Oxygen and put the child in a Knee-Chest position. followed by fluids,morphine,propranolol. TOF is a Cyanotic (early) condition. ASD & VSD aren't cyanotic (late).****Pansystolic murmur, Hepatomegally.

Tetsticular feminization
defect or absence of androgen receptor results in feminine phenotype with 46XY genotype. MIF is produced by gonads, so urtus and vagina are absent. Breat develop because peripheral production of estrogen , whereas axillary hair and pubic hair does not. Tx is testicular resection at puberty and creation of aneo vagina. Pt prestns with amenorrhea, developed breasts, absent pubic and axllary hair , absent internal reproductive organs and a 46XY karyotype.

Theophyline toxicity
1-CNS stimulation (headache, insomnia), 2-GI (Nasea, Vomitting), 3-Cardiac toxicity (arrhythmia).

Thiazide SE
Hyper GLUC= Hyper 1-Glycemia, Lipidemia, Uricemia, Calcemia. HypoK and HypoNa.

Thioridazine toxicity - 2
Unlike other antipsychotics, thioridazine is asso with cardiac arrythmias. Symptoms of thio overdose include deep sleep, coma, abnormal involuntary movements, hypotension, tachypnea and arrhythmias. EKG reveals prolong QRS. Pt needs sodium bicarbonate.****Antipsychotics like Thioridazine, galactorrhea 2ary to Dopamine blocking effect. Pt presents with lactation and menturation irregularity.

Thoracic outlet synd
refers to compresion of neuro-vascular structures supplying upper extremity. Pain wakes up the pt from sleep. There is paresthesias ans weakness of fingers. Vascular involvement presents with palor pulslessness and coldness.Cxr, MRI and angiography helps to dx the cause f compression. Conduction velocity studies identify the site of compression and overhead pully excercise helps to releif the compression.

Threatened abortion - 3
Hemorhage before 20week . Cervix is closed,no fetal tissue pasage,feal heart if normal, 25% of women have this. When th pt comes to you first step is to make sure fetus is alive. Once that is fullfiled, mngmt is reassurance and performance of USG one week later. Tx is Reassurance and outpatint follow up. DDX1:Incomplete abortion,sometissue is evacuated, cervix is dialated. DDX2:Complete abortion,whole concep passes theough cervix. Cervix is closed. USG shows empty uterus. DDX3:Inevitable abortion, low abdominal cramp radiated to the back, dialated cervix. USG shows ruptures or collapsed gestational sac with absence of fetal cardiac motion.****Complete abortion is when produce has come out and cervix is closed with blood in vaginal vault.****Cerclage is used to tx or prevent first trimester abortions when the cause is incompetent cervix.

Thrombophlebitis:
Characterized by palpable, indurated, cord-like, tender, subcutaneous venous segments, low fever.

Thymoma - 2
Is seen with CT in ANTERIOS midiastinum. All Neurogenic tumors (Neuroblastoma, are in POSTERIOR mediastinum. Pericardial cyst is in the MIDDCLE mediastinum. Esophageal Leomyomas are in POSTERIOR midiastinum.****Is asso with Red cell aplasia which is eveything except RBC,ESR,Hct,Hb and reticulocyte are normal.

Thyroid carcinoma
MC cancers 1-PAPILLARY(Papillary carcinoma is the most common thyroid cancer (60 to 70% of all thyroid cancers). Females are affected two to three times more often than males. It is more frequent in the young, but is more malignant in the elderly. It is more common in patients with a history of radiation. Hurtle cells exist. Psammomma bodies are characteristic.FNAB shows large cells with groung glass cytoplasm, and the pale nuclei with central bodies and central grooving. The prognosis is excellent even with metastasis.). 2-FOLLICULAR cancer(Follicular carcinoma accounts for about 15% of thyroid cancers and is more common in the elderly. ddx with ADENOMA is that cancers demonstrate invasion of capsul and blood vessels. It is more malignant than papillary carcinoma, spreading hematogenously with distant metastases.). 3-MEDULARY(Medullary (solid) carcinoma may occur as sporadic (usually unilateral) or as familial (frequently bilateral), transmitted as an autosomal dominant trait. Pathologically there is a proliferation of parafollicular cells (C cells) that produce excessive amounts of calcitonin, a hormone that can lower serum Ca and phosphate (PO4), Total thyroidectomy is indicated, even if bilateral involvement is not obvious. Hurtle cells exist.). 4-ANAPLASTIC(Anaplastic carcinoma accounts for 10% or less of thyroid cancers and occurs mostly in elderly patients and in women slightly more than in men. The tumor is characterized by rapid and painful enlargement, and about 80% of patients die within 1 yr of diagnosis). Do FNA for Dx, but FNA cant ddx b/w Follicular cancer and Follicular Adenoma.

Thyroid Nodules - 2
The mc is COLLOID (benign), 2nd mc is FOLLICULAR adenoma (benign). The first step in Dx of a thyroid nodule is is measurement of TSH, the subsequesnt steps (T4, T3) are all dependent on TSH levels. ***** FOLLICULAR Adenoma:histologically demonstration of invasion of the capsul and blood vessels is required. FNAB shows large numbers of normal-appearing follicular cells. Its almost impossible to ddx Follicular ADENOMA from Follicullar CANCER. Unlike PAPILLARY cancer, FOLLICULAR cancer is encapsulated and doesnt have dstinctive nuclear features. Lymph node involvement in Follicular cancer is rare, unlike papillary.

Thyrotoxicosis
one of the mc causes of thyrotoxicosis with reduced thyroid uptake is Subacute lymphocytic (painless) thyroiditis. Leakage of thyroid hormones into the circulation due to inflamatory damage to the thyroid follicales results in Thyrotoxicosis. Most cases happen postpartum. Other causes of TT with low radioactive iodine uptake are 1-Subacute granulomatous (De Quervain)thyroiditis (Asso with intense pain in thyroid), TX is just NSAID. 2-levothyroxine overdose, 3-Iodine induced TT.

TIA - 4
1-Embolic TIA are prolonged and single, they last for hours. EKG shows Atrial Fibrillation or MI. Tx includes avoidance or correction of risk facor like smoking, HT and Hypercholesterolemia. Anticoagulation is considered when TIA is caused by Emboli that arose from the heart. Heparin is administered followed by Warfarin. Transesophageal Echi is performed to clots or vegetations. 2-Atherothrombotic TIA are recurrent and shorter in duration., minutes. MRI and CT shows the site of injury. All pt should get antiplatelet therapy if there is no CI. Aspirin is the initial agent of choice. Clopidegrol is used if pt is intolerant of aspirin. Aspirin is used in combinatinon with Dipyridamole if pt has a hx of TIA despipte being on Aspirin.**** Presents as focal neurologic deficits (right-sided weakness, expresive aphasia) (MS might show same way but CT shows it). There are 3 pathophy for TIA, 1-Blood vessel abnormality(atherosclerosis, inflamation), 2-Embolic source(heart), 3-inadequate cerebral blood flow. Atherosclerosis & Emboli are the mcc in Elderly pt. in Young pt Emboli is the mcc. usually from the heart, due to arrythmia, IE, valvular dis or Myxoma. Less common are venous emboli that reach arterial circulation via ASD or patent foramen ovali. Transatlantiv flights increse the risk for Venous Thrombosis.. To identify the cause Transthoracis Echo is performed. If Echo is normal work up for hypercoagulable conditions should follow. ****All pt with atherothrombotic TIA should get antiplatelet tx if there is no CI to it. Aspirin is the initial agent of choice. If CI exists, Clopidegrol. If CI still exist then Ticlopidine.Now if TIA was caused by Emboli from the heart then anticoagulants ar considered.****In TIA asso focal symptoms resolve in <24 hr. DDX1:Reversible Ischemic Neurological Deficit, cahracterized by a transient ischemic attack followed by resolution of the symtoms in 24hrs to a week. In Hemorhagic or Complete IA, symptoms dont resolve as quickly.

Tic Borne Diseases
1- Lyme, 2-Ehrlichinosis (RMSF):Fever,malise,HA &Vomit. 3-Babesiosis:endemic in Long Island. Occurs in pt >40, SPLENECTOMIZED, or immuncompromised. Parasyte enters RBC and cuases hemolysis. . Progression is Jaundice, hemoglubinuria, renal failure and death. Unlike othe rtick diseases RASH IS NOT a symptom.. Hay intravesculr hemolysis, anemia, abnormal LFT, elevated ESR. Dx is with blod smear. Tx is either Quinine-Clidamycin or Atovaquone-Azitromycin.

Tinea Corporis
Ring shaped scally patches with central clearing and distinct borders, topical tx with 2% antifungal lotions and creams (terbinafine) or systemic tx with Griseofulvin (for extensive disease).

Tinea Versicolor
Velvety pink or whitish, hypopigmentaed macules that don’t tan and don’t appear scally, but scale on scraping. Agent is Malasezia furfur. On KOH shows "spagetti and meatballs", Topical Selenium sulfide lotion or Ketokonazole shampoo is recommended.

TMJ Dysfunction
Always think of refered talgia when pt presents with ear pain and no hx. TMJD is a cc of refered otalgia, and pain aggravated by chewing, with psychogenic grinding (bruxism).

Torret Synd - 2
Tx is TYPICAL antipsychotics, like Haloperidole or Pimozide. *** Pts are at risk of developing OCD (keep repeating the same gestures and obsessed about counting the same numbers).

Torsade de pointes - 2
Could be caused by Quinidine. Tx is first stop the drug, then increase the heart rate by Magnesium. Torsade de pointes (see Fig. on DT), or twisting of the points, causes symptoms and death in patients with the rare congenital long QT syndromes. Its importance in everyday practice is its provocation by drugs (especially antiarrhythmics, which are contraindicated in its further management) or electrolyte imbalance. Management is to stop all cardioactive drugs (eg, antidepressants, antiarrhythmics, phenothiazines), normalize electrolytes (particularly K and Mg). ***TDP is also termed Polymorphic VT. its a very rapid VT characterized by gradulay changing QRS, prolonged QT. The mcc is Quinidine. In the acute setting Mg replacemnet is the tx.

Torus palatinus
is a benign bny mass on the palate. Its basically an outgrowth of the hard palat. No medical or surgery is required. Cause is unknown. So in a young person who present with a fleshy immobile mass on his hard palateits most likely Torus Palatinus

Toxic Adenoma - 2
Increased thyroid hormones with suppressed TSH are indicative of Thyrotoxicosis. Furthermore if radioactive Iodine uptake increased only in one lobe, that confirms Toxic Adenoma. DDX with Grave's is that in Graves iodine uptake is diffusly increased not just a specific area, also have exophthalmos. DDX with Multinodular goiter, uptake is patchy. DDX with Painless Thyroiditis, uptake is markedly reduced.***** Increased hyperthyroid pt like TA are at increased risk of rapid bone loss. Direct effect of thyroid hormone on th ebone cells eventually leads to increased OsteoClastic bone resorption.

Toxic epidermal necrolysis
A life-threatening skin disease in which the epidermis peels off in sheets, leaving widespread denuded areas. Primary manifestation is an erythomatous morbilliform eruption that rapidly evolves into exfoliation of the skin. Patches of skin slides off with slightest pressure (Positive Nikolsky sign). Oral mucosa shows painful blisters. Could be caused by Sulfanamides, barbituates, phenytoin, NSAIDS. Tx is supportive. Sulfanamides could also cause Stevens Johnson's syndrome (Erythema multiform major), but typical lesions are "target" shape. Same with Erythema multiform minor.

Toxic shock Syndrome
symptoms include sudden onset of flu-like syndrome, high fever, hypotension, erythomatous rash. 1-2week after onset skin peels. Tx: Pt should be hydrated and debridment of the wound should be performed. All source of infection should be removed and anti staph antibiotics should be started.

Trachoma
MCC of blindness in the world. Dx by presents of lymphoid follicales on the conjunctiva , scarring and limbal follicles. Tx is erythromycin or tetracycline.

Transient Sinovitis of the Hip:
Inflammation and swelling around the hip joint. Cause is unknown but could be due to virus. Bed rest with the affected joint in a comfortable position is the treatment of choice. Pt gets better in 3-4 days. DO NOT give aspirin due to the fear form Reyes Syndrome.

Transplantation
Oral Trimethoprine-Sulfamethoxide is the DOC for preventing PCP in transplant pts. If hay allergy then Dapsone.

Transposition of great vessels
The mc cyanotic heart disease that presents with cyanosis w/I the first 24hr of life. TOF chows cyanosis after few years.

Trichomonas Vaginalis
T. vaginalis is a flagellated protozoan found in the GU tract of both men and women. The organism is usually pear-shaped .Copius Malodoros vaginal Greyish-green color, thin and frothy. Also vaginal pruritis, dysurea and dyspareunia. Petechial pathes on the cervix show "strawbery cervic". Dx by microscopic obervation of flagellad organism in wet mount preparation. Tx is either one dose 2000mg or 250mg 3X a day for 7 days, for both partners. Metronidazole is teratogenic avoid using in 1st trimester.

Tricuspid Atresia:
Is a cyanotic congenital heart disease characterized by cyanosis early in life and left axis deviation. Most cases, 90% , are asso with VSD. DDX is TOF.

Tricuspid Regurgitation
A pansystolic murmur at the left sternal border

Tricuspid Stenosis:
A mid diastolic rumble, best heard along the left lower sternal border

Tricyclic Antidepressant drug tox:
Is the mcc of hospitalization and death due to excessive ingestion of prescription drugs. Pt with TCA overdose presents with anticholinergic, CV and neurological symptoms. CV se is proonged QRS and AV block. Also dry mouth dilated pupils, decreased bowel sounds, urinary retention, constipation, tachycardia, flushed skin and hyperthermia. Its been realized that QRS interval more reliably predicts the level of toxicity than the serum or urine drug levels.

Trigeminal Neuralgia - 2
At surgery or autopsy, intracranial arterial and, less often, venous loops compressing the trigeminal nerve root where it enters the brain stem have been found, suggesting that the tic is a compressive neuropathy. Paroxysmal lightning pain on the face, tx is carbamazepine.

Trimeta-Sulfa Toxicity
In African-Americans it cann cause acute hemolysis, back pain due to G6PD deficiency. NOTE:vigniette says that "G6PD I snormal", Don’t fall for that because in these pts it is normal but the cuase is still G6PD def. This also happens with Primaquine.

Torus Palatinus:
Is a benign bony mass on the hard palate. No medical or surgery is required. Cause is unknown. Tx is reassurance.

Tropical Sprue
Blunting of the villi and hx of travel is DDX with CELIAC disease. A common presentation is the triad of sore tongue, diarrhea, and weight loss. All features of a malabsorption syndrome may develop. Steatorrhea is common, and D-xylose absorption is abnormal in > 90% of cases. Folic acid and vitamin B12 deficiencies lead to megaloblastic anemia. shortening of the villi and lengthening of the crypts, with changes in the surface epithelium and an inflammatory cell infiltrate of lymphocytes, plasma cells, and eosinophils. The best treatment is tetracycline

TTP
presents with Pentad of 1-severe thrombocytopenia, 2-Microangiopathic hemolytic anemia (RBC fragments), 3-Neurological signs, 4-Renall failure, 5-Fever. LDH is elevated, PT & pTT are normal. HUS is like TTP w/o neuro signs. Both need Emergent Plasmaphoresis. DDX:I T P, a dx of exclusion, pt presents with isolated decreased in platelet count. Hb and WBC are normal, Pt & Ptt are normal. DDX3IC, RBC fragments, PT,PTT and BT are elevated. Renal failure is not a feature.

Tuberous Sclerosis:
Initial presentation is seizures. The cutaneous anomaly is called adenoma sebacum, which appears b/e 5-10 yrears of age. TS occurs in first year of life with clusters of brief symmetrical contractions of the neck , trunk and extremities known s ‘infantile soasms, demonstrating EEG patterns. There is also hyperpigmented lesions (Ash leaf) and cortical tubers on head CT. The DOC is IM ACTH.

Tubo-varian abscess
Is seen in 10% of pt w PID. Admit the pt, Broad spectrum antibiotics should be started immediately AFTER taking cultures. Gentamycin+Clindamycin+Ampicillin. In absence of obvious response w/i 48 hours , drainage should be considered. If there is doubt re Dx we do Laparoscopy. If hay rupture we need to do exploratory Laparotomy.

Tumor Lysis Synd - 2
Is characterized by Hypreuricemia. Seen in pt undergoing chemotherapy, with high nucleic acid turn over such as Leukemia dn Lymphoma. Prophylactic Allopurinol is the most effective method to prevent gout in these pts. Hysration is also good but alone is not enough, it should be used with Allopurinol.****Tumors which have high cell turn over are frequently ass with TLS, like Burkitts, ALL and AML. There is Hyperphosphatemia, Hypercalcemia, HyperKalemia and Hyperuricemia. The reason is both K and PO4 are intracel so they increase, PO4 causes HypoCa and Degradation of cellular protein causes increased Uric acid.

Turcot synd
It refers to an asso b/w brain tumors (primarily medulablastoma and gliomas) and FAP ( Familial Adenomatous Polyposis) or HNPCC (Hereditary Non Polyposis Colorectal Cancer. Its autosomal recessive and occurs mostly in teens. DDX1: Gardner, An auto Dominant, colonic polyps are seen with extraintestinal lesions, like Desmoid tumors, sebacious or epidermal cysts, lipomas, osteomas (mandible) gastric polyps and nasopharyngeal angiomas. DDX2: Peutz-Jeghers, an auto dominant dis, intestinal hamatomatous polyps with cutanous melanocytic macules. DDX4:Multiple Hamartoma synd, asso of GIT hamartomas and breast cancer, thyroid cancer and gingiva hyperplasia.

Turner synd
Since all ova are lost by atresia before menarchi, pts have streaked ovaries. There are no estrogen production, as there are no functinal filicular cells in streak ovaries. Absence of follicular cells results in absence of estrogen and inhibin from the ovaries as a result of loss of feedback inhibition of estrogen on LH & FSH causing their elevation. Inhibin only causes feedback inhibition of FSH, so in the absence of inhibin, serum FSH are more than LH, which is pathognomonic for Primary Ovarian Failure. **** Variation are possible, as in one X and one Y Ch. In this case the pt needs a bilateral gonadectomy due to increase risk for gonadoblastoma.*****Cxr shows rib notching due to coarcation of the aorta.

Ulcerative colitis - 3
Toxic Megacolon is a complication of UC. Its an emergency and prompt admin of IV steriods, nasogastric decompression& fluid management is required. Pt presents with diarrhea and bloddy stools, Dx is established clinically and colonoscopy. As it progresses, the rectum looses its elasticity and lumen collapses. Once tx is sarted some pt remit but some go on and manifest wieght lost and fever, when this acute colitis symptoms are associated with radiologic evidence of coloninc dialatation its called toxic megacolon.****Pts with UC that have pancolitis, should begin surveilance colonoscopy after eight years of diease.

Ulcerative Esophagtis
CMV is the most frequent cause in HIV pt. Triad of focal substernal burning pain with Odynophagia, shallow superficial ulceration and intracellular incluisons are dx of CMV (tx is gancyclovir). If ulcers are caused by HSV(tx is Acyclovir) they are multiple, well circumscribed and look like Volcanos.

Ulnar Nerve entrapment
Decreased sensation to 4 & 5 digits, and weak grip. The mc site is the elbow, in medial epicondyl groove.

Umbilical cord compression
is asso with Variable deceleration. Fetal sleep presents with decreased logn term variability. Fetal Head compression is asso with EARLY deceleration. Uteroplacential insufficiency presents with LATE deceleration. Intrauterine infection presents with tachycardia.

Umbilical Hernia
Is due to imperfect closure or weakness of the umbilical ring. Commonin low weight and Black infants. Its covered with skin and is easily reduced. > most disappear by one year. Surey is advised if hernia persists to the age of 3-4, exceeds 2cm in diameter, causes symptoms, becomes strangulated or progresily enlarges after 1-2 years.

Urethral injury, posterior - 2
Hallmark of Urethral injury is triad of 1-Blood at urethral meatus. 2-Inability to void, 3-Distended bladder. We have two types of UI: 1-Anterior, urethral inury anterior to perineal membrane. AUI are mc due to blaunt trauma to perinuem (saddle injuries). An immediate surgert is tx. 2-Posterior urethra, consists of prostate urethra and membaneous urethra. PUI are mc asso with pelvic fracture and high riding prostate. Initial mgmnt is retrograde urethrogram and then suprapubic catheterization. Remember Urethrogram with post void film is for Bladder injuries.

Urethritis
A/01/03. Tx for uncomlicated infections, Tetracycline, Azithromycin. For Pregnant women Erythromycin.

Uric acid stones
Are visualized via CT of abdomen, or IV pyelography.*****Needle shape crystals on urine analysis indicate urc acid stones. Uric acid stones which are radiolucent have to be evaluated with either CT of abdomen ar IVP. The stone can cause Ileus, possibly due to vagal reaction due to ureteral colic. The result will be absent bowel sounds, constipation, signs of obstruction, no gas passing. Ileus will be over once ureterolithiasis is tx. Stones

Uterine Rupture - 2
Presents with intense abdominal pain asso w vaginal bleeding, ranging from spotting to massive hemorrhage.****Risk in transverse line is 0.5% and in Vertical its 5.0%. If pt does not want any more children, total hysterectomy is the Tx of xhoice. If she wants more kids then Debridment and closure is indicated.****difficult to ddx from abrupta placenta. UR is preceeded by agitation, rapid breathing, tachycardia.

UTI in females - 3
The mcc in order are: 1-Shorter urethra compare tomen, 2-Contraceptives, 3-wt periuretral envoronment, 4-close proximity to anus.*****E.Coli is the mcc but it dos not produce uease so it doesn’t alter the normal Acidic pH of urine. Proteus is a urease producing bacteria and a frequent cause of UTI. Protease makes the urine Alkaline. ****In a female that there is no response to trimetasulfa and yellow/green mucopurulent endocervical dischage and edematous friable cervix, suspect C.Trochomatis. Dx is cervical culture. Asingle doze of Azithromycin for both partners is tx of choice. DDX is Chlamydia Urethritis, suspect in sexulay active pt with prolonged dysuria (7-10) days unresponsive to therapy for Chlamydial CYSTITIS. In Chlamydia, Gonocoal prophylaxis in not needed, but in Gonococal infection, empiric tx of chlamydia is recommended. Hospitalization and IV tx is recommended for severe PID (abdomial pain, uterine.adnexal tenderness on bimanual exam, and cervical motion tenderness).

Uveitis
Is the mcc of red eye in pt with Ankylosing spondolytis.

Vaginismus - 2
Involuntary spasm of the perineal musculature that interferes with sextual intercourse.****Unlike Dysparunia, Vaginismus is not due to any medical cause. In ALL cases its due to ignorant of women's anatomy, ahuge apprehention about penetration, and an uncontrolable fear of pain. Tx includes relaxation, Kegel excercise (to relax vaginal muscle), and graduation dialation (penetration).

Vaginitis, Candida
Its not an STD. It may appear in presence of risk factors like DM, OCP, pregnancy and immunosupresive therapy. It can also trigger by Alkaline pH of the vagina during menses that is favorable for growth of Candida. Symptoms are vaginal and vulvar pruritis, burning and dysparunia. Discharge has cottage cheese appearance. Its NOT malodorous and pH is 4-4.5. Dx is by KOH wet mount preparation and visualization os pseudohypha and spores. Tx is Imidazole cream or 10 day oral Ketoconazole (especially if recurrent). Fluconazole is the new drug with fewer SE. Give 2 doses, a week apart. Most recurrences are due to non-compliance. If that is OK, then suspect anoterh oraganism and culture on Sabourausd. 50% of reccurence is due to other organism like like T.Glabrata and Candida tropicalis. In addition, 35% of pt with recurrent yeast infection and who have negative cultures, have HPV on biopsy. DDX1:Trichomonas (Metro) DDX2:Gardenella Vaginitis (Metro).

Varicose Veins
Symptoms are fatigue, tiredness in the leg, aching swelling. They are worsend by prolonged sittingor standing.The sweeling and pain rapipdly goes away when leg is elevated. The edema is asso with perforator incompetence. Ulcers occur in medial aspect of the leg. DDX Arterial insufficiencyresents with claudication that is worsened by leg elevation. Pulse is weak or absent and the ulcers are deep and at the distal parts of the leg. There is NO SWELLING in arterial insuff but hair loss and muscle atrophy is noted. DDX2 DM ulcers are like arterial but they are at the pressure points. DDX3 DVT related ulcers take years to develop and they are unlikely in active pts.

Vasa Previa - 2
Hemorrhage w fetal heart rate changes, progresing from tachycardia to bradycardia to a sinusoidal pattern. "Apt" test distinguishes maternal from fetal blood. Mortality risk is 75%. Dx w transvaginal US in combination w Doppler. VP necessitates emergency C section.

Ventricular Tachycardia
When a pt has recurrent VT first thing to do after stabilization(Amiodarone) first thing to do is to look for the cause, Most probabelt its due Diuretics (ie Furesamide causes HypOkalemia). So order electrolytes and correct them if needed.

Ventriculat Fibrilation.
Tx is defibrilation with 200-360 joules. If it fails, lidocaine or Amiodarone (DOC) can be loaded and pt shocked again. Epinephrine can sensitize and lower the treshold for conversion.

Verapamil poisoning
May cause bracardia, hypotension, AV block and cardiac arrest. Like hypotension caused by other poisons, IV saline is the first thing. Then Calcium Chloride is useful in reversing the effect of Calcium channel blocker including bradycardia, AV block and hypotension.

Vesicoureteral Reflux:
Is retrograde flow of urine from bladder to ureter and renal pelvis. It’s a risk for recurret UTI and renal scarring ultimately. Renal scaring is the major cause of end stage disease in children. Reflux exists in all children with renal scars and renal scars are in 60% of children with gross reflux. Dx is made with voiding cystourethrogram.

VIPoma
Is a Pancreatic tumor. Produce Vasoactive Intestinal Peptide. It causes diarrhea, hypokalemia resulting in leg cramp, and decrease in amount of acid in stomach. Woman are more affected. Other symptoms include weight loss, facial flushing and redness. Dx is high VIP in blood. CT or MRI find location. Tx: first goal is to treat dehydration. Next is to slow the diarrhea with Octreotide. If tumor has not metastasized, surgery is indicated.

Vitami K def - 2
Vit K def presenting as hemorrhagic dis of the new born occurs in infants born at home with no prenatal care. Its given to all infants after birth. Newborns are suseptible due to low fat stores, low vit K breast milk and liver immaturity. It can present as intracranial bleeding as well as skin and GI. In adults it is seen in pts with chronic small bowel disease, after small bowel resection or prolonged anbiotic therapy. Dx is elevated Pt or decreased clotting factors. BT is normal. Platelet in normal, PTT may or may not be elevated. DDX1:Von Willerbrand, Auto Dom, BT & PTT are elevated, PT is normal. Factor VIII is low. DDX2:Hemophilia ia X-link, PTT is elevated, BT and Pt are normal. DDX3:Liver disease, Pt is elevated, PTT & BT are normal. DDX4:ITP, platelets are low.

Vitamin D Def
Most commonly due to MALABSORPTION causes reduced plasma Calcium and Phosphorous. DDX w Hyperparathyroidism and Pseudoparahypothyroidism is that they cause decreased in Ca but increase in phosphate.****In infants: presents with bone abnormality like softness or deformity. Typical pts ar low birth weight, Unsupplimented dark skin infants, infants with inadequate exposure to the sun, and breast fed infants.Pthology is defective mineralization of the bone.*****Rickets is developed due to Vit D def in children who stay indoors all the time and eating is not blanced. Presents with costochondral junctions are enlarged, wrist and ankle are thick and xray shows cupping and frying of the distal end of radius and ulna, large anterior fontanel.

Vitamin D Toxicity, Misc. 6/2
Confusion, HA, nausea from hypercalcemia. Mental status changes. Constipation is the mc GI SE. Renal SE is nephorolithiasis, ARInsiff and CRI. Tx is stopping the vitamin, low Ca diet, keep urine acidic and giving corticosteriods.

Vitamine Def
1-Riboflavin Def (B2) is unusual in industrialized nations. Symptoms are Photophobia, Dermatitis and Anemia. 2-Vit C : Ecchymosis, petechia, bleeding gums, Hyperkeratosis, Sjogren's. 3-Niacin (B3) presents with Diarrhea, Dementia, Dermatitis. 4-Thiamin def (B1) asso with infantile and adult Beri Beri as well as Werneki-Korsakoff. Dry beri beri is symetrical peripherla neuropathy with sensory and motor involvment. Wet beriberi is includes neuropathy in addition to cardiac involvment.

Vitiligo - 2
Presents with pale patches of skin, symetrically around themouth and areola. Borders are well circumscribed and hyperpigmented. Pathology is destruction of melanocytes. Few experience spontaneous remission. ***Its asso with Pernicious anemia, Grave's dis, DM-I, alopecia areata.

Vitreous Hemorrhage -2
Sudden loss of vision with floaters in the visual field. Diabetic Retinopathy is the mcc. Fundus is hard to visualize. Basically there are 3 types of Ophtalmological Hemorrhages:1-Conjunctival, cause is minor trauma,cough, sneeze or strain, no tx needed, resolves w/i 2 weeks. 2-VH, produces blavk reflex in exam, cause is DM and retinal vein occulsionretinal tears,its absorped slowly, bleeding is controlled with photocoag. 3-Retinal Hemorrhage which is flame shaped in HT and dot&blot in DM or septic infarction, its always significant.

Volkman’s Ischemic Contracture:
Displaced anterior fat pad is a radio-G sign of Supracondylar fracture which may be complicated by VIC

Volvulus, midgut
Present in a child <1 month old with billus vomit, abdominal distension and passage of blood stained stools. Child is irritable but no signs of pain. DDX1:Pyloric stenosis is non bilus vomit. Duedal Atresia has no blood in stool. In Down synd infants. Hirshprung fails to pass stool. Meconium Ileus, fails to pass meconium, asso with CF.

Von Willerbrand disease:
Vwf is decreased.

VSD
Harsh and hollosystolic murmur heard at the left lower sternal border, is the mc cardiac congenital defect. If its small and causes no symptms for the infant, 40% of the times it closes spontaneously by 3 years and 70% chance of closure by 10years. Risk of Infective endocarditis is increased so peophylaxis for dental is indicated. If the defect is large (Soft murmur) then it might turn in to Eisenmenger syndrome and reverse of the blood flow which is irreversible. Medx and surgery is then indicated prioe to irreversible stage.

Vulvar Hypertrophic Dystrophy
Thick and hyperkeratotic due to long term scratching. Mostly seen in PM women.


Vulvar Papillomatosis
or Condylomata acuminata. Caused by HPV 6,11. Present as exophytic lesionswith araised papilloatous or spiked surfacemay grow into a calliflowerlike formation.

Waldenstrum Macroglobulinemia
is a rare chronic plasma cell neoplasm. Plasma cells multiple and invade BM,spleen and lymph nodes. They produce excess IgM that causes thickening of the blood, Hyperviscosity. Major S&S are: 1-Increase size of spleen, liver, lymph node, 2-Tiredness (from anemia) 3-Bleed easily(little platelet), 4-Night sweats, 5-HA and dizzy, 6-Visual problems, 7-Pain and numbness in extremities. DDX:MultipleMYeloma, there is IgA/G not IgM. Also No hyperviscocity (Engorgment of the retianl vein) like in WM.

Warfarin induced skin necrosis - 2. Dermo. 6/3
Ita a serious complication of oral anticoags. Sometimes its asso with protein C deficiency. Female are more prone. Its a hypercoagular state.Commonly affected sites are breast, thighs, buttucks and abdomen. Initial complain is pain followed by bullae and skin necrosis. It occurs w/I weeks of starting therapy. VitK should be administered in the early stage and Warfarin discontinued if lesion progresses. Heparin should be used until necrotic lesion heals. **** Note os the dise: Warfarin treated pts should be given FFP instead of VitK when emergency surgery is to be performed. DDX with Cholesterol Embolic synd, dont be fooled with Bruit, see the whole picture.

Washing cells
Febrile reaction is a common transfusion reaction that is caused by antibodies in the pt's plasma reacting with donors leukocytes. Leukocyte depletion technique, like cell washing , decreases the probability of febrile, NON-hemolytic transfusion reaction.

Waterhouse-Friedrichsen
Or Adrenal Glands Hemorrhage is a very serious complication of Menigococcial meningitis caused by Nisseria. Its bilateral bleedng into adrenal gland due to DIC, carries 100% mortality. So in an infant with Meningococcemia watch out for WFS, which is characterized by a sudden vasomotor collapse and skin rash due to adrenal hemrr.

Wegener granulomatosis - 2
Hematuria+Hemoptysis+Sinusitis. Underlying pathology involves Necrotizing vasculitis with granuloma formation. Postive C-anca. Typical Xray shows nodular cavities. ****Demonstration of Anti Neutrophil Cytoplasmic Antibodies confirm dx. The Cytoplasmic pattern (C-ANCA) is highly specific. Cxr shows infiltrates, nodule, cavities and masses.

Werdnig-Hoffman:
An auto recessive disorderinvolves degeneration of anterior horn cells and crnial nerve motor nuclei. It causes floppr baby synd (like Botulism)

Wernicke encephalopathy
Characterized by Ataxia, opthalmoplegia, nystagmus, and altered mental status. It's seen in chronic alcoholics due to B1 (THIAMINE) deficiency. DDx1:is Folate def which is common in alcoholics but wont prestn with B1 def symptoms. Remember B3(NIACIN) causes Pelegra, 3D, due to corn base diet. B2(FLAVIN) causes Cheilosis an Glositis.

Whipple's disease - 4
Caused by bacterium T. Whippelli, affects men 30-60yo. Presents with joint pain + abdominal pain + diarrhea + weight loss. PAS+ foamy macrophages containing a glycoprotein in intestine is Dx. Also skin Hyperpigmentation. Untreated dis is fatal and progressive. Tx is Procaine pencillin G followed by tetracycline. *Dx is by PAS staining, it can be confirmed with upper GI endoscopy with biopsy on the small intestine. ***D-Xylose absorption is abnormal in both Whipple and bacterial overgrowth. However, the test becomes normal after antibiotic tx.

Whopping cough - 2
B. Pertusis. Tx Erythromycin and cough suppresants. Hospitalize if serious. Pt might develop rectal prolapse, epistaxis and Pulmonary pathology.*****Its highy contagious. Presents with severe bouts of coughing spells after an URI. They are so severe that they can cause rectal prolapse, epistaxis, pneumothoraces and subconjunctival hemorrhage.


Wilms Tumor - 2
B/t ages of 2-5. It’s the mc primary tumor of renals in childhood. 80% have asymptomatic abdominal mass discovered by mother. In 10% massis bilteral. Tx is Nephrectomy. . If tx at early age majority have long term survival. DDX1:Neuroblastoma, if the child has <1 year suspect it. It presents with abdominal mass, fever, weight loss, hepatomegaly and HT. It usually crosses the mid line and dx is made by measuring urinary cathecholamine products. ****4yo with right flank mass, and both kidneys being palpable with stable vital signs is Willms. Thi stumor orignates from METANEPHRONS. DDX is Neuroblastoma, a unilaterla mss, w/o bilateral enlargment of kidneys. This tumor is a malignancy of neural crest cells. DDX2:Polycystic kiney dis presents with BILATERAL flank masses.

Wilson's disease - 2
Dx is decreased serum ceruloplasmin, increased urinary copper and slit lamp examination of the eye. There is also Mallory's hyaline on liver biopsy. Its recessive disease. Decreased ceruloplasmin leads to copper deposition in liver, cornea, basal ganglia (hepatolenticular degeneration). Disese starts in infancy, but since all infants have increased level of copper for the first 3 months, its best to diagnose it after the first year. f not treated thi disease is fatal.****Dx is finding Ceruloplasmin ,increased urinary copper and hepatic copper deposition in liver biopsy. Tx is Penicilamine . Fulminant Hepatitis needs liver transplantation.

Wiskott Aldrich
Characterized by recuurent Pyogenic infections, atopic eczema and thmbocytopenia. Presents w/I one year of age. First manifestationis hemorrhagic episodes followed by pyogenic infections. Its X-linked. IgM is decreased but IgA & IgG are normal. The defecy lies in inability to produce immune response to capsulated polysaccharide antigens. Pt are predisposed to ALL and Hodgkins. Thrombocytopenia is due to spleenic desrtuction of Platelets. Tx is continues antibiotics, spleenectomy and BMT. DDX1:CGD, presents under 2 years of age, defect is inability to generate oxydative burst to kill organism after being phagocytosed. Lack of NADPH oxidase. In lab, it can be confirmed with Nitro Blue Tetrazolium, which will be NEGATIVE. DDX2:Chediak-Hegashi is recurrent pyogenic infections. the defect is in neutrophils as their lysosomes fail to empty their contents. B&T cell function are normal. NBT is normal. Pt will have partial Albinism and neuropathies and Nystagmus.

Wrist fractures:
1-Colles: the mc fractureof distal radius in outstretched hand. Characterized by dorsal displacement & dorsal angulation.
2-Smith : reverse of Colles. 3-chauffer’s: fracture of radial styloid process. 4-galleizi: isolated fracture anywhere in radius with asso injury to radio-ulna joint.

Zenker Diverticulum - 2
Pt presents with orophareangeal dysphasia, neck mass and are >50yo. UES dysfunction and esophageal dysmotility (motor dysfunction and motility problem) are believed to be the cause. Barium exam helps to delineate the diverticulum, the surgical tx includes excision and frequently cricophareangeal myotomy. ****Zenker is defined as herniation of mucosa through the fibers of cricopharyngeal muscle. Barieum Esophagograpghy is the confirmatory test od choice, not Esophagoscopy.

Zinc Def
Alopecia, mental changes, diarrhea, dysgeusia, smell abnormalities, maculopapular rash around the mouth and eyes and impaired wound healing.

Zollinger Ellison (Gastrinoma) -4
Characterized by multiple severe drug resistant ulcerations in GI track. The mcc of ZE is a gastrin-producing pancreatic tumor. As a result of uncontrolled gastrin production, parietal cell hyperplasia occurs and stomach acid production is significantly increased. Multiple Duedenal ulcer are typical, and a Jejunal ulcer is almost pathognomic for this condition. Steatorrhea may develop, becuase increased production of stomach acid inactivates pancreatic enzymes. Tx of choice is K-H atpase inhibitor Omeprazole, indefenitely until surgery. If medx fails we have to go to toal Gastrectomy. *Is Asso with MEN-I, Hyperparathyroidism,Pancreatic carcinoma and Pituitary tumors.